Jump to content

Wikipedia:Reference desk/Humanities

From Wikipedia, the free encyclopedia

This is an old revision of this page, as edited by 82.31.133.165 (talk) at 12:06, 26 October 2012 (→‎Child slavery in nineteenth-century America.: new section). The present address (URL) is a permanent link to this revision, which may differ significantly from the current revision.

Welcome to the humanities section
of the Wikipedia reference desk.
Select a section:
Want a faster answer?

Main page: Help searching Wikipedia

   

How can I get my question answered?

  • Select the section of the desk that best fits the general topic of your question (see the navigation column to the right).
  • Post your question to only one section, providing a short header that gives the topic of your question.
  • Type '~~~~' (that is, four tilde characters) at the end – this signs and dates your contribution so we know who wrote what and when.
  • Don't post personal contact information – it will be removed. Any answers will be provided here.
  • Please be as specific as possible, and include all relevant context – the usefulness of answers may depend on the context.
  • Note:
    • We don't answer (and may remove) questions that require medical diagnosis or legal advice.
    • We don't answer requests for opinions, predictions or debate.
    • We don't do your homework for you, though we'll help you past the stuck point.
    • We don't conduct original research or provide a free source of ideas, but we'll help you find information you need.



How do I answer a question?

Main page: Wikipedia:Reference desk/Guidelines

  • The best answers address the question directly, and back up facts with wikilinks and links to sources. Do not edit others' comments and do not give any medical or legal advice.
See also:




October 17

Out of state discrimination laws in the US (part 2)

Looking at the previous thread on the subject, I want to ask a couple more general question. There are many companies that are incorporated in one state but have employees physically located in another. Have there been any cases in which a person (successfully or otherwise) sued their employer for violations of labor or other laws in the state of incorporation, rather than the state the employee is physically located in? Is there a general legal principle on how such cases are decided? Thanks. Someguy1221 (talk) 00:23, 17 October 2012 (UTC)[reply]

To a certain extent there might be jurisdiction shopping involved. What will really matter is the specific act. If something happened in your home state that was a local issue, the local law will apply. It would only be if there were some sort of higher level corporate malfeasance, say (in my imagination as a layman) a policy decision from the home office not to address a manager transferred from another office with a known problem, that there might be a case under that state's jurisdiction. Your lawyer would have to argue that before a judge to keep him from dismissing it as having no standing (law) in that court. It will really depend on the specifics and, since it will be a civil case, whom you are suing for what. The defendants are also likely to argue to the presiding judge that the case should be handled locally, assuming allowing it to proceed elsewhere isn't to their advantage. μηδείς (talk) 00:40, 17 October 2012 (UTC)[reply]
The US Constitution makes the federal courts available to people who are suing residents of other states, and obviously all state courts are open to their residents who wish to sue others who are present in those courts' jurisdictions. I don't expect that a court in the company's home state would listen to a suit regarding actions in another state, but the availability of both federal and state courts (and the consequent jurisdiction shopping that Medeis mentions) is the general legal principle. Nyttend (talk) 00:46, 17 October 2012 (UTC)[reply]
According to Federal jurisdiction (United States), Federal courts can take cases under state laws if the parties are residents of different states and the question involves damages meeting a certain threshold, currently $75,000 according to federal law. μηδείς (talk) 00:55, 17 October 2012 (UTC)[reply]
Oh, yes, I forgot about there being a minimum amount in question; you can't take your out-of-state company to federal court just because it negligently caused you to lose $5 somehow. Nyttend (talk) 02:11, 17 October 2012 (UTC)[reply]
A Federal court sitting in diversity jurisdiction will apply the substantive laws of the state in which the court sits. That in turn means they'll look to the state's choice of law provisions to see which state's laws apply. I think most state courts will also entertain suits from a non resident against a resident business, with some possible caveats. It varies quite a bit, but that's not uncommon at all.Shadowjams (talk) 17:59, 17 October 2012 (UTC)[reply]

People who look extremely similar

Which people look extremely similar? And please limit it to famous/notable people in the same field (politics, et cetera).

I know of William McKinley and Mark Hanna and of Elizabeth Warren and Michele Alliot-Marie. Futurist110 (talk) 02:25, 17 October 2012 (UTC)[reply]

There are some ideas at Look-alike. -- Jack of Oz [Talk] 03:20, 17 October 2012 (UTC)[reply]
(edit conflict) Do a Google search for the phrase "Separated at birth" and you'll get a shit load of websites dedicated to this subject. Two that I can think of that oddly look alike from American football is Indianapolis Colts center Jeff Saturday and former New York Giants center Shaun O'Hara. That the two snapped footballs to the Manning Brothers for several years has been brought up, given their similar appearance. SeeJeff Saturday andShaun O'Hara.google search turns up a few notable sports journalists who have noted the striking similarity in appearance, including Adam Schefter and the Bleacher Report. --Jayron3203:26, 17 October 2012 (UTC)[reply]

They say that the Mwanza Flat Headed Agama and Spiderman were separated at birth. Sources =[1] --Jethro B 03:55, 17 October 2012 (UTC)[reply]

Geena Davis and Tanya Memme come to mind. (For more than one reason.) Mark Twain and Kurt Vonnegut. Karl Malden and Richard Herd. There was a short book series a few decades back with lookalikes, some of them whimsical, for example Mick Jagger and Don Knotts' character "Mr. Limpit". One of my early efforts here was to try to create a page called "Separated at Birth", but it was shot down as original research. I bet it could be done now, with strict sourcing. ←Baseball Bugs What's up, Doc?carrots04:04, 17 October 2012 (UTC)[reply]

In the news here in the UK: Gary McKinnon and Benedict Cumberbatch - several celebrities have been joking about the resemblance on Twitter in the past 24 hours. (Obviously they are in different lines of work, but Cumberbatch is famous for portraying social misfit and investigative geniusSherlock Holmes, and recently also portrayed maverick statistical expert Christopher Tietjens in Parade's End.)AlexTiefling (talk) 09:53, 17 October 2012 (UTC)[reply]
I hear Benedict Cumberbatch looks an aweful lot likeCummerbund. μηδείς (talk) 18:16, 17 October 2012 (UTC)[reply]
Kofi Annan tells a good story about how he was on holiday once in an isolated location, where he thought he would be away from the public eye. But an autograph hunter came up to him and said "I really admire your work, Mr Freeman." Not wanting to spoil the guy's day, Annan signed the autograph "Morgan Freeman". --Viennese Waltz 10:15, 17 October 2012 (UTC)[reply]
Another one. At a White House reception in honour of Duke Ellington's 70th birthday on 29 April 1969, Richard Nixon mistook Cab Callowayfor Ellington and warmly congratulated him on his 'birthday'. Not sure what Cab said in return. Maybe he sang "Minnie the Moocher" in Mood Indigo. -- Jack of Oz [Talk] 12:09, 17 October 2012 (UTC)[reply]
Tina Fey did a near perfect impersonation of Sarah Palin (you might say one is in entertainment and the other in politics, but those fields seems to overlap considerably, at least in this case). StuRat (talk) 19:24, 17 October 2012 (UTC)[reply]
It gets eerier: Jeremy and Jason were born on exactly the same day, and not only that, in exactly the same place. Incredible! Another feather in the cap for numerology and astrology. (Cough) -- Jack of Oz [Talk] 21:44, 17 October 2012 (UTC)[reply]
The girl from CSI (er, not the main one) looks a lot like Kelly Clarkson. Adam Bishop (talk) 00:17, 18 October 2012 (UTC)[reply]
I had never heard of Kelly Clarkson, but having now seen a picture of her, I wonder if by "the girl from CSI (er, not the main one)" you meanAnna Belknap from CSI: NY. Angr (talk) 00:22, 18 October 2012 (UTC)[reply]
Me and Brad Pitt, common mistake. Shadowjams (talk) 09:36, 18 October 2012 (UTC)[reply]
Really ? And which part of his body does your face resemble ? :-) StuRat (talk) 14:06, 25 October 2012 (UTC) [reply]

Paul Ryan has made at least one speech at The Atlas Society where he spoke about his love for her philosophy. Is he a member of the society? Is he a former member? 69.62.243.48 (talk) 04:27, 17 October 2012 (UTC)[reply]

It seems unlikely, not that Ryan would be intellectually sympathetic to the society, but that he could have been a member without it being made public. Being an Objectivist does not play well with the American voting public: liberals dislike the reverse communism, and conservatives dislike the atheism. Ryan is smart enough a politician to avoid formal ties with such a group, and the election is close enough that President Obama or his allies would have made this known if it were the case. --BDD (talk) 15:14, 17 October 2012 (UTC)[reply]
this talks about that speech, but doesn't answer your question. I am amused to note that Ryan called Rand's works "required reading in my office for all my interns and my staff", which I would call a well-deserved punishment. Wnt (talk) 17:34, 17 October 2012 (UTC)[reply]
Actually, just googling paul ryan ayn rand will give you a few very useful pieces (I like the top 3) on the relationship between Ryan and Rand's philosophy. While his admiration for that philosophy has been consistent, so have been his attempts to distance himself from it in certain ways. So again, I think it's very unlikely he's a member of the society. By the way, at least one of those results describes the society as a think tank, so its membership may be more professional researchers rather than anyone who adheres to Objectivism. --BDD (talk) 18:04, 17 October 2012 (UTC)[reply]
It's all rather obvious, isn't it? Rand Paul, Ayn Rand, Anne Rice, Ian Rankin, Paul Ryan, Ron Paul, RuPaul, Pope John Paul II ? It doesn't take rocket science to figure out what's going on here. Wake up and vote before it's too late, people!μηδείς (talk) 19:49, 17 October 2012 (UTC)[reply]
"Mitt Romney and Paul Ryan" is an anagram for "My ultimate Ayn Rand porn." Coincidence?209.131.76.183 (talk) 19:10, 19 October 2012 (UTC)[reply]
Paul Ryan is the vampire love child of Pope John Paul II and RuPaul?
Yes, well, here's the proof. μηδείς (talk) 02:45, 21 October 2012 (UTC)[reply]

Global companies

Why is it that large global companies such as CocaCola have significantly different product ranges worldwide and they don't share their products? For example, Coca Cola USA has products CocaCola UK don't produce and they don't share it with them. Clover345 (talk) 10:39, 17 October 2012 (UTC)[reply]

Coca cola products are often directed at American consumers' unique eating habits. Ankh.Morpork 11:22, 17 October 2012 (UTC)[reply]
The primary consideration when deciding what products to sell where is what products you think people want to buy where. Tastes vary from country to country (based largely on what people are used to, I think). --Tango (talk) 11:26, 17 October 2012 (UTC)[reply]
But with CocaCola products, surely this is not really a significant point of consideration as most of their products are sweet fizzy drinks?Clover345 (talk) 11:30, 17 October 2012 (UTC)[reply]
If Americans don't particularly want to buy the kinds of sweet fizzy drinks that Italians like, what would it benefit Coca Cola to sell the same sorts of drinks in both places? --Jayron32 11:44, 17 October 2012 (UTC)[reply]
Some products are specifically targeted at a particular market such as its tea and coffee products. In Japan, Coca Cola have focused on the tea market and their Marocha Green Tea is a highly popular product. Another example of regional focus would be Kvass sold primarily in Russia. Also, the branding of the products varies according to the regional differences. While fruit juices in the US and Europe are sold under the Minute Maid and Disney juices brand names, similar products are sold as Qoo in Asia, Kapo in Latin America and Bibo in Africa. Also, Coca Cola have also purchased companies that operate on a national scale. e.g Kola Inglesa dominates the Peruvian market because that is where it was originally produced. Ankh.Morpork 11:57, 17 October 2012 (UTC)[reply]
I've never seen Disney branded fruit juices in Europe. Astronaut (talk) 12:48, 17 October 2012 (UTC)[reply]
I think the products were eventually discontinued. I was citing it as Winnie The Pooh Roo Juice was an obvious example of targeted branding.Ankh.Morpork 13:22, 17 October 2012 (UTC)[reply]
Even Coca Cola itself is made differently in different countries. I live in the UK and enjoy Diet Coke, but the variety they sell in Europe is far too sacharriny for my tastebuds. --Dweller (talk) 12:59, 17 October 2012 (UTC)[reply]
To answer the general Q, let's list reasons to sell the same product under the same brand worldwide:
A) Less expensive to only design, produce, and market a single product. How much this is a factor will vary by product. For something with minimal development costs, like soft drinks, it's not much of a factor. For something with huge development costs, like commercial aircraft, having a significantly different model for sale in each nation would be impractical.
B) Provides more flexibility. If demand is higher than production in one nation, it's then easy to bring in product from other nations with overcapacity.
Now let's list reasons to have different products:
1) They may have acquired local companies, which already have a customer base for their product. Continuing to sell that product may also generate less resentment, than, say, if you fire everyone and then try to sell your own imported product there, instead. The local government may also have laws to prevent this type of thing.
2) As mentioned previously, different tastes exist in different places. In addition, there may be cultural or religious taboos, like not eating pork in certain places.
3) There can also be differences in infrastructure. A different type of car is appropriate for a place where gasoline/petrol is cheap, distances are long, and highways are straight and level, than a place with high fuel prices, and narrow, twisting, hilly roads.
4) Local laws may require a different product. For example, labeling requirements and safety regulations may vary. It is sometimes possible to make a single product for the most stringent requirements, but not always, as laws may be incompatible, or such a product may be too expensive to compete in nations lacking the strict laws.
5) Language differences also require different labeling, product names, etc. StuRat (talk) 18:57, 17 October 2012 (UTC)[reply]

Is longevity hereditary?

I ask this because my grandfather (1845 – 1940) lived up to 95 while my father (1895 – 1955) died at 60. Is longevity hereditary?Iowafromiowa (talk) 11:34, 17 October 2012 (UTC)[reply]

Well, before you answer the question, you'd need a sample size greater than one! google search turns up some promising leads for study. --Jayron32 11:42, 17 October 2012 (UTC)[reply]
The first serious-looking article that I found says; "While most studies confirm the prior belief on the existence of an intergenerational correlation in mortality, they do not have the required data to quantify the relationship in a statistically reliable way, let alone study how this relationship changes with the father’s age at death." The Hebrew University of JerusalemLONGEVITY ACROSS GENERATIONS . Alansplodge (talk) 12:07, 17 October 2012 (UTC)[reply]
The reason of death is also relevant. If one died in an accident you can stop thinking about genetics. Groupask (talk) 12:50, 17 October 2012 (UTC)[reply]
Statistically, over a large sample maybe not. Risk aversion, impulsiveness, etc. could all have a hereditary component. -- Q Chris(talk) 14:02, 17 October 2012 (UTC)[reply]
You also need to live long enough to get hit by the car. Changing the probability of dying by one cause changes the probabilities of dying by every other cause because it changes the amount of time you are exposed to that risk. --Tango (talk) 11:24, 18 October 2012 (UTC)[reply]
Yes, there is definitely a hereditary component to longevity - plenty of studies have shown that. However, there are so many other factors and also so much randomness that it's pretty meaningless to try and apply it to individuals. On average, people with long-lived parents will live longer, but you need a very large number of people before the other factors start to average out. --Tango (talk) 11:22, 18 October 2012 (UTC)[reply]
But one thing that's definitely applicable to individuals is parenthood (or lack thereof). If your parents didn't have any children, then neither will you. You can bet your (haunted) house on it.  :) -- Jack of Oz [Talk] 17:46, 18 October 2012 (UTC)[reply]
Are you sure Jack? I got banned from a "freethinking" website where the moderator claimed that according to natural selection every individual must be happy according to evolution. I told him that according to evolution, the only thing one's ancestors must have been is parents. I was called a collectivist because of that argument. μηδείς (talk) 02:42, 21 October 2012 (UTC)[reply]
Freethinking indeed: the moderator clearly did not feel bound by such things as conventional meanings of words. —Tamfang (talk) 22:09, 21 October 2012 (UTC)[reply]
Indeed. I don't know what "according to natural selection every individual must be happy according to evolution" means. But I'm afraid to ask for clarification. -- Jack of Oz [Talk] 07:57, 22 October 2012 (UTC)[reply]
What age did your grandmother reach? --NellieBlyMobile (talk) 16:55, 22 October 2012 (UTC)[reply]

Web-site answering questions

Which web-sites out there do answer questions? I'm not searching for a user to user site, but more of a traditional kind like in the past, with "Dear reader, ..." . I want something with high standards in writing. Groupask (talk) 12:38, 17 October 2012 (UTC)[reply]

Then why ask here ;) ? But seriously, what type of questions do you have in mind? That might help us direct somewhere suitable to your needs. Mingmingla (talk) 14:36, 17 October 2012 (UTC)[reply]
It sounds like you want something like Dear Abby. The reality is, there are far more people with questions than those willing to give researched, high quality, well written answers, for free. So, if you want answers for free, you'll have to post at a site like this, and take your chances on the responses you get. Yes, many may have poor grammar and spelling, or be off-topic, or jokes, but, with any luck, there will be good answers mixed in, too. StuRat (talk) 18:44, 17 October 2012 (UTC)[reply]
Yes, but also with serious questions. I don't necessarily want my questions answered, just interesting questions about science, history, humanities. The straight dope is an option. Groupask (talk) 19:09, 17 October 2012 (UTC)[reply]
I suspect that you'd find, with a site like that, that most questions remain unanswered. However, you can read through answers to other question. HowStuffWorks is a good one. StuRat (talk) 19:13, 17 October 2012 (UTC)[reply]
(edit conflict) I would definitely recommend The Straight Dope and the Straight Dope Message Boards http://boards.straightdope.com/sdmb/. SDMB has been around much longer than Wikipedia (I was active there in the 1990s) and though it has been a few years since I was active, IIRC it has a culture of being fairly well researched and thoughtful. Other sites like "Yahoo Answers" and "Wiki Answers" are a total crapshoot. --Jayron32 19:16, 17 October 2012 (UTC)[reply]
Google answers used to be really good, because the askers would pay so the answerers tried to come up with the best answer - thorough and exactly what was asked for. It got closed down but if you are simply looking to read interesting questions and answers the archive is still readable.http://answers.google.com/answers/ --184.147.123.169 (talk) 21:31, 17 October 2012 (UTC)[reply]


Try physics forums. manya (talk) 06:47, 18 October 2012 (UTC)[reply]
Stackexchange. 20.137.2.50 (talk) 12:54, 18 October 2012 (UTC)[reply]
For questions on Internet rumors, there's Snopes.com. StuRat (talk) 02:59, 19 October 2012 (UTC)[reply]

Married young in Victorian England?

I'm doing a little research into my great-grandparents. They are both listed in the census taken on 2 April 1911, with their ages and children (one of whom is my late grandmother). The funny thing is, when I try to find their date of marriage the obvious record suggests a marriage when my great-grandmother was only 15 years old. Would such a thing have been possible in 1890's England (maybe if she was pregnant)? Of course, maybe I have the wrong marriage record or maybe there are errors in the way in which their names are recorded, or in the 1911 census data, but I am intrigued by the possibility of marriage at such a young age in Victorian society or possible reactions of parents to an impending child being born out of wedlock. Astronaut (talk) 12:45, 17 October 2012 (UTC)[reply]

Is her birthdate known from records other than the census? My experience is more with the US than the UK or Commonwealth census, but census takers often carelessly put down an age that was a few years off, or the person reporting their age would sometimes make themselves younger for vanity, or older to avoid revealing that they had a child at an extremely young age, or the censustaker got their info from a neighbor who was guessing about details. I've seen cases where a woman married very young, but the first known child came years later, and have speculated that she might have married due to a young pregnancy but the first baby was a miscarriage or stillbirth. Many records were just sloppy. Edison (talk) 14:41, 17 October 2012 (UTC)[reply]
The Criminal Law Amendment Act 1885 had raised the age of consent to 16 - so it would have been illegal to marry a 15-year-old in the 1890s.
However, I have recently been doing quite a lot of research on these censuses, and I'm familiar with their quirks. Unlike prior censuses, to which Edison's remarks about the enumerators would apply, the 1911 census was filled in by householders wherever possible. This reflects the vastly increased levels of literacy that had been achieved by that date. Consequently, if the census sheet is signed by the head of the household (rather than just with an X), the ages given may be taken as the ages that that person believed the members of the household truly were.
The census was taken on 2 April 1911. So make sure, in your calculations, that you allow for your great-grandmother's birthday to be as early as 3 April. Conversely, allow for the marriage to be as late as possible in the quarter it's reported for. (Obviously, if you order the certificate from the GRO (£9.25), you'll get the exact date, as well as her alleged age at the time.) If her name is sufficiently distinctive, you may be able to get hold of her birth certificate, or a baptism record, which will give you a precise indication of when she was born. The marriage certificate will also mention her father's name and occupation, which is a great help in finding births, baptisms, and censuses taken during the subject's childhood.
If you want, I could try researching your question directly, if we are able to arrange a way of passing personal information.AlexTiefling (talk) 15:01, 17 October 2012 (UTC)[reply]
Currently, people in England and Scotland need to be at least 16 to marry with parental consent, 18 without. I could not find any information on England (though edit-conflicting Alex, above, did), but apparently in Scotland, men age 14 and girls age 12 could marry without parental consent until 1929. And young couples used to flee to Gretna Green to take advantage of these lax laws. So marrying at 15 does not seem to be implausible for the UK. See Marriage in Scotland. --Stephan Schulz (talk) 15:03, 17 October 2012 (UTC)[reply]
Our article about the 1885 Act strongly implies (see 'Repeal') that it applied to the entire United Kingdom, not just to England and Wales (as Hardwicke's Act of 1753 had done). In any case, the principal reason for elopement to Gretna Green was to escape the Hardwicke Act's requirement for parental consent for under-21s. I don't believe that the contracting of marriages which would be illegal in England and Wales on account of the bride's age was a major motivator for the Gretna Green Dash; nor do I think that such would have been legal by the 1890s. The 'any witness' form of marriage allowed in Scotland would not have circumvented the law on the age of consent. AlexTiefling (talk) 15:24, 17 October 2012 (UTC)[reply]
Hmm. Our article Marriage_in_Scotland#Border_marriages says However in Scotland it was possible for boys to get married at 14 years and girls at 12 years without parental consent [...] In 1856 Scottish law was changed to require 21 days' residence for marriage, and since 1929 both parties have had to be at least 16 years old (though there is still no parental consent needed). Looking over the 1885 Act, it talks about "unlawfully and carnally know"ing girls under the age of 14 and 17. As far as I can tell, this does not affect lawful marriages. The age of consent is not necessarily the same (or lower) than the age of marriage. Indeed, apparently one of the points brought against the 1885 act was a possible conflict with lower legal ages for marriage. --Stephan Schulz (talk) 16:17, 17 October 2012 (UTC)[reply]
It's somewhat ironic that Google presents me with a dating ad sporting an Asian women that looks borderline legal under the 1885 act on the page that has the text: [2]. --Stephan Schulz (talk) 16:24, 17 October 2012 (UTC)[reply]
Depending on what were you searching for, Google spots your intentions, and show ads that you would like to see. I won't ask what it is.83.60.246.149 (talk) 22:23, 17 October 2012 (UTC)[reply]

There are several references on the net to the 1763 Marriage Act in England, in which " minimum age of marriage was fixed at 16. Prior to this date, the church accepted the marriage of girls aged 12 or more and boys aged 14 or more. In addition, a dispensation on licence could be obtained from a bishop which allowed marriage at a younger age."[3] Alansplodge (talk) 17:02, 17 October 2012 (UTC)[reply]

How did Google get bigger than Stanford (moneywise) when Stanford owned the Page Rank patent?

The Page Rank algorithm was assigned to Stanford. Before Page and Google et al were big and had money and an army of lawyers, how did they get Stanford to give them such a favorable exclusive rights license that would allow them to reach financial escape velocity when, at that early moment, with the patent assigned to them, Stanford had all the cards? Benevolence? Where can I read (if it's available) how the license deal was negotiated? Peter Michner (talk) 16:27, 17 October 2012 (UTC)[reply]

Page Rank is only a very small part of Google, and search engines used to come and go every few months. Nobody could foresee the success of Google, and a more restrictive license may well have stopped Google cold, leaving Stanford with no money at all. Moreover, Page and Brin, as the actual inventors, were in a much better position than anybody else to evaluate (and utilize) the patent. --Stephan Schulz (talk) 16:54, 17 October 2012 (UTC)[reply]
It's also important to remember that Google's search engine function is something of a loss leader; its main job is to get your eyes on its ads. Search alone doesn't make any money for Google; that the search directs viewers to target ads does make money. Google has also branched out considerably into many tech fields entirely unrelated to search engines at all. It is a multi-service company with its fingers in many pies. The search engine gave it the initial leg up, but where Google has flourish where others have failed is largely Google's ability to grow outside of the search engine market. Google's story is basically one of leverage, using its search engine technology to build itself into a business based on actual money making ventures (i.e. all the non-search stuff they do). --Jayron32 17:41, 17 October 2012 (UTC)[reply]
It's tough to make predictions, especially about the future. With hindsight, PageRank is clearly a winning idea and Google's founders have turned out to be the kind of people who can transition successfully from academia to a profitable business. Technology-heavy universities like Stanford, MIT, and Cambridge are always spawning startups like Google, with staff and students and IP that came from the university. So they have an office of IP licencing, who frankly guess how much to charge, or what proportion of the stock of a new company they can ask for. I defy anyone to sit in an office all day and read scientific papers and patents and really have a confident idea of how much that invention is, or could reasonably be, worth, and so how much to charge for it. For a startup, usually all they can ask for is stock (because the company has no cash), and owning stock means they don't end up looking super-stupid in the off-chance a company soars (that they only made $336M is because they didn't hold on to the stock for longer). 99%+ of technology startups crater, and the very few people who seem to have a knack at distinguishing which won't are almost always living on a yacht somewhere. Remember also that angel investors and VCs are very thorough about finding details of what they're buying into and how it could possibly be encumbered - they'd run a mile from a Google which only had say a 3 year licence on patents on its core technology. Stanford, being experienced at this kind of thing, knows they can't write terms that would drive off such investors, as they'd just strangle the startup at birth and kill their own chance seeing returns from it. -- Finlay McWalterTalk17:15, 17 October 2012 (UTC)[reply]

Perverted Justice (moved from Science Refdesk)

[moved from [4]] - Wnt (talk) 16:46, 17 October 2012 (UTC)[reply]

I was reading the wikipedia perverted justice article and Im confused as to how they convicted these people if the decoy was actually over 18? It even says in the article

"The father of a man arrested in a July 2006 sting by Perverted-Justice has appeared before a Georgia Superior Court judge to seek an arrest warrant for Von Erck, alleging that Von Erck solicited the commission of a felony from the man.[77][78] The judge found that probable cause existed to believe Von Erck impersonated a girl and solicited the man with the intent for him to commit a felony, but declined to issue the warrant because when the act occurred, there was no actual girl and thus no crime occurred.[77] The act of soliciting a felony is itself a felony.[78] Attorney Gary Gerrard filed an appeal with the Georgia Court of Appeals, alleging that under Georgia law, solicitation is a felony whether or not a crime occurred.[79] That appeal was ultimately rejected.[80]"

Something dosent add up.--Wrk678 (talk) 10:17, 17 October 2012 (UTC)[reply]

The law is often explicitly written so that such stings can result in convictions. It sounds like that wasn't the case in Georgia, but it may well be the case in other states. There may be a specific offence of attempting to sollicit that says it is the age the offender thinks the person is rather than their actual age. Or it may just be a matter of how attempted offences are defined in that jurisdiction. --Tango (talk) 11:32, 17 October 2012 (UTC)[reply]
The legal term is Entrapment, and "sting" operations get very dicey when it comes to entrapment. --Jayron32 17:37, 17 October 2012 (UTC)[reply]
I would like to point out, however, that Entrapment explicitly applies to law enforcement officers (in the United States). And in fact, the third sentence of our article on entrapment reads "However, there is no entrapment where a person is ready and willing to break the law and the government agents merely provide what appears to be a favorable opportunity for the person to commit the crime," which is usually how law enforcement skirts around these issue with Perveted Justice. Come on sinebot, where were you.Livewireo (talk) 19:23, 17 October 2012 (UTC)[reply]
It is dicey, which is to say that it isn't always clear cut. First of all, courts have generally recognized that unsworn citizens, when working at the explicit aid of law enforcement, are considered to have been "deputized" and are effectively law enforcement themselves. That is, law enforcement departments can't summarily get around restrictions placed on their operation by outsourcing questionable activities to non-law enforcement personnel. If someone is performing an investigation in cooperation with, or at the behest of, law enforcement, they are held to the same standards as the sworn officers. Secondly, entrapment is fuzzy around the edges, and this is one of those edges. There is no bright line between "gives someone the idea to commit a crime" and "provides a favorable opportunity to commit the crime". --Jayron32 19:04, 17 October 2012 (UTC)[reply]

I believe that the guy who was trying to get Von Erck arrested in the quoted section above was convicted, so how is that possible if a judge says "there was no actual girl and thus no crime occurred." ?--Wrk678 (talk) 20:49, 17 October 2012 (UTC)[reply]

Female head of state and head of government

Was the period when Margaret Thatcher was Prime Minister of the UK and Elizabeth II was Queen the first time in history that a country in which the head of state and the head of government were two different people had women in both those roles? Has it happened since? Please note I'm not asking about the situation where the head of state and head of government are the same person, only the situation where they are two different people.Angr (talk) 21:01, 17 October 2012 (UTC)[reply]

Queen Elizabeth is the monarch of several of the members of the Commonwealth of Nations, some of whom who have had female heads of government during her reign (Indira Ghandi, for example). Does that count? 69.62.243.48 (talk) 21:12, 17 October 2012 (UTC)[reply]
In theory, yes, but not for that example, since India was a republic with a president while Indira Gandhi was PM. (To be precise, India had a series of male presidents and acting presidents during Indira Gandhi's PM-ship.) But her article led me to Sirimavo Bandaranaike, who was PM of Ceylon while Elizabeth was its monarch, and who was also the world's first female head of government, so there can't have been another occurrence before her. So now only the second part of my question remains: has it happened again since Thatcher/Elizabeth? Iceland has had a female president and a female PM, but not at the same time. Angr (talk) 21:26, 17 October 2012 (UTC)[reply]
There's a discussion further up this page about a related topic ("Four top dogs..."). The short answer to your last question is "yes". Finlandhad a female president and PM simultaneously not long ago, too. AlexTiefling (talk) 21:23, 17 October 2012 (UTC)[reply]
(edit conflict)x2 Queen Elizabeth has never been the Head of State of India. That role has been taken by the President of India since 1950. See List of Presidents of India. None were women during Indira Gandhi's administration. As far as answering the question, the OP could cross reference List of elected or appointed female heads of government with List of elected or appointed female heads of state and known Female hereditary monarchs, which are fairly easy to keep track of. The answer is that the pairing that predates Thatcher & Elizabeth II was in 1960, with Sirimavo Bandaranaike as Prime Minister of Ceylon and Elizabeth II who was Queen of Ceylon at the time. --Jayron32 21:24, 17 October 2012 (UTC)[reply]
In fact, I see Finland had the situation twice. So we have Elizabeth/Bandaranaike in Ceylon (as was), Elizabeth/Thatcher in the UK, and Halonen/Jäätteenmäki and later Halonen/Kiviniemi in Finland. Any others? Angr (talk) 21:32, 17 October 2012 (UTC)[reply]
Just found Margrethe II/Thorning-Schmidt in Denmark, which is in effect right now. Angr (talk) 21:39, 17 October 2012 (UTC)[reply]
As is Elizabeth II and two of her realms: Jamaica (Portia Simpson-Miller) and Australia (Julia Gillard) currently. --Jayron32 21:47, 17 October 2012 (UTC)[reply]
(ec x 2) It has certainly happened since.
  • Chandrika Kumaratunga became Prime Minister of Sri Lanka in August 1994. In November she was elected President, and she appointed her motherSirimavo Bandaranaike as Prime Minister. This was Bandaranaike’s third time as PM; she had become the world’s first female head of government in 1960 (when Sri Lanka was Ceylon and still a Commonwealth Realm), following the assassination of her Prime Minister husband. A real family affair.
  • a number of Commonwealth countries have had female governors-general and female prime ministers, although not all at the same time. As discussed here recently (see "The 4 top dogs are ladies"), in 2005-06 New Zealand had females simultaneously as Queen, Governor-General, Prime Minister and Speaker of Parliament; and since very recently Australia has had the same. -- Jack of Oz [Talk] 21:37, 17 October 2012 (UTC)[reply]
And someone redlinked by the rather female-sounding (and delightfully multicultural) name of Lucinda da Costa Gomez-Matheeuws was apparently very briefly PM of the Netherlands Antilles while the monarch was Juliana. Angr (talk) 21:42, 17 October 2012 (UTC)[reply]
Margrethe II of Denmark was "sort of elected" as future head of state in the Danish constitutional and electoral age referendum, 1953 where female monarchs became possible. She was 13 at the time but a lot more popular than her uncle Knud, Hereditary Prince of Denmark who was heir presumptive. Without the constitutional change, Knud and his son (who was also 13 in 1953) would have become successive kings since 1972. The unpopularity of both of them was a significant cause for holding the referendum and getting it passed. Margrethe II has been a popular head of state since 1972. Helle Thorning-Schmidt became prime minister in 2011. PrimeHunter (talk) 01:47, 18 October 2012 (UTC)[reply]

Duke of Rothesay, Earl of Carrick, Baron of Renfrew, Lord of the Isles and Prince and Great Steward of Scotland

If the eldest child of Prince William and Duchess Kate is a girl, will the Scottish titles of the heir apparent be held in abeyance until such a time as there is a male heir apparent to the throne, perhaps a generation later? 69.62.243.48 (talk) 22:05, 17 October 2012 (UTC)[reply]

Firstly, those titles are those of the Duke of Cambridge's father, so they're not going to be bestowed on anyone else for a bit. Secondly, the situation changes depending on whether a new Act of Succession is passed. As CHOGM recently approved such a move in principle, it's highly likely. This is therefore the assumption under which I'm answering this question. (If there's no new Act of Succession by the time the Duke of Cambridge is King, the short answer to your question is "yes".)
As with the very similar recent question about the Duchy of Cornwall, the answer has to be: the Act of Parliament and Letters Patent enabling the change to the law of succession will spell this out, because the government and the Crown employ people who know this stuff much better than we do. I am, however, reminded that the Electress Sophia petitioned Queen Anne to be created Princess of Wales in her own right, and was refused. No matter how strong the presumption, the heir presumptive does not get the titles of the heir apparent. However, I think it highly likely that the succession to all the royal titles will be strictly equalised in the event of any new Act of Succession. The dissolution of the Union might occur before that, though, in which case the Scottish titles, dignities and privileges of the monarch and their heirs will be up for negotiation anyway.AlexTiefling (talk) 22:15, 17 October 2012 (UTC)[reply]
(edit conflict) It'll be a LONG time before that becomes an issue, and indeed, 2011 proposals to change the rules of royal succession in the Commonwealth realms has not passed yet, so no, they would not. As of right now, if the eldest child of the Duke and Duchess of Cambridge is a girl, she'll just be an (untitled) princess, and when and if William becomes king, she'll just be the heir presumptive, presuming no changes to the law and no sons are born to William. When and if William becomes King, if at that time his eldest daughter is heir apparent, because of the proposed changes actually passing, according to Duke_of_Rothesay#Legal_basis, the title just won't exist during that time period. It is simply conferred automatically on the eldest son of the reigning monarch. If there is no son, the title has no holder. Presumably, the lesser included titles follow the same rules. --Jayron3222:20, 17 October 2012 (UTC)[reply]
Yes, William can reasonably expect to become monarch about 30 years from now. His as yet unborn eldest child, in about 65 years. That's a long time for gossip mags to be twitteringly and gushingly grooming them for greatness (while relatively ignoring the actual heir, Charles, Prince of Wales), -- Jack of Oz [Talk] 23:04, 17 October 2012 (UTC)[reply]
Although if he lives as long as his grandmother, it'll be nearly 40 years before William acceeds. Alansplodge (talk) 00:39, 18 October 2012 (UTC)[reply]
"He" being Charles and "his grandmother" being Queen Elizabeth The Queen Mother. Just clarifying. -- Jack of Oz [Talk] 02:57, 18 October 2012 (UTC)[reply]
Exactly - it made sense to me anyway ;-) Alansplodge (talk) 10:24, 18 October 2012 (UTC)[reply]
Sense, yes. Just that it could easily have been read as entirely about William, the grandmother being a reference to the current Queen, who's 86 and going strong. Until you do the maths, and realise that doesn't work, so the first part must have been about Charles and his grandmother, not William and his. -- Jack of Oz [Talk]11:04, 18 October 2012 (UTC)[reply]
Not one of my most lucid posts; thanks for disentangling it. Alansplodge (talk) 12:19, 18 October 2012 (UTC)[reply]

"When and if William becomes King, if at that time his eldest daughter is heir apparent, because of the proposed changes actually passing, according to Duke_of_Rothesay#Legal_basis, the title just won't exist during that time period. It is simply conferred automatically on the eldest son of the reigning monarch. If there is no son, the title has no holder." However, having a daughter first does not prevent a person from having a son afterwards. William's heir apparent could be his daughter, but he could also have a son. I don't think the title passes to "the eldest son who is also heir apparent". It's much more likely that it passes simply to the eldest son. Therefore, the title could be conferred automatically on the eldest son who is not the heir apparent, which leads to a number of issues. Surtsicna (talk) 13:32, 19 October 2012 (UTC)[reply]

Or Parliament could just change the rules to suit. Alansplodge (talk) 15:52, 19 October 2012 (UTC)[reply]



October 18

Purpose of books on table in House of Reps

In the Australian House of Reps there's a bunch of books on the centre table.

http://www.smh.com.au/opinion/political-news/may-abbott-continue-says-gillard-20121016-27pm7.html

Do those books serve an actual purpose or are they there to make the place look more authoritative? Surely the clerks would have those records on a laptop?

101.169.85.83 (talk) 00:48, 18 October 2012 (UTC)[reply]

[5] - the best I can find - suggest that in the UK they'reHansard and books of parliamentary rules, such as, presumably, Erskine May. There is a cite, but to a defunct PDF location. I think we can take it that they're mainly decorative. Oz mirrors the UK, and as it turns out, visa versa ... the UK dispatch boxes came from down-under. --Tagishsimon (talk) 01:00, 18 October 2012 (UTC)[reply]
Thanks Tagishsimon. 101.169.85.83 (talk) 01:05, 18 October 2012 (UTC)[reply]
Here's the answer from the horse's mouth. As a long time observer of our Parliament in practice, I can't remember seeing anyone actually consulting those books. That's not to say they don't, though. Sometimes it is still easier and quicker to consult a familiar book than hunt through various computer links and breadcrumbs to get what you want. For example, if I wanted to check out some clause in the Constitution, I'd go straight to the book in my personal library rather than to any online copy thereof. -- Jack of Oz [Talk] 01:38, 18 October 2012 (UTC)[reply]

Red republicans?

Do US Americans find it strange that red is the official color of republicans? 83.52.248.109 (talk) 14:05, 18 October 2012 (UTC)[reply]

Not an answer, but see Red states and blue states for a (sort of) explanation. Ghmyrtle (talk) 14:18, 18 October 2012 (UTC)[reply]
I don't know that it's the "official color of Republicans" but rather the media-imposed shorthand for Republicans since 2000 (before 2000, the only real association between Republicans and red was the preferred color of Nancy Reagan's dresses). The thing is, that Red and Blue are the only two active colors available in the patriotic American palette of red, white, and blue, and if Red were assigned to the Democratic party, then this would give rise to all kinds of snickering and innuendo, so it has to be the other way around (even if this is opposite to UK political color symbolism).AnonMoos (talk) 14:25, 18 October 2012 (UTC)[reply]
But the republicans would have a vested interest in being identified with blue - to make the democrats look more communist/socialist/threatening. It's like saying that Obama is Muslim/Arab/threatening. 83.52.248.109 (talk) 14:38, 18 October 2012 (UTC)[reply]
And that's probably part of why the journalists ultimately settled on Republicans=Red, Democrats=Blue. If the Republicans wanted to reserve blue as their own particular color, then they should have done so before 2000 (but they didn't). AnonMoos (talk) 15:01, 18 October 2012 (UTC)[reply]
It's just coincidence. The various networks assigned arbitrary colors to the parties prior to each election and used the colors to make electoral college predictions and on election night for the results. Democrats had been assigned red in the past, and republicans blue, entirely snickerfree. I remember a green versus yellow scheme during the Reagan era. There is certainly nothing "official" going on--as if this were a decision by the BBC--and parties were not offered a choice--the decisions were made arbitrarily in news rooms. In 2000 there was a coincidental confluence of colors. This was noticed by the media and a convenient meme arose. See Fixation (population genetics). μηδείς (talk) 16:50, 18 October 2012 (UTC)[reply]
As I remember, in the 1992 and 1996 elections (the last two for which I was living in the States and actually watching the news coverage), which color was used for which candidate on the maps on the Election Night TV news coverage varied from network to network. I remember being very startled one Election Night when I changed channels from ABC to NBC and suddenly it looked like the other candidate was winning! Then I realized that NBC's use of red and blue were reversed from ABC's use of them. Angr (talk) 21:14, 18 October 2012 (UTC)[reply]
Medeis -- the Democrats could be assigned red in some ephemeral television graphic without much problem, but if there's going to be a long-term stable metaphorical assignment of Red vs. Blue, then assigning red to Democrats would bring in all sorts of unfortunate connotations, while assigning blue to Democrats causes much fewer problems (and Nancy Reagan can have her favorite dress color)... AnonMoos (talk) 21:46, 18 October 2012 (UTC)[reply]
I really don't think Americans associate red with communism much, anymore. Maybe older Americans, who lived through the various red scares. --Trovatore (talk) 21:54, 18 October 2012 (UTC)[reply]
It hasn't been very actively used over the last 20 years of U.S. politics, but I think that it's more prominent and ready to be reactivated (particularly among politically-involved types), when a suitable opportunity arises, than you may realize... AnonMoos (talk) 22:14, 18 October 2012 (UTC)[reply]
As a politically active free-marketeer since the 1980 US election--before I was old enough to legally vote--I can assure you, AnonMoos, that fewer than 95% of Americans my age have any actual awareness of any association of red with the left. I say this as someone who (1) has worn red symbolically as striking union member, and (2) who has no cognitive dissonance in seeing it as "representing" the GOp since the 2000 election. Were people as historically aware as one might wish them to be, perhaps it would be otherwise. But they aren't, so it isn't. μηδείς (talk) 22:55, 18 October 2012 (UTC)[reply]
Funny thing how 1980 is about the time that the "Reds" film was released, to great critical acclaim and reasonable box office. The red=communist connection is in the past, but it's in the living past, not the remote dead past, so that someone like Rush Limbaugh could get a great amount of mileage out of riffing on such a theme -- and a journalist might think twice about doing something that would provide the likes of Rush Limbaugh a source of cheap insults... AnonMoos (talk) 02:57, 19 October 2012 (UTC)[reply]
Well, I can't prove it didn't happen. But I like Medeis's "coincidence" explanation better, in the absence of evidence to the contrary.--Trovatore (talk) 04:14, 19 October 2012 (UTC)[reply]
I'm presuming people here read the article linked above as the first reply? It suggests albeit without a source that in 2000 initially there was less unification but this changed because of the long period of uncertainty meaning there was continual coverage and so the media began to conform on one colouring pattern. This doesn't rule out there being a greater confluence in 2000 then in previous years which may have aided the process but does suggest if 2000 had had a clear cut and undisputed on election day the confluence may not have taken hold. Nil Einne (talk) 05:55, 19 October 2012 (UTC)[reply]
Trovatore -- I'm sure that chance played a role in various transitory and ephemeral color schemes for TV graphics, but if at some point it had seemed like a possibility that a lasting Blue=Republicans / Red=Democrats scheme could be established, then probably someone would have asked "Do we really want to label Democrats as ‘Reds’"? -- while if it seemed possible that a lasting Blue=Democrats / Red=Republicans scheme would be established, then there would be no similar question to be asked... AnonMoos (talk) 07:42, 19 October 2012 (UTC)[reply]
I think you're speculating. It's not clear to me that anyone would have thought that important enough to bring up.--Trovatore (talk) 08:26, 19 October 2012 (UTC)[reply]

As an Australian, I'm perfectly happy to solve the problem by loaning the American parties our national colours of green and gold. (Although green might be unacceptable to those who hate environmentalists.) HiLo48 (talk) 22:45, 18 October 2012 (UTC)[reply]

HiLo48, other parties have already taken those colors. The Libertarian Party uses yellow (close to gold) as the primary color in its logo, and the Green Party obviously uses green.    → Michael J    00:35, 19 October 2012 (UTC)[reply]
I clearly remember Professor Sibley wearing red ties the way others wave flags. He wasn't a Communist, but according tothis wore red to "to remind himself and others of his solidarity with the working class and the socialist movement." Zoonoses (talk) 05:00, 19 October 2012 (UTC)[reply]
Died the year The Berlin Wall fell. Poetic justice? Heartbreak? ...Could it be...Satan? μηδείς (talk) 02:37, 21 October 2012 (UTC)[reply]

Neckties in presidential debate

Following from the question above, the first thing I noticed in the debate was that Obama was wearing a red necktie and Romney a blue one -- the reverse from the first debate. That surely wasn't a coincidence. Has anybody seen any discussion of why they decided to do that?Looie496 (talk) 15:20, 18 October 2012 (UTC)[reply]

One place it isn't is the Debate MoU, in which they seemingly nail down every single other possible question. I'm envisioning their staff members meeting in a neutral hallway and flipping a coin, but there's no reason it couldn't have been done by an earlier informal phone call either. A simple Google.com search for obama romney necktie color yields plenty of pages making the same observation and an excellent unrelated first result, but none I've clicked on so far actually has any reasons why they would have done so. ☯.ZenSwashbuckler.☠ 16:58, 18 October 2012 (UTC)[reply]
ThisSlate article discusses the question, and says that it is simply coincidence -- but I don't believe it. Looie496 (talk) 17:14, 18 October 2012 (UTC)[reply]
It would have been hilarious if Romney had come out in white tie. Then at least 47% of the voters would have been seeing red, and the Republicans would have been blue. Clarityfiend (talk) 00:02, 19 October 2012 (UTC)[reply]

Relegion ,

In which roman catholic semenarys would roman catholic priests have been trained for work in plymouth between the years 1850 to 1900 — Preceding unsigned comment added by86.160.170.17 (talk) 15:16, 18 October 2012 (UTC)[reply]

Do you mean Plymouth or some other place called Plymouth? If you meant the city in Devonshire, then a quick look through Wikipedia shows that Catholic priests in the south of England would have trained at St. Edmund's College, Ware until 1869, when a new seminary was founded at St Thomas's in Hammersmith in London. Thereafter, each diocese began to establish their own colleges; a scheme to have a central seminary at Oscott College in Birmingham seems to been abandoned in the early 1900s.[6]. St. John's Seminary (Wonersh) opened in 1891, serving the Archdiocese of Southwark. The Diocese of Plymouth falls within the Ecclesiastical province of Southwark, so maybe there's a connection there, perrhaps someone with a better understanding of the RC hierarchy in England could comment. AlansplodgeAlansplodge (talk) 17:30, 18 October 2012 (UTC)[reply]
The IP geolocates to Launceston, Cornwall, so Plymouth, Devonshire, does seem most likely. Angr (talk) 21:09, 18 October 2012 (UTC)[reply]
By that time, there was a fair amount of mobility for clergy in the UK, for both Roman Catholics and Anglicans. So you might very well expect to find a priest working in Plymouth who had been trained in France, Italy, or elsewhere. If you're interested in a specific priest, or the priest(s) at a specific institution, you'd be well advised to find the relevant Catholic Directory for the time. Had you been interested in Anglican priests, you'd find things nicely centralised in Crockford's Clerical Directory, of which larger reference libraries in UK will have an archive (Crockford has a mini-bio of the career of each incumbent Anglican priest and official). Things RC don't seem to be quite as well ordered - there certainly was a publication for Scotland, Catholic Directory for the Clergy and Laity in Scotland, but I can't find an equivalent that would cover Plymouth. It might be worth a trip (or maybe a phone call) to the County Reference Library in Truro to see if they have the appropriate Directory. -- Finlay McWalterTalk 22:31, 18 October 2012 (UTC)[reply]

What happened in these 3 years of king Yeongjo reign?

  • 1729년: 기유처분
  • 1740년: 경신처분
  • 1741년: 신유대훈

I couldn't understand it, and I didn't find any information in Yeongjo article. — Precedingunsigned comment added by Sayom (talkcontribs) 16:23, 18 October 2012 (UTC)[reply]

unidentified London building/castle

Hi, does anyone familiar with London know what building this is part of? I took the picture a couple of years ago, and I was (judging by shots taken before and after) either near Tower of London or the Globe Theatre. Obviously Tower of London would seem like a good option, but I haven't been able to find on the web a portion of it that looks like this. Julia\talk 21:38, 18 October 2012 (UTC)[reply]

It's the Curfew Tower of Windsor Castle. Mikenorton (talk) 21:54, 18 October 2012 (UTC)[reply]
Ooh, okay, interesting. Apparently I did quite a lot of travelling that day that I don't remember! Thank you so much! Julia\talk 22:02, 18 October 2012 (UTC)[reply]



October 19

-pour Iranian christians

I notice that Iranians who are Christians have surnames that end with -pour like Christine Amanpour. Is that always that case? — Preceding unsigned comment added by 70.53.231.99 (talk) 03:05, 19 October 2012 (UTC)[reply]

I'm confused by your question. First, "Christian" is not an ethnicity, so I'm not sure that a specific surname would be connected to a faith like that. Second, I see no evidence at Christiane Amanpour that she or anyone in her family were Christians. So, unless you have more examples than that, I'm not sure that your question is based on any correct assumptions. Christianity in Iran indicates that many Christians in Iran are ofArmenian decent, so one may find a large number of Armenian surnames among Iranian Christians. But I don't know if Amanpour is an Armenian surname; many Armenian surnames I can think of end in -ian. (Ross Bagdasarian, Kim Kardashian, Avedis Zildjian, etc.) --Jayron32 03:19, 19 October 2012 (UTC)[reply]
If her family were Muslim, or Jewish, they would hardly have been likely to name her Christiane, I think. I suppose they could be non-religious, but it seems a good bet that they were at least "culturally Christian". --Trovatore (talk) 04:34, 19 October 2012 (UTC)[reply]
Yeah, but her father's name was Mohammad, so there is that. --Jayron32 04:38, 19 October 2012 (UTC)[reply]
Hmm. Good point. --Trovatore (talk) 04:46, 19 October 2012 (UTC)[reply]
It wouldn't surprise me if -pour meant "city", like the similar-sounding Indic language suffix... AnonMoos (talk) 04:05, 19 October 2012 (UTC)[reply]
According to this paper, "pour", or other similar spellings, is Persian for "son of", and is widely used in Iranian names, not specifically for Christians. Looie496 (talk) 04:14, 19 October 2012 (UTC)[reply]
Just to confirm: It is not at all the case that Iranian Christians have surnames that end with -pour (meaning "the son of"), and I'm not sure if Christiane Amanpour is Christian at all, though she is the daughter of a British mother. Arbitrary naming is not uncommon among Iranians.Omidinist (talk) 05:15, 19 October 2012 (UTC)[reply]
Our article says Amanpour attended a Catholic girl's school. Her father's name was Mohammed, her mother'sd Patricia. NNDB says she's Roman Catholic. μηδείς (talk) 05:33, 19 October 2012 (UTC)[reply]
She very well may be, but if so it has nothing to do with her surname "Amanpour". They are unrelated. --Jayron32 12:35, 19 October 2012 (UTC)[reply]
Indeed. So far the evidence suggests she got the name Amanpour from the Muslim side of her family, even if she herself grew up RC. Not that attending Catholic school is good evidence; a Hindu friend of mine attended a Catholic school in India. Angr (talk) 19:50, 19 October 2012 (UTC)[reply]
I am not aware that I attributed her father's family name to her religion. Did I? I was simply reporting information that seemed to contradict unsupported implications above it. μηδείς (talk) 21:00, 19 October 2012 (UTC)[reply]
No, you didn't. I assume her father's first name is an indicator of his religion, as it seems unlikely for anyone but a Muslim to be named Mohammed. And as Trovatore points out, it's unlikely for anyone but a Christian to be named Christine. (Or at least, in both cases, the child of anominal Muslim or Christian at the time the baby's name is given.) Angr (talk) 19:40, 22 October 2012 (UTC)[reply]

Ashkenazi Jew languages

So far, I know that Ashkenazi Jews speak German, Polish, Russian and Yiddish. What other languages do they speak?--70.53.231.99 (talk) 03:27, 19 October 2012 (UTC)Don Mustafa[reply]

They spoke most of the languages of central Europe and eastern Europe west of the Pale of Settlement, but in most areas Yiddish was their internal community language. Of course, all this is historical now that relatively few of them live in central or eastern Europe... AnonMoos(talk) 03:33, 19 October 2012 (UTC)[reply]
(edit conflict) They probably speak hundreds of different languages. Many Ashkenazi Jews migrated all over the world in the diaspora preceding, during, and after the Holocaust, and many probably migrated well before that. I'd imagine you can find Ashkenazi Jewish people speaking literally every European language. --Jayron3203:34, 19 October 2012 (UTC)[reply]
English and Hebrew are also two main languages that in the present day they speak (about 100 years ago, Yiddish wouldl've been more predominant). Of course, it depends the country they live in, which can be anywhere on the entire world. So we can't really restrict the languages someone can speak, in fact, someone can learn any language if they want to. --Jethro B04:12, 19 October 2012 (UTC)[reply]
Rusyn language. μηδείς (talk) 05:30, 19 October 2012 (UTC)[reply]
NOTE - it'd also depend the level of observance of the Jew in question. A Reform, Conservative (Conservative Judaism doesn't mean traditional or reactionary, it's a bit past reform on the spectrum), or modern-Orthodox Jew (traditional, religious Jew who incorporates the surroundings and the present-day era) are not as likely to speak Yiddish as an ultra-Orthodox Jew, where Yiddish is frequently spoken as the norm. --Jethro B 05:35, 19 October 2012 (UTC)[reply]
Dont mizrahi's speak yiddish too? cant be all Russian? Or ami talking out of my bum?Lihaas (talk) 16:09, 19 October 2012 (UTC)[reply]
No, because they wouldn't have been exposed to it, and Yiddish was largely suppressed in Israel (even for Holocaust survivors) from around the founding of the state and subsequent large influx of North African immigrants. Same for Sephardim, whose international tongue is mainly Ladino(also called Spaniolit). One of the hardships, often fatal, of the Thessalonika Jews deported to the Auschwitz concentration camp is that they couldn't understand what the German and Slavic camp staff were saying and had no language in common with the vast majority of their fellow inmates. -- Deborahjay (talk) 17:29, 19 October 2012 (UTC)[reply]

The reason I asked that question because I read the languages of Israel article and it said that Spanish is an Ashkenazi language due to immigration of Jews from Latin America. Also, it forgot Portuguese language being spoken in Israel because of Jews from Portugal (Sephardi) and Brazil (ashkenazi).--70.31.22.91 (talk) 23:49, 21 October 2012 (UTC)Don Mustafa[reply]

This prize is award to the best original full-length novel. I wonder what it means to be "original full-length novel"? Somehow I feel like the words "original full-length" are unnecessary. It could just be the prize for the "best novel" unless "original full-length" means something else. Thanks!184.97.240.247 (talk) 04:08, 19 October 2012 (UTC)[reply]

At the least it means no translations and no novellas or novelettes. Looie496 (talk) 04:17, 19 October 2012 (UTC)[reply]
(edit conflict) Well, "original" means that it can't be an adaptation from another work (for example, novelizations of films), and "full length" means that it can't be a novella. --Jayron32 04:18, 19 October 2012 (UTC)[reply]
Novella has a word count between 17,500 and 40,000. So how many pages is equivalent to that? Plus I don't understand how Happy Potter book doesn't win this award, it is the most popular book around the world. Why?184.97.240.247 (talk) 05:02, 19 October 2012 (UTC)[reply]
It depends on how many words there are to the page, and since books are published in many formats (hardcover, trade paperback, pocket paperback, e-books now), page has no standardized meaning, but word count should be fairly consistent. And the prize isn't awarded for popularity, it is awarded for quality, basically the same thing as the Academy Award for film. --Jayron32 05:07, 19 October 2012 (UTC)[reply]
Are you saying Harry Potter is not a quality novel? What makes Harry Potter so success is its quality. It didn't just success for no reason.184.97.240.247 (talk) 06:58, 19 October 2012 (UTC)[reply]
Who's to say the Harry Potter books haven't been considered? Lots of books get considered, but very few get onto the shortlist, and only one of those wins. -- Jack of Oz [Talk] 07:18, 19 October 2012 (UTC)[reply]
I never said that they weren't "quality" (for any arbitrary definition of quality), I was refuting your claim that they should have wonsolely because they were popular. It was not a criticism of Harry Potter novels (which I enjoyed thoroughly), but rather a refutation that popularity has anything to do with the Man Booker prize. --Jayron32 12:33, 19 October 2012 (UTC)[reply]
The Harry Potter books are, rightly or wrongly, normally considered children's books, and the Booker Prize is for adult fiction. Also, many would not consider them examples of what might be described as literary fiction, which is the raison d'etre of the Booker. --Viennese Waltz 12:31, 19 October 2012 (UTC)[reply]
In any case, if you take a look at the previous winners of the Man Booker prize you can see that Harry Potter is really not in the style of the others. These awards are chosen by judges who have some idea of what a Man Booker winner is like. Harry Potter, for all of its charms, is simply not that. If it was, it would never have appealed to children or even most adults. --Mr.98 (talk) 14:21, 19 October 2012 (UTC)[reply]
The website lists the terms and conditions. The most important rule may be 3 i): "The decision of the Literary Director as to whether a book is eligible shall be binding and no correspondence shall be entered into." Zoonoses (talk) 05:12, 19 October 2012 (UTC)[reply]
I think the Harry Potter omission is because those books aren't classed as fiction for adults, but as fiction aimed at teenagers. J K Rowling herself is currently publicising her first book written specifically for adults, which presumably will be eligible for entry for the Booker prize next year.--TammyMoet (talk) 12:32, 19 October 2012 (UTC)[reply]
Then I think the award should says "This is an award to the best original full-length adult fiction novel". It's funny how it wasn't an original goal of the prize but somehow it has only chosen "adult fiction novel". I think Harry Potter deserves to win than many of those books that won.184.97.240.247 (talk) 07:56, 20 October 2012 (UTC)[reply]
Well, think again! Children's books are explicitly permitted, but there's a condition:
  • Rule 3(g): Children's books will only be accepted on the condition that they have also been published by an adult imprint within the specified dates.
My assumption here is that children's books have, by definition, limited appeal, and the convenors want to recognise books that have broad appeal. Hence, as long as a children's book has been accepted by a publisher who is not marketing the work as particularly for children, it should be fine. I don't know that the Harry Potter books met this stipulation. -- Jack of Oz [Talk] 09:18, 20 October 2012 (UTC)[reply]
The longlisting of The Amber Spyglass demonstrated that children's/young adult fiction can indeed be eligible for the Booker (it also wonWhitbread Book of the Year in 2001, the first time the award went to a non-adult novel). The OP is fully entitled to admire the Harry Potter novels, but unless they have also read the actual Booker winners it's not clear how they can judge the former more deserving of the prize than the latter. - Karenjc 16:48, 20 October 2012 (UTC)[reply]

looking for the dumbest methodology ever ... well, sort of

Hi, as part of my research, I may do some qualitative research, but I have looked at some of the "official" paradigms out there, and they strike me as something of a joke (to wit, grounded theory). The problem is that they seem to all involve a huge amount of overhead from nailing down different categories and so forth, which, from what I have seen, does as much to stifle thought as to clarify it. I'm wondering if anyone knows of something really vague and general, which can be interpreted however you want, so as not to force me to waste time following some predefined scheme for how my thoughts are going to go. The closest I can find is thematic analysis, which seems so ill-defined even its practitioners don't know what it is (seehttp://eprints.uwe.ac.uk/11735/2/thematic_analysis_revised...). Does anyone have any other suggestions? Thanks in advance, IBE (talk) 06:51, 19 October 2012 (UTC)[reply]

I'm thinking about it. In the meantime, if you are doing qualitative research, I expect that you'll need to have some kinds of interviews or conversations. If you recruit your participants as you go along, with no kind of sampling frame, that's called "snowballing". Are you doingparticipant observation - that's a method not a paradigm. Sounds like you're doing social anthropology rather than sociology or any other kind of social research. There's participatory action research, but that would give you even less control of the process and require pre-planning.Itsmejudith (talk) 07:02, 19 October 2012 (UTC)[reply]
Not quite that in-depth, just evaluating some educational software I'm producing, and it may involve qualitative assessment as well as quantitative. The quantitative bit will be the main bit, but it isn't likely that there will be unequivocally positive results. I strongly suspect the value of such things depends on the person using them, so I'm expecting in advance that results will be somewhat mixed. Hence the need for a fairly blank slate. Also, since the qualitative research is likely to be just a side component, for framing the quantitative research, I'm trying to escape from any massively involved, formulaic approaches. Participatory action research scares the living heck out of me, if you are wondering. IBE (talk) 07:18, 19 October 2012 (UTC)[reply]
Of course, you're the the person doing the evaluation. The good news is that you're using mixed-methods research, which is very good practice. You can operate some kind of triangulation between your quantitative and qualitative findings. You can do it either way round: the qualitative element is your homework to help you make sense of the quantitative findings, or the quantitative element is your homework to help you make sense of the qualitative findings. Participatory action research is full-on, but you don't have to adopt the whole thing. Letting some research users comment on the process might help both you and them. Itsmejudith (talk) 09:09, 19 October 2012 (UTC)[reply]
Very funny. Looie496 (talk) 14:32, 19 October 2012 (UTC)[reply]
Is it? I think he's just doing a small evaluation and wants to keep it simple. A questionnaire backed up by some interviews. Nothing wrong with that so long as he doesn't extrapolate too far. Itsmejudith (talk) 16:15, 19 October 2012 (UTC)[reply]
I think he's just having a go at my last comment. This may be relatively mild for Looie. IBE (talk) 05:27, 22 October 2012 (UTC)[reply]

I think the dumbest methodology ever is the Strong programme, which basically involves writing the history of science from the point of view that there is no truth or objective reality, so that all changes in accepted scientific explanations (e.g. phlogiston vs. heat, luminiferous ether vs. no ether etc. etc.) must be regarded as purely due to fads or social influences. However, it doesn't appear to have much to do with your question...AnonMoos (talk) 16:18, 19 October 2012 (UTC)[reply]

The world exists independently of our knowledge of it. Therefore there are research questions about the knowledge-gathering process that are independent of the truth of the knowledge acquired. Little to do with the OP's question. Itsmejudith (talk) 22:24, 19 October 2012 (UTC)[reply]
I think that some of the Strong Programmatists tend to believe that everything is a social construct, and that it's possible to write an adequate history of science without paying attention at all to whether some theories might be closer descriptions to physical reality (assuming physical reality even exists) than other theories are. Or at least that's the impression of themselves that the Programmatists tend to create among actual scientists... AnonMoos (talk) 23:50, 19 October 2012 (UTC)[reply]
Yes, they do give that impression, deliberately I reckon. Nevertheless, sociologists may point out that it is possible to do soc of religion without reference to the truthfulness of the religions studied. You can even say a lot about soc of education without commenting on whether the curriculum is based on correct science. Soc of the film industry without taking an interest in the quality of the movies. Itsmejudith (talk) 22:22, 20 October 2012 (UTC)[reply]

has an amateur ever won the nobel prize?

by our account, Paul Gauguin was a stock broker until his late thirties, when he started painting after hanging out by painters. I wonder if anyone started in the sciences as an amateur (e.g. in their late twenties, in their thirties, fifties, sixties, whatever) and went on to win a nobel prize?

I am interested primarily in Physics, Chemistry, or Medicine. --91.120.48.242 (talk) 07:25, 19 October 2012 (UTC)[reply]

Gentleman scientist is probably the best starting point. It mentions Peter D. Mitchell as a modern-day independant scientist that has won a Nobel prize. --Tango (talk) 12:10, 19 October 2012 (UTC)[reply]
Marconi might fit your definition. 86.166.191.232 (talk) 17:32, 19 October 2012 (UTC)[reply]

Gore vs. Bush: A Political Fable by Jeff Greenfield Summary

Can someone please give me a good summary of all of the major events in this book? Thank you very much. Futurist110 (talk) 08:32, 19 October 2012 (UTC)[reply]

The reason that I'm asking is because I couldn't find a good summary of all the major events in this book anywhere online.Futurist110 (talk) 19:43, 19 October 2012 (UTC)[reply]
Why not just read the book? Blueboar (talk) 22:44, 19 October 2012 (UTC)[reply]
I tried finding this book at my local library and couldn't, but I want to get a grasp of what's in the book before I read all of the details.Futurist110 (talk) 23:53, 19 October 2012 (UTC)[reply]

Question about World War II

Why did so many civilians on the Nazi side commit suicide as the war approached to an end? Iowafromiowa (talk) 10:06, 19 October 2012 (UTC)[reply]

I'm curious - are there statistics to show that this is true? I can think of several possible explanations, but I'm loath to speculate without data.AlexTiefling (talk) 10:08, 19 October 2012 (UTC)[reply]
Yes, in fact, I can't remember now, but there were cases of entire towns committing suicide. Found it, mass suicide in DemminIowafromiowa (talk) 10:09, 19 October 2012 (UTC)[reply]
Nazi propaganda had done a good job of telling everyone how barbarous the invading Soviets were going to be, with good cause. The war on the Eastern Front was one of total war, where wholesale atrocities against civilians by both sides were commonplace. See Soviet war crimes#World War II andRape during the occupation of Germany#Soviet army and Flight and expulsion of Germans (1944–1950). Alansplodge (talk) 10:28, 19 October 2012 (UTC)[reply]
Legends of Red Army atrocities would have been my first guess. The people of Pomerania might have heard first-hand accounts of the fall of East Prussia to the Red Army, too. (I watched a German documentary about this a couple of years ago. The Nazi leadership in the area mostly got on the last plane out, leaving the civilians to face the Soviets alone.) AlexTiefling (talk) 10:33, 19 October 2012 (UTC)[reply]
I can't cite specific sources, but another possible explanation is that after years of being told that it's the superhumans' destiny to rule and dominate, and the subhumans' to serve or be exterminated, it became obvious to many Germans in 1945 that if they were losing the war, then it was them who turned out to be the inferior race after all. Imagine that you think in these categories and you come to reach such a conclusion about yourself; there isn't much incentive to keep on living, is there? — Kpalion(talk) 14:39, 19 October 2012 (UTC)[reply]
Maybe among senior Nazis, but for ordinary folk, I think unbearable fear of the immediate future would have been a more important factor. My understanding is that these mass suicides were a feature of the Eastern Front; those in the west expected rather more humane treatment from the Western Allies. Alansplodge (talk) 15:40, 19 October 2012 (UTC)[reply]
I've just found Mass suicides in 1945 Nazi Germany. Alansplodge (talk) 15:48, 19 October 2012 (UTC)[reply]
It looks like the suicides were heavily weighted towards high Nazi officials and their families. As for civilians, I don't know why they didn't head West in the hope of making it to parts of Germany to be occupied by the West. Of course, there's a good chance they could be killed along the way, but that seems a lot better than certain death, to me (keep the cyanide capsule handy, in case things get bad on the way west).StuRat (talk) 23:26, 19 October 2012 (UTC)[reply]
I think a lot did, but look at this article talking about the Soviet conquest of Berlin, which says; "Estimates of rape victims from the city's two main hospitals ranged from 95,000 to 130,000. One doctor deduced that out of approximately 100,000 women raped in the city, some 10,000 died as a result, mostly from suicide." Alansplodge (talk) 23:46, 19 October 2012 (UTC)[reply]
Ian Kershaw's excellent new(ish) book about the last months of Nazi Germany, The End, discusses this. Aside from the fear of the Red Army, lots of people with prominent Nazi links or who had been involved in war crimes (or both) committed suicide as they expected to be prosecuted and either jailed or executed at the end of the war. That said, and from memory, Kershaw considers the emphasis which has been placed on the number of suicides to be somewhat overblown - he highlights the huge number of murders and semi or extra-judicial killings of people who wanted to end the war or surrender which were committed by the Nazi government in the war's last months. Anyway, I strongly recommend Kershaw's book for a rounded discussion of this period. Nick-D (talk) 22:52, 25 October 2012 (UTC)[reply]

Titanic question

I've read that many passengers later said that the Captain may have committed suicide, or other officers may have. What's the most accurate report on that?, who is most likely to have committed suicide? Thank you. Iowafromiowa (talk) 10:20, 19 October 2012 (UTC)[reply]

According to his article, Smith was reported by a couple of witnesses as jumping into the water as his ship finally sank, but that could hardly be called suicide. It also goes on to say that most likely he simply remained at his post til the end. I find it implausible that he (or any other officer) would go to the trouble of finding some means to end his life when the cold would do it for him soon enough. This Daniel Allen Butler article (despite being titled "Captain Smith committed suicide") says that his fate is a "mystery", but that he likely didn't commit suicide. Encyclopedia Titanica reports only rumors that one of the officers shot himself, either First Officer William McMaster Murdoch (his article says a witness saw his dead body with what appeared to be a head injury, leading to speculation of a gunshot wound), Smith or others.[7]Clarityfiend (talk) 11:00, 19 October 2012 (UTC)[reply]
We could not say that anyone who died as a direct result of the ship's sinking committed suicide. Men who had the chance to get away in a boat but opted to allow a woman or child take their place - that is not suicide. Officers who went down with the ship rather than abandoning it - that is not suicide. People who jumped overboard and died in the fall - that is not suicide, unless we somehow knew they intended to kill themselves by jumping, rather than save themselves by jumping. If someone slashed their wrists or shot themselves rather than facing the icy waters - that would be suicide, but we have no evidence of the captain doing any such thing. -- Jack of Oz [Talk] 21:14, 19 October 2012 (UTC)[reply]
We have an article; Captain goes down with the ship. My grandfather, who earned his Master's ticket before the Titanic sank, used to say that the tradition was because even if not to blame, the captain would be unemployable thereafter. There seem to be only three examples quoted in our article though, and one of those is the captain of the Titanic. Alansplodge (talk) 23:34, 19 October 2012 (UTC)[reply]
As our article says, the principle is more about being the last to leave than actually going down with the ship. In the case of the Titanic, it wasn't possible for everyone to leave, so the principle mandated that the captain be one of those that stayed on board. In most cases (especially with modern laws about carrying enough lifeboats), everyone does leave, with the captain simply being the last. In all three examples given in our article, there were other people on board as well when the ships sank. Would a captain by employable if they lost their ship but managed to save all the people on board? (Or, at least, those not killed in whatever happened to the ship to make it sink.) There is a common naval practice of putting the captain on court-martial whenever a ship is lost, but if they were acquitted of any wrongdoing would they still be unable to work as a captain again?--Tango (talk) 14:40, 20 October 2012 (UTC)[reply]
The merchant service is rather different to the armed forces. I think his point was that an employer wouldn't want someone associated with a sinking whether at fault or not. There were always more masters than ships, so they could afford to be choosy. However, he died 30 years ago, so you'll have to take what he said at face value. I can assure you that those were his words, but can't provide further detail. It may have been a myth among merchant officers or maybe not. Alansplodge (talk) 14:56, 20 October 2012 (UTC)[reply]

Could we say that there's a persecution of homosexuals in the Middle East?

Thank you. Iowafromiowa (talk) 10:43, 19 October 2012 (UTC)[reply]

Perhaps, but I think that's an unhelpful generalisation. The specific problems faced by LGBT people in Middle-Eastern countries vary widely. It might be worth looking at things like the Turkish army's ban/exemption for gay men (it's not enough to fancy men - they want to see a picture of you having sex; but as there's national service, getting out of the military may be seen as desirable) or Iran's position on transgender issues (they're getting very good at gender transitions, because the religious leadership regard it as a cure for sinful same-sex attraction). The Middle East is a huge area, and very varied. There's absolutely no unifying position.
And I don't mean to bait, but here in the UK we have a fairly comprehensive set of nationwide anti-discrimination laws, civil partnerships, and moves towards equal marriage on the way. Could I argue that because the US has utterly failed to do any of these things at the federal level, and has actively interfered with some of them, there's anti-LGBT persecution in the USA? The threads further back about being able to come out at work show that this has a more than incidental impact on the lives of LGBT people. AlexTiefling (talk) 10:52, 19 October 2012 (UTC)[reply]
I think this is misleading and unnecessarily "politically correct". There's a unifying position in that, at a minimum, homosexuality is generally illegal in Muslim countries. (Homosexuality#Middle East) Wongot (talk) 15:30, 19 October 2012 (UTC)[reply]
So is Turkey not a Muslim country, by that yardstick? And did you choose to ignore my response below? Mr.98's answer is exemplary; your 'more specific' response is actually less specific than his. "Politically correct" as a criticism is useless; it's just a 'boo-word', telling me nothing more than that you don't like what I've said. Say what you really think, rather than parroting a cliché. AlexTiefling (talk) 15:46, 19 October 2012 (UTC)[reply]
Turkey is really not a Muslim country. I mean, demographically, of course, it is. But in terms of the apparatus of state, it is not. In fact there's a strain of official anti-clericalism that, in many Western countries, would be considered to infringe on the religious liberty of Muslims.--Trovatore (talk) 04:30, 20 October 2012 (UTC)[reply]
Oh, I'm aware of that. There are some interesting parallels with France, for example. But Wongot seems to be accusing me of 'political correctness' precisely because I don't want to give a blanket answer that assumes the Middle East consists entirely of autocratic constitutionally-Muslim states. I know things are tough for LGBT people across much of the region. But I still think a more nuanced answer is better than a generalising one. Mr.98's response strikes the balance better than my own, I think. AlexTiefling (talk) 07:29, 20 October 2012 (UTC)[reply]
I think one could say, as a generalization, that most countries in the Middle East persecute homosexuals. (We have a large article that outlines all of this.) There are several notable exceptions, though. Israel is the most tolerant by far, though even they do not allow homosexuals to be married in the country (though they do recognize foreign same-sex marriages). The other relatively tolerant countries are Jordan, Iraq, Turkey, and the Palestinian West Bank — though "relatively tolerant" just means that it isn't illegal, not that it is protected or given equal rights. In all other countries in the region, homosexual activity is illegal and comes with fairly terrible consequences. I would call that unambiguous persecution. I agree with AlexTiefling that there certainly is a spectrum of tolerance, but this is clearly on the side of "persecution," and I don't think it's an unhelpful generalization to say that all countries in the region except five persecute homosexuals, and that even among those five, at least four of them are probably pretty socially intolerant of homosexuality even if they don't have laws outlawing it explicitly. --Mr.98 (talk) 14:11, 19 October 2012 (UTC)[reply]
More specifically, see Homosexuality#Middle East and the articles linked to in that article section. Wongot (talk) 15:26, 19 October 2012 (UTC)[reply]
Hmm, the article I linked to goes into far more detail. That particular article has only one paragraph on modern practices, apparently doesn't consider Israel as part of the Middle East, and gives only one specific example (Iran) which is more about what someone said than what the practices were. --Mr.98 (talk) 15:34, 19 October 2012 (UTC)[reply]
I absolutely agree! I wasn't attempting to claim otherwise - only to sound a necessary note of caution. AlexTiefling (talk) 14:21, 19 October 2012 (UTC)[reply]
Well, according to this map it's located at the center of the world.Blueboar (talk) 22:59, 19 October 2012 (UTC)[reply]
If X is a place in the Middle East and there's a persecution of homosexuals in X, then we can say that there's a persecution of homosexuals in the Middle East. —Tamfang (talk) 08:16, 20 October 2012 (UTC)[reply]
And wise and enlightened opinions, too! Not to mention hot babes in string tangas, and amazing Prosciutto di Parma. --Stephan Schulz (talk) 09:55, 20 October 2012 (UTC)[reply]
I can't speak for everybody else, but I can say that. I can also say hurnfidqpodxys. Plasmic Physics (talk) 12:26, 20 October 2012 (UTC)[reply]
So, given that there's persecution of homosexuals in the universe, Tamfang, can we say there's universal persecution of homosexuals?μηδείς (talk) 02:35, 21 October 2012 (UTC)[reply]
"in the universe" is not equivalent to "universal". There is capital punishment in the United States, but not "universally" in the United States, but only in some states. ←Baseball Bugs What's up, Doc?carrots13:08, 21 October 2012 (UTC)[reply]
Thank you for playing. —Tamfang (talk) 22:11, 21 October 2012 (UTC)[reply]

September 11 attacks

Can you bring a boxcutter aboard now? I don't travel much. Thank you. Iowafromiowa (talk) 10:48, 19 October 2012 (UTC)[reply]

Not in your carry-on luggage; they will be (ideally) spotted on the X-ray machines or metal detectors and you won't be allowed to bring them through security. You can check them in your on-board luggage, though. --Mr.98 (talk) 11:23, 19 October 2012 (UTC)[reply]

FDR

Can you publish pictures of FDR? — Preceding unsigned comment added by76.179.169.163 (talk) 13:19, 19 October 2012 (UTC)[reply]

What do you mean by FDR? - Lindert (talk) 13:32, 19 October 2012 (UTC)[reply]
Also, what do you mean by "can you publish"? If you are looking for photos of Franklin Delano Roosevelt, visit that article, or just go to an image search engine like images.google.com and search for FDR. Wongot (talk) 15:24, 19 October 2012 (UTC)[reply]
Our cache of FDR images is here. --Tagishsimon(talk) 15:32, 19 October 2012 (UTC)[reply]
You might also be interested in Wikipedia:Image use policy. Taknaran (talk) 17:13, 19 October 2012 (UTC)[reply]

Golden Dawn (Greece), nationalism and Germany

Being Greek and nationalistic, how can a party use German Nazi symbols? Greece was obviously a victim of Nazi Germany, or do some Greeks dispute that? Can someone explain this contradictory association of nationalists using foreign symbols? OsmanRF34 (talk) 15:03, 19 October 2012 (UTC)[reply]

It's important to note that a lot of the symbolism used were not intrinsically German. There's a wilful antiquarianism about a lot of fascist thought, especially at the more mystical end. The Nazis appropriated Indian symbols for their own use - symbols which have existing close parallels in classical Greek and Roman art. If you go to the Roman-German Museum in Cologne, you'll see in the basement there the Hakenkreuzmosaik, or Swastika-Mosaic. This is a genuine Roman artifact, and emblematic of how widespread a symbol it was in the Classical world. Similarly, there are Greek key variants in which there are two lines, not one, and each time they cross, they form a swastika (or its mirror-image).
Being proponents of an ancient and distinctive Indo-European nation, the Golden Dawn are drawing on similar ideas to those invoked by the Nazis through the work of the Ahnenerbe and the sentimental antiquarianism surrounding locations like the Externsteine. (Persian/Iranian nationalism has had a similar theme from time to time, too - bear in mind that 'Iran' and 'Aryan' have related etymology.) Recent history has little to do with it. In 1940, when northern France, including Paris and Reims, were under German occupation, the collaborationist government of southern France, based in Vichy, put up adverts telling the populace "You are not betrayed! You are not abandoned!". The obvious falsity of the claim was secondary to the unifying tone it brought.
Besides all which, aside from being Greek instead of German, the Golden Dawn are very much neo-Nazis. Indeed, they're arguably the most popular and successful genuinely neo-Nazi political party in Europe since 1945. They're a lot more Nazi in theme than (say) the British National Party, who are much more like the Portugese Corporatists. Fascism and its related ideologies come in many forms, but both the beliefs and the tactics of the Golden Dawn very closely resemble those of the NSDAP. So it makes sense that their iconography is so reminiscent as well.
It is, of course, ironic that Greek protesters against Angela Merkel's recent visit trotted out the WW2-era Nazi flag to mock her - deliberately recalling Germany's occupation of Greece - when something recognisably like Nazism is growing so much closer to home.AlexTiefling (talk) 15:25, 19 October 2012 (UTC)[reply]
Isn't Jobbik in Hungary also neo-Nazi and more successful than the Golden Dawn? Futurist110 (talk) 23:48, 19 October 2012 (UTC)[reply]
Certainly comparable, at the least. You're right; Golden Dawn's success is not unique - they just happen to be in the news more at present.AlexTiefling (talk) 07:41, 20 October 2012 (UTC)[reply]

OsmanRF34 -- I've wondered about that too: Fringe parties in countries which were victims of Nazi invasions using flags with an obvious Nazi-influenced structure (i.e. black emblem on white area on an overall red field). The most conspicuous example is the National Bolshevik Party... --AnonMoos (talk) 16:06, 19 October 2012 (UTC)[reply]

Such fringe parties want principally to shock, don't expect to find a well-developed and non-contradictory ideology behind them. That's also why there were cases of Neonazis Jews (with German symbolism) in Israel. Gorgeop (talk) 23:15, 19 October 2012 (UTC)[reply]

I'm not an expert on Greece or Greek politics, but I was under the impression that Greek far-right nationalists use German Nazi symbols to represent Greek racial and ethnic supremacy, rather than "Aryan"/Anglo-Saxon racial and ethnic supremacy. This is similar to how some whites in the Southern United States use the Confederate flag right now to represent their culture rather than racism and slavery. Futurist110(talk) 23:46, 19 October 2012 (UTC)[reply]

Think you mean "Aryan"/Nordic racial supremacy. Anyway, when people in or from the U.S. south wave confederate flags, they're recalling something based on their own region or ancestry (regardless of the dubious politics involved, or the fact that most of them display the confederate naval jack instead of the actual confederate battle flag). However, when Greeks wave a Nazi-structured black-on-white-on-red flag, they seem to be making a positive allusion to outside hostile conquerors of Greece... AnonMoos (talk) 12:12, 20 October 2012 (UTC)[reply]
Nordic and Anglo-Saxon are almost the same thing, so the terminology doesn't matter too much. That said, I'm not an expert on Greece, but it would appear to be that Greek neo-Nazis could simply be using the Nazi flag to praise Nazi ethnic/racial policies (except replacing Nordics with Greeks at the top of the hierarchy), rather than praising the Nazi occupation of Greece (with the exception of the Holocaust in Greece, which they might definitely be praising). Futurist110 (talk) 22:17, 20 October 2012 (UTC)[reply]
They're not really "almost the same thing". Anglo-Saxon supremacy means exalting Englishness in various senses, while Nordicism means a cult of blue-eyed blondness and/or appropriating elements from the Norse sagas (or at least the Wagner opera version of Norse sagas) or from runes (or at least the Guido von List version of runes), etc. If Golden Dawn had a blue and white flag, then it would not raise much of an eyebrow, but when they adopt a black-on-white-on-red flag it gives rise to suspicions that they're exalting the Nazi conquerors and oppressors of Greece...AnonMoos (talk) 06:29, 21 October 2012 (UTC)[reply]
Meandros flag
I'm not sure what 'symbols' you are referring to. If it is the Meandros flag (pictured right), although it bears some resemblance to the German swastika (not least in the choice of colour scheme), that's not what it is. It's a Greek symbol. V85 (talk) 18:22, 20 October 2012 (UTC)[reply]
Of course, any resemblance is purely coincidental ;-) Alansplodge (talk) 19:25, 20 October 2012 (UTC)[reply]
This is your swastika: This is your swastika on qualudes:
Baseball Bugs What's up, Doc? carrots20:50, 20 October 2012 (UTC)[reply]
2... 4... 3: -- OBSIDIANSOUL 02:40, 23 October 2012 (UTC)[reply]

Why do older women choose short haircuts?

Why - in the US, anyway - do almost all older women choose short haircuts? Wongot (talk) 15:21, 19 October 2012 (UTC)[reply]

Mothers seem to prefer them. They are certainly easier to maintain, and probably it doesn't matter to look good when you are old.OsmanRF34 (talk) 15:34, 19 October 2012 (UTC)[reply]
(sigh) Please remove your ageist remark, as an older woman I find it offensive. Thank you. --TammyMoet (talk) 15:36, 19 October 2012 (UTC)[reply]
It's not ageism. Appearance matters less when you are getting older. Even if you compare a teenager with a young adult, you'll discover that the latter cares less about other non-image related aspects about himself and about his partners. You can construe my comments, assuming good faith, from a positive point of view. Although you can also interpret it with an unintended meaning that nothing matters anymore when you are old.OsmanRF34 (talk) 15:45, 19 October 2012 (UTC)[reply]
One day, young man, hopefully you will be old yourself. Then you can come back to me and tell me that I'm wrong to find your remarks offensive. Until then I will reiterate that your remarks are offensive to me. Not only that, but you are wrong. It is much more important to keep up appearances once you reach a certain age. Oh and you can also tell me how getting your hair cut every other month (at greater expense for women than men) is easier to maintain than just not having a hair cut at all. --TammyMoet (talk) 18:21, 19 October 2012 (UTC)[reply]
If he keeps putting down women, old age might not be in the picture. ←Baseball Bugs What's up, Doc? carrots20:48, 19 October 2012 (UTC)[reply]

I think there's a natural effect among both men and women that once they've attracted a long-term mate (or have given up the search), they can relax a little from previous strenuous efforts to be attractive (e.g. a man not going to the gym so many times a week, etc.). AnonMoos(talk) 15:54, 19 October 2012 (UTC)[reply]

Damn! Just spent a lot of money on a very nice (shortish) haircut. Apparently I didn't need to. Itsmejudith (talk) 16:11, 19 October 2012 (UTC)[reply]
To the OP: (1) Personal preference: [8][9]
(2) Cultural expectation: [10][11]
(3) Physical difficulty caring for long hair: [12][13]
And surely for many more reasons, as there are individuals. Taknaran (talk) 16:51, 19 October 2012 (UTC)[reply]
I think you've hit the main reasons in your post. I do want to point out a current counterexample to this trend and about its ability to change older women's hairstyles--Hillary Clinton now has very long hair, and she will turn 65 this year. Futurist110 (talk) 20:40, 19 October 2012 (UTC)[reply]

I don't know why it's so, but it's long been the tradition for older women to at least wear their hair up even if the hair itself is long. Up until the 1960s or 1970s at least, it would have been quite unusual for a woman (in the U.S. at least) over 40 to wear her hair down flowing over her shoulders and back. Even if she had it long enough that it could, she would wear it up somehow, e.g. in a bun. Angr(talk) 20:28, 19 October 2012 (UTC)[reply]


Kim Novak
Well, I am in my fifth decade, and I still wear my hair long. I am curious, Tammy, how you decide which hair not to have cut. :)μηδείς (talk) 20:40, 19 October 2012 (UTC)[reply]
As someone in her sixth decade, let me enlighten you that once you get past the menopause there is no need to have pubic hair cut or shaved, not is there any real reason to pluck eyebrows. Underarm hair still needs some topiary though. As an aside, I had my hair so long I could sit on it until I turned 40, then my new man said he preferred it short, that I looked better. Someone sent me a photo I had taken with very long hair: I have to say he was so right! --TammyMoet (talk) 08:52, 20 October 2012 (UTC)[reply]
I wonder if [14] is relevant (sebum decreases somewhat with age). Wnt (talk) 20:40, 19 October 2012 (UTC)[reply]
Long hair is a lot of work to maintain, and is done so by young women, mainly to attract men. Since older women are often either married or not looking for a man, that makes it extra work for no benefit. I like to refer to older women with white hair all pushed up on top as "Q-tips". :-) StuRat (talk) 20:47, 19 October 2012 (UTC)[reply]
Miss Novak in 1962
I am not sure where you get the idea that maintaining long hair is harder than having it cut and styled regularly. My sister and I both wear ours long. My mother looks like Kim Novak, however, so she keeps hers short, never below the collar. μηδείς (talk) 20:54, 19 October 2012 (UTC)[reply]
Also in my fifth decade (and nearly out of it). Occasionally wonder whether or not to cut my long hair, but it has never suited me short and I can't be bothered with the hassle of maintaining a short precision cut like my teenage daughter's, which requires far more maintenance than mine. If it gets to the point where I feel like mutton dressed as lamb I'll probably take it to shoulder-length, but while I can continue to carry it off, it stays. - Karenjc 21:01, 19 October 2012 (UTC)[reply]
Exactly. I just called my mom and asked. She says she wears it "short" by which she means something like Novak in the '62 picture. Half of the time I spend talking to her is about her hair, her last haircut, her last die job, and who's cutting her hair now since she severed ties with the last stylist.
My late youngest sister wore hers long until she joined the crew team in college, when she cut it medium (bobbed below the ear) but kept a Chinaman's queue. I cut my or have my own ends cut maybe twice a year--I did so myself just last week. But I haven't gone to a shop since the 80's. BTW, I think Hillary Clinton looks sooo much better that she's let her hair grow out. μηδείς (talk) 23:16, 19 October 2012 (UTC)[reply]
It's amazing to see Medeis exchanging information about her hair after all her complaints about the decay of the RD. OsmanRF34(talk) 23:32, 19 October 2012 (UTC)[reply]
Feel free to hat whatever thread you like Osman. Given this one hasn't been hatted, pardon me for telling you to bug off. μηδείς(talk) 02:27, 21 October 2012 (UTC)[reply]
I'm a young man and had long hair (well below my shoulders) while at Uni and now have very short hair. I can confirm that long hair is generally less work. I got it cut at most every few months (and that was just while I was growing out my fringe - once it had all got to the same length, I could go more than 6 months without getting it cut). In the morning, I would just put a brush through it and then tie it in a ponytail and it was done. Now, I have to put wax in it to get it to what I want, which makes washing it much more work. I have to get it cut at least once a month or it starts to look unkempt. The only thing that is easier about it is drying it after washing it - I can now just towel it dry in about a minute, or leave it to dry on its own and it's fine. I used to have to be very careful to avoid it going frizzy. So, unless you just have a number two all over that you can do yourself in a couple of minutes and not leave enough hair to need any maintenance, then I don't think short hair is more convenient than long hair.--Tango (talk) 13:16, 20 October 2012 (UTC)[reply]
I'd also point out that, with hormonal changes, one's hair grows thinner as one ages. This applies to women as well as men. Wearing short hair tends to maximise the look of the hair in these circumstances. You only have to look at some of the old rock stars to see how awful thinning long hair can look. --TammyMoet (talk) 08:55, 20 October 2012 (UTC)[reply]
Good point, see Combover. Differences between male pattern baldness and female hair loss may account for styling choices among older women who find a short, full cut, permed or blow-dried for volume, makes the most of their crowning glory if it has been affected by the ageing process. - Karenjc 12:10, 20 October 2012 (UTC)[reply]

One very cogent point that hasn't been made is that as one gets older, one's face tends to sag (unless the old plastic surgeon gets stuck in!). Long hair can visually "drag down" the face, so many older ladies choose shorter hair as that makes the face look more lifted. It's an optical illusion, but it's how human perception works. Quintessential British Gentleman (talk) 01:51, 21 October 2012 (UTC)[reply]

Why do lawyers now advertise, when in previous times they did not?

Forty years ago or so in the US, it was verboten for lawyers to advertise. They just did not do it. Now, I seem to see lawyer advertisements, from the Yellow Pages to the television. Why the change? Wongot (talk) 15:23, 19 October 2012 (UTC)[reply]

On a practical level, I suspect because their professional association deicded to allow them to when previously it had forbidden it. (That, at least, is my remembrance of the UK situation.) As to why they took that decision, we could only speculate. Demand, presumably.--Tagishsimon (talk) 15:26, 19 October 2012 (UTC)[reply]
Please supply references in your answers, this is a reference desk. Wongot (talk) 15:34, 19 October 2012 (UTC)[reply]
See Legal advertising in the United States. Basically in 1977 the Supreme Court threw out a state law banning legal advertising; state bars can now regulate advertising but not prohibit it. --Mr.98 (talk) 15:29, 19 October 2012 (UTC)[reply]
Perfect and thank you. Wongot (talk) 15:33, 19 October 2012 (UTC)[reply]
In many states before that time, the only advertisements that law firms were allowed to run were "tombstone" advertisements -- basically the unadorned name and location of the firm in an enclosing rectangle, where it was not even allowed to mention that it was a law firm which was being advertised. Sometimes when I see the ambulance-chasing adds on early-morning TV, I feel a little nostalgic for those days... AnonMoos (talk) 15:50, 19 October 2012 (UTC)[reply]
Here's something that the article Legal advertising in the United States only hints at, but I remember as being assumed by economists at the time to be an important, maybe the most important, consideration on the part of the state bar associations in their opposition to advertising (sorry I don't have any citations on this): advertising allows lawyers to advertise prices that undercut the prices of other lawyers, and so those other lawyers end up not being able to charge as much as they otherwise would. Same with doctors, as I recall. Of course they tried to disguise this implicit price-fixing as "professional ethics". Duoduoduo (talk) 17:51, 19 October 2012 (UTC)[reply]
I rarely see an advertisement for a lawyer mention prices. The closest I've seen are those which say they work on a contingency basis (only pay them if you win your lawsuit) and perhaps some fixed price work ("A no-fault divorce, with no children or property settlement, for only $200").StuRat (talk) 18:22, 19 October 2012 (UTC)[reply]
We have an article Tombstone (advertising), but it's not much... AnonMoos (talk) 21:50, 19 October 2012 (UTC)[reply]
It isn't that lawyers did not advertise in the past, more that the ads were limited in their content. EvenAbraham Lincoln advertised.    → Michael J    18:17, 19 October 2012 (UTC)[reply]

Are the remains of the Kings of Norway still under the site of the ruins of Christ Church, Bergen or were they buried above ground and destroyed when the church was demolished? Actually how are European monarch normally buried? Do the tomb effigies in European Churches that look like stone sarcophagus contain the bodies of dead kings and queens? Doesn't Christians have something about burying the dead in the ground? -The Emperor's New Spy (talk) 15:39, 19 October 2012 (UTC)[reply]

Sometimes they do, sometimes not. The monuments in St Denis Basilica don't contain any bones because all the bones were dug up and reburied in a mass grave during the French Revolution (same with the ones in Fontevraud Abbey for example. But the bones certainly used to be there. On the other hand there are actual burials in Westminster Abbey. Adam Bishop (talk) 00:32, 20 October 2012 (UTC)[reply]
This might also interest you. It is a blog covering the recent exhumation of the (purported) remains of Magnus Ladulås in Riddarholmskyrkan. The blog itself might not be very interesting unless you speak Swedish, but there are some pictures at least! By the way, there are lots of Christians whose remains are not "buried" under ground, see the "Incorruptibility" for some examples.Gabbe (talk) 18:41, 20 October 2012 (UTC)[reply]

Award-winning crime novelist David Housewright

I've been told a page concerning the Edgar Award-winning mystery writer David Housewright was submitted Sept. 13, 2012. It is not up. Is this the result of a backlog of pages waiting to be reviewed, or had the page been rejected for some reason? — Precedingunsigned comment added by David Housewright (talkcontribs) 16:00, 19 October 2012 (UTC)[reply]

That would be at David Housewright. As I right in assuming that you are the 'Award-winning mystery writer' himself? If not, you should familiarise yourself with the Wikipedia:Username policy - it is not permitted to impersonate others. If you are David Housewright, I suggest you readWikipedia:Autobiography - and also Wikipedia:Identifying reliable sources policy - we need better sources than your own webpage for many of the statements made. It might be worth looking at the Wikipedia:Notability (people) guidelines too. AndyTheGrump (talk) 16:11, 19 October 2012 (UTC)[reply]
And if you are David Housewright, you should read Wikipedia:Conflict of interest and Conflict of interest editing on Wikipedia before changing a single comma of the article David Housewright. -- Jack of Oz [Talk] 20:58, 19 October 2012 (UTC)[reply]

Econ. Nobels

Is there any place (preferable free) to get thedissertations of the nobel laureates>?Lihaas (talk) 17:42, 19 October 2012 (UTC)[reply]

I imagine you're unlikely to find them all in one place, if that's what you mean. —Tamfang (talk) 08:34, 20 October 2012 (UTC)[reply]
What dissertations do you mean? Their doctoral dissertations? You'll find them in the university library the doctorate was awarded by. Or do you mean the papers that they were awarded the Nobel prize for? You can find back issues of major journals in any good university library - the details of what papers they wrote were probably given in the formal announcement of the prize. --Tango (talk) 15:12, 20 October 2012 (UTC)[reply]
I expect he means the address the winners give at the prize ceremonies. And I don't mean 378 Springfield Drive, Charmville, New Hamphsire. -- Jack of Oz [Talk] 18:38, 20 October 2012 (UTC)[reply]
There is also the Nobel lecture, which I believe is separate from the acceptance speech. He could mean that. Since those are both spoken, rather than written, I wouldn't call them a dissertation. --Tango (talk) 17:25, 21 October 2012 (UTC)[reply]
Videos & pdfs for recent econ Nobel lectures are here. You can find the acceptance speeches (all, I think) and other material and Nobel lecture pdfs (for some) from following the links for each winner fromhere.John Z (talk) 01:15, 22 October 2012 (UTC)[reply]

Some voices

Hi I see that some voices in wikipedia (Matteo Goffriller) are reporting information ( before unedited) drawn from my book witoit reporting reference. Just an example All the historical data on Matteo Goffriller are drawn from my book titled Violin and Lute Makers of Venice 1640 -1760 by Stefano Pio (it is me) edited by Venice research, venice, Italy 2004. It would be appreciated if you mention the source Thank you Stefano Pio — Preceding unsigned comment added by151.95.60.179 (talk) 18:31, 19 October 2012 (UTC)[reply]

Yes, our editors should indeed cite their sources... but, how do you know that the information in the article came from your book and not one of the other cited sources? Blueboar (talk) 22:27, 19 October 2012 (UTC)[reply]

Hi - it may be worth mentioning this on the talkpage for this article (Talk:Matteo Goffriller). I'm not sure whether its frowned on or not but technically you can edit the article yourself to include the references (I know there's something about using your own work/Wikipedia liking third party references but I can't find the page right now). ny156uk (talk) 22:28, 19 October 2012 (UTC)[reply]

There is no good or evil, except in your eyes or mind

I'm having some difficulties in wrapping my mind around such way of thinking. What about the more extreme cases of psychopaths, rapists or murderers? Is evil just in my eyes or mind? I'd be OK if it were more restricted to harmless cases. Gorgeop (talk) 23:20, 19 October 2012 (UTC)[reply]

That's moral relativism. "Absolute belief in moral relativism is absolutely rare". Nearly everyone finds some behaviors to be universally evil. Although, in the case of animals, moral relativism is far more common. That is, we think that our cat torturing a mouse before killing it is fine, because "that's just what cats do". StuRat (talk) 23:29, 19 October 2012 (UTC)[reply]
I agree with what StuRat wrote but also want to point about that people's morality can change over time. Also, I don't consider some things (like polygamy and incest) to be morally unjustifiable, in contrast to most people. Futurist110 (talk) 23:38, 19 October 2012 (UTC)[reply]
It's absolutely evil what StuRat does with apostrophes. "Cat's"? Clarityfiend (talk) 04:08, 20 October 2012 (UTC)[reply]
Fixed. StuRat (talk) 07:11, 22 October 2012 (UTC) [reply]
Or chaos theory, you just cannot calculate what act produces what outcome. (In this interpretation, it is also a moral relativism).OsmanRF34 (talk) 23:37, 19 October 2012 (UTC)[reply]
After a long period of time, perhaps, but, in the short run, you can be reasonably certain that pushing somebody off a cliff won't do them much good.StuRat (talk) 23:40, 19 October 2012 (UTC)[reply]
Chaos Theory does not provide moral guidance. It especially doesn't mean that we can abandon belief in causality. Sensitive dependence on initial conditions is important, but it doesn't allow us to ignore the obvious. The legal doctrine of the chain of causation is much more relevant here -did the actions being tried lead in a foreseeable way to the consequences in question? Or did some other thing intervene to have a more significant effect on the outcome?
In any case, morality existing 'only in the mind' doesn't mean it's not absolute or commonly-held (although it might). The fear of painful death is practically universal, for example. It's a tricky (and probably intractable) question to determine if there is some kind of moral constant.
My own belief is that there is, but that it exists at some very basic level, and different self-consistent but divergent moral codes and practices can be built on that foundation. The value on which I base my own moral code is "Are other people, in general, worth saving?" - to which I answer yes, and then derive a bunch of other stuff. But one might argue that my existing experience as a liberal Christian has predisposed me to find those answers and consequences appealing and acceptable, and to believe they're logical. AlexTiefling (talk) 07:50, 20 October 2012 (UTC)[reply]
I'm a moral relativist, and I have to say that moral questions are only interesting when there are extreme disagreements. If I think you should hold the door for me and you disagree, we might get mad at each other for a while, but at the end of the day it doesn't matter. If I think the greatest glory is to sack a wealthy city, kill all the men in front of their families, and rape the women before enslaving them, and you happen to live in that city, that's a problem.
Let me try to explain moral relativism, or at least the meta-ethical Humean, sentimentalist philosophy I subscribe to. No, rape and murder are not objectively evil. Insofar as a rapist or murderer intrinsically desires to rape or murder, such an intrinsic desire cannot be rationally criticized, because it is not based on reason. In the same way, you cannot say whether my desire to eat an apple is rational or irrational; if I simply desire an apple for the sake of having an apple, that desire has no basis in reason. The only way in which reason can affect morality is if I desire something instrumentally based on false beliefs about the world. For example, if I kill someone in order to relieve thirst, that would be irrational--killing does not relieve my thirst. If I think women should be oppressed or slavery is justified because the Word of God says so, that is irrational--there is no evidence that God exists, even less evidence that any extant text is the Word of God, and no reason why the Word of God must always be moral even if it does exist (see Euthyphro dilemma).
Moral relativism does not necessarily say that all intrinsic desires must be tolerated. In the case of rapists, it is true that rapists' desires cannot be rationally criticized insofar as they are not based on reason. However, they're not the only people who have desires; non-rapists intrinsically desire to not be raped. Since rape does far more harm to the victim than good to its perpetrator, and this is even for true for murder, it is fully rational for society to put rapists and murderers into prison.
Finally, to respond to StuRat above: moral relativists like me and Hume do not find any behaviors to be evil, let alone universally evil, because there is no such thing as evil. To paraphrase Hume, "Tis not contrary to reason to prefer the destruction of the whole world to the scratching of my finger." --140.180.242.9 (talk) 00:42, 20 October 2012 (UTC)[reply]
So, morality through social contract? I prefer that you don't try to murder me, in exchange I do not try to murder you? OsmanRF34(talk) 01:02, 20 October 2012 (UTC)[reply]
Game theory is not morality. What 140.180 is saying is that he believes that you can still have social constructs that impose a de facto morality, if you are looking for certain types of aims and value certain types of individual states (e.g. if you respect individuals' and groups' desires for certain basic outcomes, which historically is a big "if" amongst human societies). You can say, "it's better to live in a society where life and property are protected, because otherwise the societies tend to fall apart in really awful ways and I don't want to live in that sort of society." You can value things even with moral relativism and you can act on those values. You just can't claim that your values are objectively better than someone else's in a philosophical sense, though that doesn't mean you can't think they are better for other reasons. --Mr.98 (talk) 02:21, 20 October 2012 (UTC)[reply]
I can think of three sources of morality. Most fundamental is the concept of atman, which I would purposefully misdescribe as, conscious beings are implementations of some fundamental quality of consciousness, and as it is this universal quality which causes consciousness, it is this universal quality which experiences it. Wrongdoer and victim do not remember living one another's lives, nonetheless it is the "same person" that experiences both. In this conception, morality exists solely in the minds of the people involved, but they are the same mind. The second source reflects Christian notions which I'll misdescribe as the revision of the universe in a series of drafts of increasing perfection; as those moments not dedicated to goodness are expunged, personalities are preserved only by doing good - this postulates an exterior judgment, a divine purpose to the evolution of the universe. The final layer would emanate from such elements of Daoism as De, in which morality can be seen in a more practical context as how the world works, which I suppose is more like a law of nature. It seems possible to me that all could be valid. Wnt(talk) 15:41, 20 October 2012 (UTC)[reply]
I think it's much more mundane than all those philosophical points. The source of moral values is the understanding that a lack of moral values runs a serious risk of causing the social group (family, tribe, nation, etc.) to disintegrate. Underlying that conclusion is the assumption that it's important keep the social group from disintegrating. "Safety in numbers", and all that. ←Baseball Bugs What's up, Doc? carrots16:29, 20 October 2012 (UTC)[reply]
That is a strawman Medeis. Moral relativists do not claim to hold absolute knowledge that morality is relative; moreover, when did anyone say that passing moral judgments is "evil"? Moral relativism is only self-refuting if you assume that relativists take their philosophy to be absolutely true, but it is perfectly possible for a relativist to accept that moral relativism is relative, i.e., it is possible that for some people there is an absolute moral truth (presumably for yourself, for instance). 24.92.74.238 (talk) 05:00, 21 October 2012 (UTC)[reply]
Your point seems to be that moral relativists are free to be inconsistant and to contradict themselves. That was my point as well.μηδείς (talk) 18:21, 21 October 2012 (UTC)[reply]

GDP Question

Which GDP is more useful--nominal GDP or PPP GDP? I always used nominal GDP since that is what Goldman Sachs used when making their economic projections, but I'm not sure which one is better to use. Futurist110 (talk) 23:55, 19 October 2012 (UTC)[reply]

Better for what? People use different measures to assess a country. Nominal GDP might be useful if you want to export something to a country. If you care about how well the populations is economically doing, you better pick PPP GDP. Add to that Gini coefficient, age expectation, and maybe some happiness index. OsmanRF34 (talk) 00:00, 20 October 2012 (UTC)[reply]
Better at making economic comparisons between various countries, including by the side of their economies. Futurist110 (talk) 00:04, 20 October 2012 (UTC)[reply]
Size of their economies? Then nominal GDP. OsmanRF34 (talk) 00:46, 20 October 2012 (UTC)[reply]
If Futurist110 means size, not necessarily. Indeed, probably not, if you are doing "real" comparisons. PPP would do better, the nominal is affected by manipulable or varying exchange rates. See our Purchasing_power_parity#Need_for_adjustments_to_GDP. E.g. If one uses nominal terms in one measuring unit, US dollars, then as Australian dollars vs US ones have doubled in value in the last decade or so, nominal overstates the "real" growth of the Australian economy, while it understates the growth of China, say. For doing financial comparisons, useful to investors, nominal makes more sense, which is probably why the Vampire squid uses it. And of course it is one less fudge factor applied to the data.John Z(talk) 01:55, 20 October 2012 (UTC)[reply]
Actually, I'd want currency exchange rates to be factored in to the comparison. Futurist110 (talk) 20:50, 20 October 2012 (UTC)[reply]
It depends what you want to compare. If you are interested in the country's influence in international trade, then you almost certainly want nominal. If you are interested in the standard of living of the inhabitants, then you almost certainly want PPP. For some things, it is a little less obvious. For all comparisons, you need to be aware of the limitations of the metric you are using. Both nominal GDP and PPP GDP have plenty of limitations.--Tango (talk) 13:20, 20 October 2012 (UTC)[reply]

Neither nominal nor PPP GDP will tell you anything about how well the population is doing economically. For market size measures it would be better to look at nominal GDP per capita at average market exchange rates. (For the sake of this discussion, I’m ignoring private consumption, disposable incomes or other non-GDP measures.) If you’re selling expensive watches, for example, a smaller economy with fewer people (e.g., Japan) might be a better bet than a larger economy with a lot of people (say, China).

Comparisons between economies also needs some clarification. What is the purpose of the comparison? If you’re trying to show that India (for example) is not as bad off as its nominal GDP per capita of Rs71,900 in 2011 might suggest, then use PPP, which was $3,652. The problems are (1) India’s GDP is generated and originally tabulated in Indian Rupees, and so a common measure such as the US dollar needs to be used; and that PPP looks like a straight-forward dollar figure, but in fact is an artificial construct that has little bearing on how large a market for imports might actually be.

In fact, PPP is totally inappropriate as a means of understanding total economic size. It is a measure of the different local currency prices of a specific urban consumer basket of goods – no trade or investment, if you please! – and not merely an alternative exchange rate. Further, if you’re interest is in international trade, or economies with a large trade component (say, Singapore), PPP is useless as it requires distorting the value of internationally traded goods and services with a purely domestic, consumer-specific modification. As such, PPP is misused more often than it is used correctly, particularly by the CIA and Wikipedia! DOR (HK) (talk) 05:47, 22 October 2012 (UTC)[reply]


October 20

Japanese crime fiction writers works into English

How many writers have had their works translated into English like Keigo Higashino?--70.31.16.144 (talk) 00:35, 20 October 2012 (UTC)Don Mustafa[reply]

Do you want a number, or a list of some (or all) of their names? -- Jack of Oz [Talk] 02:54, 20 October 2012 (UTC)[reply]
No, I don't. I was hoping you would do it for me.--70.31.17.229 (talk) 15:42, 20 October 2012 (UTC)Don Mustafa[reply]
Do what? Provide you with a bare number? Or provide you with a list of names? I doubt we could do either, accurately. We could probably come up with some names, but it wouldn't be a comprehensive list. -- Jack of Oz [Talk] 18:34, 20 October 2012 (UTC)[reply]
Translate them? —Tamfang (talk) 18:47, 20 October 2012 (UTC)[reply]

You might want to browse Category:Japanese crime fiction writers.Alansplodge (talk) 19:19, 20 October 2012 (UTC)[reply]

Biased ads?

I just saw an ads on youtube and that ads is on this website. This is pure propaganda and perhaps anti-Americans. It looked like a movie at first glance but it is plainly just an ads about bad future for America. I'm wondering if this kind of ads is acceptable within America's laws.184.97.240.247 (talk) 07:46, 20 October 2012 (UTC)[reply]

Of course that is allowed. What's so shocking about a website or a Youtube film propagating a (political) message? I don't think you have really grasped the concept of 'freedom of expression'. - Lindert (talk) 08:15, 20 October 2012 (UTC)[reply]
Of course it is allowed. However, it also is a very very stupid and quite racist movie, so people should use the same right to free expression to point that out. --Stephan Schulz (talk) 09:41, 20 October 2012 (UTC)[reply]
What's racist about it? --Tango (talk) 13:32, 20 October 2012 (UTC)[reply]
Presenting "the Chinese" in a 19th century Yellow Peril manner, including schadenfroh collective laughter at the end?--Stephan Schulz (talk) 14:01, 20 October 2012 (UTC)[reply]
Laughing at Americans is hardly a stereotypical Chinese trait... the whole world does that... ;) --Tango (talk) 14:23, 20 October 2012 (UTC)[reply]
I wouldn't say the whole world would laugh at Americans. It would more like the whole world admire Americans on how Americans are the leading motivation in technology and science. 184.97.240.247 (talk) 17:38, 20 October 2012 (UTC)[reply]
Not to mention stand-up comedy! --Stephan Schulz (talk) 18:07, 20 October 2012 (UTC)[reply]
I don't think it really counts as propaganda, it's just a political ad. And it isn't anti-American, it is just against the current government. You can be opposed to a government without being opposed to the country - that's a fundamental principle of democracy. (The UK describes the main opposition party as "Her Majesty's Most Loyal Opposition" to emphasise the fact that they are opposing the government but are loyal to the Queen and country.)--Tango (talk) 13:32, 20 October 2012 (UTC)[reply]
Are you saying this ads is supposed to be an ads that anti-Obama and pro-Romney? Then whoever did this ads did a bad job. I think this ads isn't just anti-Obama but also anti-Americans overall and an insult to Americans. It clearly depicts the future of all Americans being enslaved (that's the same thing as working for them just to pay of the debt) by China.184.97.240.247 (talk) 17:38, 20 October 2012 (UTC)[reply]
As a patriotic American myself, I don't see it as anti-American. It's a cautionary reminder that we've become too dependent on other countries, and this is just one example extrapolated to an extreme. ←Baseball Bugs What's up, Doc?carrots20:44, 20 October 2012 (UTC)[reply]
I don't see it as "racist". Now, if they had the guy talking with a fake Charlie Chan type accent, that could be racist. It's just an anti-Obama ad that plays on fears - as do some of the pro-Obama ads. The general theme of all these ads is: "If you don't vote for our guy, America will collapse." The irony is that the creators of this ad seem to have forgotten that it's Romney's buddies in the business community that outsourced so many jobs to China, helping fuel China's economy and helping to allow China to acquire so many American bonds. There's now a term for that kind of forgetfullness: "Romnesia". ←Baseball Bugs What's up, Doc?carrots16:04, 20 October 2012 (UTC)[reply]
And Nixon started it all by recognizing China. If Americans can't recognize America on the map, why should they recognize China ? :-)StuRat (talk) 16:23, 20 October 2012 (UTC) [reply]
Yes. Mike Royko wrote a funny piece in 1972 as if he were a time traveler visiting Nixon in 1950 and telling him in vague terms what the future would hold. And of course Nixon was shocked beyond belief that the (unnamed) sitting President would actually be buddying up to Red China. ←Baseball Bugs What's up, Doc? carrots16:45, 20 October 2012 (UTC)[reply]
@Bugs: How do you remember a Mike Royko column from 1972 and do you have a link?  :) A Quest For Knowledge (talk) 20:53, 20 October 2012 (UTC)[reply]
How do I remember something I saw once in 1972 and haven't seen since? It's a curse, what can I say? I just now googled the subject, and here it is. See if you can get to it.[15] If not, then google something like [mike royko nixon china 1972] as I did. ←Baseball Bugs What's up, Doc? carrots21:08, 20 October 2012 (UTC)[reply]
Thanks for the link! A Quest For Knowledge (talk) 01:14, 21 October 2012 (UTC)[reply]
Their biggest problem is that they don't realise China doesn't actually own that much US debt. Most US public debt is owned by Americans. Less than 10% by China. --Tango (talk) 16:38, 20 October 2012 (UTC)[reply]
That's another conveniently-left-out fact. "Romnesia" is reaching epidemic proportions. ←Baseball Bugs What's up, Doc? carrots16:45, 20 October 2012 (UTC)[reply]
An even bigger problem, I think, is the failure to realize that this hurts China more than it does us. China relies on being able to sell things to the US -- having huge amounts of our currency is just a nuisance to them. It's true that they could do us some damage, but if they did, they would be devastating their own economy at the same time. Looie496 (talk) 16:54, 20 October 2012 (UTC)[reply]
Agreed! China should afraid about this than so do Americans. I can't believe the laws actually allow some stupid, wrong-fact, insult ads like this.184.97.240.247 (talk) 17:38, 20 October 2012 (UTC)[reply]
Whenever it is illegal to be wrong about something, that means the government has a monopoly on truth. A very dangerous thing. -Lindert (talk) 17:41, 20 October 2012 (UTC)[reply]
Yup... in a democracy, you have the right to be wrong... and Freedom of Speech means you have the right to clearly demonstrate to everyone else just how wrong you are. Blueboar (talk) 17:59, 20 October 2012 (UTC)[reply]
I can sorta see what the OP's concern is. In most developed countries there are laws against misleading and deceptive advertising. But they apply only to businesses. A political machine, although it seeks to manage not just other businesses but an entire state or nation, is the furthest thing from a properly run business one can possibly imagine. They're above such tawdry concerns; they're all about principles and rights and truth. (Cough) -- Jack of Oz [Talk] 18:31, 20 October 2012 (UTC)[reply]
However, in the UK, TV advertising by political parties is banned - they get free Party political broadcasts instead. This has broad public support."UK advertising rules save us from the climate lobbying mess in the US" However, in old-fashioned print, there's a bit of a free-for-all. Alansplodge (talk) 19:11, 20 October 2012 (UTC)[reply]
There's a lot of soapboaxing in this thread, not that I'm surprised. To directly answer the OP: the ad criticizes the Obama administration. It came out years ago, far before the current presidential race. The right to criticize the government is the most sacred principle of democracy; without it, informed voting is impossible. In the US, hate speech is protected under the First Amendment, and it's unconstitutional for either the government or the states to enact laws prohibiting it (see hate speech#United States). Of course, even if hate speech were illegal, that's not what the ad is. Anyone who thinks the Chinese wouldn't be glad to see the world's hyperpower collapse knows nothing about realpolitik. --140.180.242.9 (talk) 19:30, 20 October 2012 (UTC)[reply]
That's it in a nutshell. There's a lot of slack cut for advertising in general, and political advertising is practically anarchy. I've seen more bad lies this past year than in my golf game. But it's part of the blessing of free speech. Once we stop seeing those kinds of ads, we'll know we're screwed. ←Baseball Bugs What's up, Doc?carrots20:41, 20 October 2012 (UTC)[reply]
Succesful troll is succesful. --Saddhiyama (talk) 21:43, 20 October 2012 (UTC)[reply]

Wow, what a total load of crap this thread is with outright politicking and not a single encyclopedic question or answer! This is yet another good bit of evidence for the ANI that will eventually shut down the ref desk. μηδείς (talk) 02:20, 21 October 2012 (UTC)[reply]

On what basis? The OP asked a reasonable question and he got answers. Executive summary of the answers: "Yes". ←Baseball BugsWhat's up, Doc? carrots02:59, 21 October 2012 (UTC)[reply]
Hey hold your mouth dude. There is no reason to talk crap. I don't know where you live but at where I live Politic is part of encyclopedia. And I don't care if one day someone shuts this ref down. None of any of us business. It is open now, that all it matters.184.97.240.247 (talk) 02:36, 21 October 2012 (UTC)[reply]

Is US a founding member of the League of Nations?

This map on WP[16] shows United States as a founding member of the League of Nations. And yet the US senate never ratified the admission treaty. The map has been there for 5 years now and has gone through dozens of revisions under various editors, so I assume the majority voice, and hence the map, is correct and I'm the one having a serious misunderstanding somewhere.A8875 (talk) 10:53, 20 October 2012 (UTC)[reply]

Maybe I'm colour-blind, but the map you refer to shows the US in the "never members" category. The big country in North America is Canada ;-).--Stephan Schulz (talk) 11:21, 20 October 2012 (UTC)[reply]
That's what I see too, and not being able to distinguish grey from blue would be an usual form of colour-blindness, so I doubt we both have it! I think the OP is getting a little confused geographically. --Tango (talk) 13:22, 20 October 2012 (UTC)[reply]
That map needs serious work. How Occupied France can be considered a "founding state" that stayed (!) to the end, while Vichy France is portrayed as a "founding member" (!) that left is beyond my mortal mind's comprehension. μηδείς (talk) 02:15, 21 October 2012 (UTC)[reply]
It's a messy situation. The Vichy government withdrew from the League on April 18th, 1941. At that time, it was recognised as the legitimate government of France by many states, including the US and Canada. In 1944, the Provisional Government of the French Republic took over and was widely recognised at least by the Allies. It retroactively declared most of the acts of the Vichy government invalid. So in 1942, France was not a member. But in 1945, it had been a member all along. Time travel will have been going to have made grammar more complicated - even if it's only political. Maybe the best way to represent this on the map by having a separate colour for France. --Stephan Schulz (talk) 04:45, 21 October 2012 (UTC)[reply]

Last inmate executed by hanging in the U.S.?

Can anybody help me? Thank you. Iowafromiowa (talk) 12:35, 20 October 2012 (UTC)[reply]

See Capital punishment in the United States#Methods. There is a table showing the date, name and location of the last executions by various methods, including hanging. --Tango (talk) 13:25, 20 October 2012 (UTC)[reply]

Thank you. Iowafromiowa (talk) 13:42, 20 October 2012 (UTC)[reply]

You are most welcome. --Tango (talk) 14:24, 20 October 2012 (UTC)[reply]
Resolved

Provide Wife's Means of Subsistence

Are there European countries in which, according to marriage laws, a husband is obliged to provide his wife's means of subsistence, even though the wife has an income of her own? Is it in common law, in statute law, or in canon law? Any comment would be appreciated.--Omidinist(talk) 15:58, 20 October 2012 (UTC)[reply]

See Abandonment#Abandonment_of_family. That one paragraph seems to be all we have on the subject, though... If anyone can find some good sources, we should expand it. --Tango (talk) 16:43, 20 October 2012 (UTC)[reply]

The Moon

Hi! I was wondering what the regulations on owing the Moon are. Is it ,like Antarctica, devoted to research and that no country is allowed to own any land on it? Thanks, B. Jakob T. (talk) 17:43, 20 October 2012 (UTC)[reply]

Regulating the kind of thing you're on about was the subject of the Moon Treaty. But that's not been ratified, so isn't much real use. TheOuter Space Treaty should apply, which is a start. -- Finlay McWalterTalk 17:49, 20 October 2012 (UTC)[reply]
The Moon Treaty has indeed been ratified, and by 13 nations that are REALLY BIG when it comes to lunar exploration - Australia, Austria, Belgium, Chile, Kazakhstan, Lebanon, Mexico, Morocco, Netherlands, Pakistan, Peru, Philippines, and Uruguay. I bet you're just aching to know who was the first Lebanese lunonaut.  :) -- Jack of Oz [Talk] 18:15, 20 October 2012 (UTC)[reply]
That is all well and good for the legal status... but culturally the moon is American... after all, it has an abandoned car in the front yard! :>) Blueboar (talk) 18:22, 20 October 2012 (UTC) [reply]
"If you've had an abandoned car in your yard for 40 years, you might be a redneck." ←Baseball Bugs What's up, Doc? carrots20:36, 20 October 2012 (UTC)[reply]
As others have said, the legal situation is very vague at the moment. That is because it hasn't been in anyone's interests to get it sorted out. Once people start putting permanent bases on the moon, and particularly once they start mining its resources, then ownership of land and mineral rights will become and issue and will probably get sorted out. At the moment, it is purely academic. There is so much moon to go around that it probably won't be particularly controversial, although things like peaks of eternal light may be rare enough and valuable enough for people to want to argue over.--Tango (talk) 17:35, 21 October 2012 (UTC)[reply]
Supposing gold was discovered on the moon, it would be an interesting fantasy to try and go get it before anyone else does. But until such time as it would take less money to get there and back than the value of whatever has been mined, it's impractical. ←Baseball BugsWhat's up, Doc? carrots02:28, 22 October 2012 (UTC)[reply]

American Christians and Bible Study

Why do American Christians often attend Bible Study sessions on Wednesdays? What's up with Wednesdays?75.185.79.52 (talk) 19:17, 20 October 2012 (UTC)[reply]

Well, it's not difficult to find counter-examples; Monday Night Bible Study,Tuesday Bible Studies (TBS),Thursday Men’s Bible Studies, Night Bible Study and Saturday Night Bible Study. Alansplodge(talk) 19:35, 20 October 2012 (UTC)[reply]
Nevertheless, in a lot of places where I've lived in the US lots of people go to their churches on specifically Wednesday, and there are many more cars in the church lot on Wednesday evening then any other evening. So the question is a good one -- why specifically Wednesdays?Duoduoduo (talk) 20:01, 20 October 2012 (UTC)[reply]
Because Wednesday evening is halfway between two Sunday mornings, perhaps. Pais (talk) 20:29, 20 October 2012 (UTC)[reply]
(ec) Wednesday sermons are a tradition that dates back to at least the time of the Reformation. This book mentions the practice in 16th century Germany and the Netherlands. I don't think there is a definite answer to the why-question though, other than it being roughly in the middle of the week (note that Thursdays are just as nearly in-between Sundays as Wednesdays). -Lindert (talk) 20:33, 20 October 2012 (UTC)[reply]

What happened to the crew of the Mary Celeste?

I read the article but couldn't sort out what really happened to them. What a mystery. Is it true there was hot tea served? Thank you.Iowafromiowa (talk) 19:17, 20 October 2012 (UTC)[reply]

The article says "Popular stories of untouched breakfasts with still-warm cups of tea on the cabin table are untrue..." and has a list of theories. -- BenRG (talk) 22:22, 20 October 2012 (UTC)[reply]

TOEFL test

There is a question regarding this test which I couldn't find. An answer from someone who has taken this test before on computer-version is very appreciated, otherwise point out to me the answer with confirmed source. On the "speaking section", according to my Princeton Review TOEFL book. I have to read a passage then listen to an audio then answer some questions about it. So first of all, do I suppose to read out loud when I read the passage or just in my head? I know that when I'm answering the questions I must say them out loud because it is speaking section. And am I doing this with a supervisor? If not then how the heck the computer can grade my level of speaking? I don't understand how the speaking section going to be like.184.97.240.247 (talk) 20:16, 20 October 2012 (UTC)[reply]

For the second part of your question, this might be helpful. It explains: Your recorded responses are sent to the ETS Scoring Network, where three to six certified human raters score them holistically on a scale of 0 to 4. The average score on the six tasks is converted to a scaled score of 0 to 30. So the computer doesn't grade your answers; people do.
For the first part of the question, I couldn't find a definitive answer. The instructions just say you have a short time to "read" a passage, then listen to an audio clip on the same subject, then answer a question that requires you to synthesize the information from both. It doesn't state explicitly that the "reading" has to be out loud, but I have found several sites that say you should take notes while you are reading, eghere (see questions 3 & 4). It would be quite difficult to take notes while concentrating on reading something aloud, all in only 45 seconds. A Google search suggests you are not the only person asking this question, and there are contradictory answers out there. However, I cannot find any site that says you are marked on your reading of the passage, or any criteria for marking. On the other hand, I can find many that say you are marked on the response you provide to the questions, andhere are the marking criteria for this. Therefore my instinct is that in this part of the test you are not being assessed on your ability to read out loud in addition to your ability to discuss material you have read and heard, and reading aloud is not required, which is why the wording does not specify it. But I cannot give you an absolute reference to confirm this, and if anyone else can provide one it would be good. - Karenjc 11:03, 21 October 2012 (UTC)[reply]
I'm very appreciated your effort on searching this up. Your reasons are very good so by using logic I can almost certain that I don't need to read out loud. It wouldn't make any sense if I have to take note in read out loud in 45 seconds. Either way I will be ready on the test day.184.97.240.247 (talk) 06:11, 22 October 2012 (UTC)[reply]
You're welcome. Best of luck with the test. - Karenjc 16:40, 22 October 2012 (UTC)[reply]
I have taken the TOEFL twice before; you are NOT supposed to read the text aloud. You will have to read it normally, and you will only "speak up" when you answer the questions afterwards. Your voice will not be recorded when you read the text.
In order to get comfortable before your test date, I suggest you download the sample test from the ETS website. This will help clear any doubts you have and make you more familiar with the test's structure. Hisham1987 (talk) 06:07, 23 October 2012 (UTC)[reply]

Currently to this date, I wonder how often airplanes or boats going through this triangle area?184.97.240.247(talk) 20:30, 20 October 2012 (UTC)[reply]

Why is it named "Bermuda Triangle", I especially concerned about Bermuda, what does that area has to do with Bermuda? I can understand the triangle part but not the Bermuda? Who was the first to name it "Bermuda Triangle". The name couldn't just pop up from nowhere.184.97.240.247 (talk) 20:35, 20 October 2012 (UTC)[reply]
It's one corner of a triangle drawn on the map whose purpose is to contain the allegedly "mysterious disappearances" in the area. The other two corners are basically Puerto Rico and Florida, which don't sound quite as exotic. According to the article, the term first turned up around 1964. It was a big deal for a while, and then the 1975 book came out and demolished the claims, and the term has since passed into popular culture as one of those joke-myths that everyone knows about but has never actually seen, like Bigfoot, the Loch Ness Monster, or the Great Pumpkin. ←Baseball Bugs What's up, Doc? carrots20:58, 20 October 2012 (UTC)[reply]
So do people still crossing this triangle often even in today?184.97.240.247 (talk) 02:33, 21 October 2012 (UTC)[reply]
Define "often". ←Baseball Bugs What's up, Doc?carrots02:55, 21 October 2012 (UTC)[reply]
Often probably means several hundred boats and planes per day traverse at least part of the Bermuda Triangle. Considering that the ports and airports around Miami are busy, and that The Bahamas lies pretty much entirely within the Triangle, I imagine at any one point in time there are dozens if not hundreds of air and sea vessels within the space defined by the triangle. --Jayron32 03:58, 21 October 2012 (UTC)[reply]
Here is a video of the total number of airplane flights in one 24-hour period, worldwide. There's a significant number passing over the Triangle every day. And that doesn't count boats. --Jayron32 04:03, 21 October 2012 (UTC)[reply]
The OP can also go to any number of online sites like Marinetraffic.com, which display in real time all sorts of pleasure and cargo vessels sailing in those waters. It will also show you pictures (not real time) of many of those ships, including some kick-ass megayachts. A pleasant time-waster. Textorus (talk) 05:37, 22 October 2012 (UTC)[reply]

Execution by gas

What is the problem with lethal gas executions? It seems less painful, comparing itb to the electric chair, hanging or shooting, and maybe even comparing it to the lethal injection. Gorgeop (talk) 21:30, 20 October 2012 (UTC)[reply]

You would prefer choking to death over simply falling asleep? ←Baseball Bugs What's up, Doc? carrots21:36, 20 October 2012 (UTC)[reply]
You don't have to choke to death, depending on the gas. Aspiring some gases can make you fall asleep and die. Not even heavy breathing.Gorgeop (talk) 21:44, 20 October 2012 (UTC)[reply]
(ec) Let's interview people who have been through each, and compare their experiences.
More seriously, cyanide poisioning, from all indications we can see from the outside, is a dreadful way to go. The morituri go through convulsions that deeply disturb witnesses. You can tell yourself, if it makes you feel better, that they are not really conscious of suffering during these, but as far as I know there is no evidence for that position. --Trovatore (talk) 21:38, 20 October 2012 (UTC)[reply]
And where does it say that someone has to witness a death sentence? Gorgeop (talk) 21:44, 20 October 2012 (UTC)[reply]
In many state laws. Blueboar (talk) 21:46, 20 October 2012 (UTC)[reply]
But are they forces to watch all the execution? Gorgeop (talk) 22:53, 20 October 2012 (UTC)[reply]
I second that question. It strikes me as unusual at best that there would be laws actually forcing anyone to watch. (But then, it strikes me as odd enough that there are audiences at executions in the first place, it seems so... morbid.) 164.71.1.221(talk) 05:10, 22 October 2012 (UTC)[reply]
Imagine it would be difficult to execute somebody with nobody in the room. And what you're suggesting to me sounds just like a gas based version of the injection, I can see a lot more things that would go wrong with gas vs liquid's, easier to leak out etc--Jac16888 Talk 21:49, 20 October 2012 (UTC)[reply]
What does it have to do with anything whether there are witnesses or not? I mentioned the witnesses as evidence of suffering; the witnesses are disturbed because of what they think the person is going through. I wasn't talking about the effect on the witnesses as a consideration in itself.--Trovatore (talk) 21:51, 20 October 2012 (UTC)[reply]
The gas doesn't have to be poisonous, helium would kill you if you breadth enough of it, and there are no convulsions either.Gorgeop (talk) 22:53, 20 October 2012 (UTC)[reply]
Nitrogen works too, and is much cheaper. —Tamfang (talk) 22:32, 21 October 2012 (UTC)[reply]
The British Royal Commission on Capital Punishment (1949-53) (Cmd. 8932) examined alternative methods to hanging. It assessed them against the requirement of humanity, which it defined as being that the act of execution be quick and free from any additional 'poignancy', and also that it produce instantaneous unconsciousness passing quickly into death. They concluded that hanging was far quicker than the gas chamber, and that the requirement to remove almost all the prisoner's clothes to prevent pockets of gas lodging in them was unwelcome (para 727). They also mentioned that "while hanging is tainted by the memory of its barbarous history, 'gassing' is tainted by more recent but not less barbarous associations" (para 732). The British Medical Association put forward the idea of using Carbon Monoxide as a lethal gas, which would not necessarily require a gas chamber, but the commission found that forcibly applying a mask to a conscious prisoner would involve a distressing and unseemly struggle (para 736). The Commission did not recommend a change in method. Sam Blacketer (talk) 23:02, 20 October 2012 (UTC)[reply]
The idea that hanging causes "instant unconsciousness" is rather dubious. If the spinal cord is severed, the victim cannot move his body, so this may look like unconsciousness to observers, but severing the spinal cord in itself does not in general cause unconsciousness.--Trovatore (talk) 20:03, 21 October 2012 (UTC)[reply]
The Royal Commission based its view on a memorandum from HM Coroner for North London Bentley Purchase (a fascinating character, incidentally) who had conducted post-mortem examinations on prisoners hanged at Pentonville. Purchase said that in all 58 examinations, the immediate cause of death was fracture dislocation of cervical vertebrae and the report does not doubt that it caused instantaneous unconsciousness. The fact that the heart may continue to beat for up to 20 minutes afterwards is described as "a purely automatic function" (para 714). Not being medically trained I don't have any base to enter this discussion but thought it worth mentioning why the Royal Commission came to its decision to endorse long drop hanging as humane. Sam Blacketer (talk) 21:26, 21 October 2012 (UTC)[reply]
Uh huh. And how would they know, exactly, that it caused instant unconsciousness? I call bullshit. --Trovatore (talk) 21:42, 21 October 2012 (UTC)[reply]
If the brainstem is violently separated from the midbrain, disrupting the latter, unconsciousness would be instantaneous. Not that I am saying such separation always happens. But that is the purpose of the long drop. μηδείς (talk) 23:30, 21 October 2012 (UTC)[reply]
Is that the purpose? I thought it was to subluxate the cervical vertebrae and so cause the spinal cord to be severed. But severing the spinal cord does not in general cause instant unconsciousness, which makes me suspicious. The fact that there is an incentive for the executioners to claim instant unconsciousness, combined with the difficulty of determining that, makes me suspicious too. --Trovatore (talk) 09:48, 22 October 2012 (UTC)[reply]
Same goes for injection: one of the drugs used causes paralysis, so that witnesses cannot see if the victim suffers. Presumably that's also why the occupant of the gas chamber was (is?) gagged. —Tamfang (talk) 22:32, 21 October 2012 (UTC)[reply]
That's not very accurate. He is first made unconscious with sodium pentathol, then killed with various paralyzing agents that stop the heart. He's not paralyzed while conscious for the benefit of the audience. I can tell you that you don't feel having four wisdom teeth extracted under sodium pentathol. So you aren't exactly suffering from a mere paralytic while under from SP already. μηδείς (talk) 03:10, 22 October 2012 (UTC)[reply]
Sodium pentothal is an "ultra short-acting" thiobarbiturate. There is debate over whether the convict might regain consciousness in time to experience not being able to breathe.
I doubt you had four wisdom teeth extracted using just pentothal. Pentothal is used for quick induction of anesthesia, but then another anesthetic is used to maintain it. --Trovatore (talk) 09:51, 22 October 2012 (UTC)[reply]
They had some trouble reviving me (I had no detectable pulse for a minute) and I was told that it was a reaction to the sodium pentathol. But it's surely a technical matter of getting the prootcol right. They can certainly knock you out and kill you by the time you wake up.μηδείς (talk) 17:34, 22 October 2012 (UTC)[reply]
Possibly, but this is not really known. If they actually cared, they wouldn't use an ultra-short-acting one at all. You may havereacted to the pentathol, but I'm virtually certain it was not the only anesthetic they used on you; it just isn't done.--Trovatore (talk) 20:12, 22 October 2012 (UTC)[reply]
Killing with helium would be an unusual punishment, therefore not constitutional in the US. Other countries simply don't care much about humane executions (what's the point after all. You are executing people, it will always be horrible). I personally believe that the lethal injection is maybe the 'cleanest.' No blood, no kicking, no screaming (well maybe before the execution a little). OsmanRF34 (talk) 00:13, 21 October 2012 (UTC)[reply]
That's a nice bit of OR, but the punishment is not breathing helium, but execution, which itself is not unusual in capital cases. As for any random person dying, by helium is hardly more unusual than by cyanide, as compared to car accident or lightning strike. μηδείς (talk) 02:06, 21 October 2012 (UTC)[reply]
The first use of gas, Old Sparky or injection was "unusual" too. The point of the Eighth Amendment, as I understand it, is to forbid sentencing judges to say: "And because the victim was especially popular and hanging is too good for you, we're gonna kill you in a special way."—Tamfang (talk) 22:32, 21 October 2012 (UTC)[reply]
That's utter nonsense. You could just as well argue that no one should be imprisoned in a newly built prison because, no one ever having been imprisoned there before, it would be unusual for someone to be imprisoned on the site. Unusual is simply meant as a broadening synonym of cruel. Torturing someone to death in some slow strange way would fall under the prohibition. Killing them with a new technology, like a firing squad, instead of an old one, like a gallows, would not be. μηδείς (talk) 23:27, 21 October 2012 (UTC)[reply]
Medeis, if you mean my paragraph above (or some part of it) is utter nonsense, where's your disagreement? —Tamfang (talk) 01:00, 22 October 2012 (UTC)[reply]
Your use of what might be scare quotes makes it hard for me to interpret whether we are agreeing here or not. In any case, what would be nonsense would be a literal argument that because a certain tool was first being used for an execution that it would be "unusual" in the forbidden sense. Just like I argued, if no one had ever been imprisoned in Smithville before because there was never a prison there before, it would be unusual in a trivial sense for someone to be imprisoned in Smithville. The same with claiming that execution by firing squad with a machinegun would be unusual, or asphyxiation by gas instead of rope. That is what would be utter nonsense, a total confusion of the method with the result. I do agree that a method chosen because it was intentionally torturous would be problematic, since not just death, but disproportionate suffering might be the intent. But "cruel" covers that. My guess is that what was meant by including "unusual" was in part to hedge against people who would say that hanging itself was cruel. The case of the perjurer Titus Oates as mentioned in Eighth Amendment to the United States Constitution would seem to be relevant. Once again, in his case it was the punishment, not the tool, that was viewed as unusual. μηδείς (talk) 02:23, 22 October 2012 (UTC)[reply]
Might be scare quotes, might not, I can't decide either. – My suspicion is that cruel and unusual was a set phrase in judicial jargon.—Tamfang (talk) 05:45, 22 October 2012 (UTC)[reply]
Of course WHAAOE: Cruel and Unusual Punishment. --TammyMoet (talk) 10:10, 22 October 2012 (UTC)[reply]
I thought I had given (and have just added) this link http://legal-dictionary.thefreedictionary.com/Cruel+and+Unusual+Punishment in my last response--it explains the origin of the term in the English Bill of Rights of 1689 in regards to the Titus Oates case. What was unusual was the prescribed punishment, not the means used to carry it out. μηδείς (talk) 17:39, 22 October 2012 (UTC)[reply]

I think the discussion and links at Gas chamber#United states will answer the original question. Newyorkbrad (talk) 00:17, 21 October 2012 (UTC)[reply]

The BBC had a programme not so long ago, presented by Michael Portillo, examining the most humane way of judicially killing someone. His conclusion was the use of nitrogen. He also concluded that it is not used because, just before they die, people become really, really happy - and executing a happy person does not fit with concepts of justice and revenge. (As an aside, there are people in the UK who are not happy they brought him out of the nitrogen intoxication...) --TammyMoet (talk) 09:35, 21 October 2012 (UTC)[reply]
There's no shortage of morbid humor connected with death and executions. To have the guy literally die laughing might be a bit much. ←Baseball Bugs What's up, Doc? carrots15:27, 21 October 2012 (UTC)[reply]
See also Monty Python's The Meaning of Life, Part VII, scene 1. Newyorkbrad (talk) 02:28, 22 October 2012 (UTC)[reply]
ObWHAAOE: Nitrogen asphyxiation. Marnanel (talk) 20:08, 22 October 2012 (UTC)[reply]

Main Factors for Anti-Semitism

I was wondering, but are the large presence of Jews and/or Muslims in a particular country or area the main causes of anti-Semitism? I mean, most of the countries that have more anti-Semitism are the ones with either a lot of Jews and/or with a lot of Muslims. In countries where they are very few Jews or Muslims (many countries in South America, China, Japan, etc.) there is almost no anti-Semitism. Here's another example--it appears that Hungary has more anti-Semitism than any of the other former Warsaw Pact countries (excluding the former USSR), as evidenced by the strong showing of Jobbikin the polls there. Hungary also has the most Jews as a % of the population right now out of any of those former Warsaw Pact countries.Futurist110 (talk) 22:13, 20 October 2012 (UTC)[reply]

First off, anti-semitism has a particular meaning that does not include prejudice against Islam or semitic speakers (like e.g. Arabs) in general. Secondly, I don't think your anecdotal evidence is very useful. Jobbik is generally xenophobic, not specifically anti-semitic, for example. In general, conditions where there is a notable presence of a minority (to act as a nucleus for fear and resentment), but not a strong presence (which leads to familiarisation) are most conductive to prejudice. However, there are plenty of historical circumstances that lead deviation from the general rule. --Stephan Schulz (talk) 22:41, 20 October 2012 (UTC)[reply]
Stephen, I don't think Futurist110 was saying anti-Semitism applies to Arabs, I think he was saying that in an area with a lot of Arabs, anti-Semitism is more common.
Futurist, that is tough to say, as tough as it is to assign a "factor" for any form of irrational hatred, be it anti-Semitism, Islamophobia, white supremacism, etc. Certainly, Europe in the Medieval Ages had hardly any Arabs but was the most anti-Semitic place of its time. A reason for a surge in such hatred could be economic reasons, as it's easy to scapegoat and blame others rather than yourself (consider whether Adolf Hitler would've been elected in Germany had there not been such huge inflation, or whether he would've just been viewed as a nut). In Greece, where the economy sucks, the far-right Golden Dawn party has been elected, which is anti everyone who isn't "pure Greek," the whole shebang. --Jethro B 22:59, 20 October 2012 (UTC)[reply]
Yeah, I meant that a lot of Muslims nowadays are anti-Semitic due to the whole Arab-Israeli conflict, and thus places with a lot of Muslims often have more anti-Semitism (even if there are virtually no Jews left in them right now, such as in most of the Arab and Muslim worlds). Likewise, if there are a lot of Jews in an area, then resentment and hatred against them often tends to be created, especially if the population is uneducated, as was the case in Europe in the Middle Ages and even in the 19th and early 20th centuries, since in a lot of cases Jews tend to become wealthier and more successful than their non-Jewish neighbours. Hitler was viewed as a nut before the Great Depression by many Germans, but a good number of Germans probably did share his anti-Semitic views, just not to the genocidal extent that Hitler had them. As for Jobbik, it is not exclusively anti-Semitic, but it is still pretty anti-Semitic, considering how Csanad Szegedi had to leave the party after it was discovered that his mother was ethnically Jewish. Futurist110 (talk) 23:53, 20 October 2012 (UTC)[reply]
I don't know how true the part about Jews tending to be wealther and more successfull... I mean, surely that was claimed by many anti-Semites, but until recent times, most European Jewry tended to be poor and live in a shtetl. Now, with religious freedom and some better mobility and rights, the situation has progressed. --Jethro B 00:58, 21 October 2012 (UTC)[reply]
Sheesh. Futurist, if there's a significant presence of Jews in a certain place, that means there's a lot of people to hate * I F *you're anti-semitically inclined. But it could never be said that the presence of the Jews is the cause of anti-semitism, as per your opening question. That would legitimate anti-semitism by making Jews inherently hate-worthy, and making it their own fault for being Jews. That's not even remotely acceptable as a good faith question, in my book.
Unless, of course, you have great difficulty in expressing yourself in English. But given all the questions you've asked here, that is far from my impression of you. I recommend you have a cold shower and think more clearly even about whether you really need to ask a question at all, before launching into such an ill-considered utterance. There's a lot to be read about such subjects, and you need to do some reading and googling and your own research first. Not just pose uncritically the first question that happens to pop into your head each morning. -- Jack of Oz [Talk] 00:08, 21 October 2012 (UTC)[reply]
What the heck are you talking about? Of course the presence of Jews is one of the causes of anti-Semitism. It's hard to hate "the people o'er yonder" with a passion if those people are 5000 km away and have little impact on your life. That's just human nature. On the other hand, if Jews are living within your own country, and you believe that them to be greedy God-killing infidels who deliberately sabotaged your country's last war effort, and that they're taking all the good jobs, controlling all the banks, inciting Communist revolutions, etc., you'd be more likely to be anti-Semitic. The naive view that you seem to have about anti-Semitism is that a bunch of hateful people magically gathered somewhere and decided to hate a randomly picked group of people. You seem to ignore the religious, economic, historical, and cultural reasons behind anti-Semitism, as well as the human psychology of hatred.
Jack's answer is not in good faith, but the answers above his answer Futurist's questions insightfully. It's easy to scapegoat other people when your economy is in the dumps, and it helps that Jews historically controlled a disproportionate percentage of the economy. Jobbik is not necessarily anti-Semitic; it's simply xenophobic, so the fact that Hungary has a lot of Jews makes Jews a target. Anti-semitism is mainly Islamic nowadays primarily due to the Arab-Israeli conflict. Unlike many other cases of ethnic hatred--between Irish Catholics and Protestants, between Serbs and Croats/Bosniaks, between Basque separatists and the Spanish, etc--Islamic anti-Semitism of the type seen today has no deep historical roots. --140.180.242.9 (talk) 00:38, 21 October 2012 (UTC)[reply]
My answer was most certainly in good faith. What would you make of any suggestion that the cause of rape is the presence of women? Or the cause of fraud is the existence of money? Suggesting that Jews are the cause of the hatred visited upon them is redolent of classic "blame the victim" mentality. Words have meaning. Use them with caution, or accept the consequences. -- Jack of Oz[Talk] 01:16, 21 October 2012 (UTC)[reply]
Jack, I'm inclined to believe that Futurist's question is well-intentioned but phrased extremely awkwardly. His userpage identifies him as an ethnic Jew (although not religious) from Ashkenazi Jewry, a member of Wikiproject Israel, and from my interactions with him, he isn't a self-hater. --Jethro B 00:58, 21 October 2012 (UTC)[reply]
In that case I reiterate my comments about thinking first and asking questions here only when other avenues of enquiry have been exhausted. - Jack of Oz [Talk] 01:16, 21 October 2012 (UTC)[reply]
IP guy, I'm inclined to disagree somewhat. Europeans in the 19th century had an inferior view of Africans, despite the fact that most didn't visit Africa or have large African populations. Irrational hatred doesn't need a motive or a factor. A surge in such irrational hatred can sometimes have a motive or factor, such as scapegoating someone... But the origins of such irrational hatred is often different. To a certain extent though, you are correct. Many far-right parties in Europe now focus on Arabs, rather than Jews, since although Jews have historically always been a tiny percentage of the world and Europe, in the past few decades there have been many Arabs coming to Europe, thus becoming a new target for these parties, who set a "higher priority" for them. That doesn't reduce their feelings towards Jews though (except perhaps in the case of Marine Le Pen, although even that's debatable).
Also, while I agree that anti-Semitism in the Arab world became prominent from the Arab-Israeli conflict (which certainly isn't any reason to hate a religion), there are certainly historical roots. Jews under Arab rule had good times and bad times. In Spain, during the time of the Moors, the Jews were lucky and had a better life than in most European countries. At other times, it was not such. Consider what Saudi Arabian King Ibn Saud told a British official in 1937 as his reasoning for anti-Semitism, "Our hatred for the Jews dates from God’s condemnation of them for their persecution and rejection of Isa (Jesus) and their subsequent rejection of His chosen Prophet." The Arab-Israeli conflict is not mentioend here as a reason. He even says that it'd be better for England to control the Mandate of Palestine for hundred years, which would contradict the current attempts of Palestinian/Arab statehood! (source - [[17]]) The Banu Qurayza in the 600's are another example. So again, while keeping in mind that Jews generally fared better under Araab rule than Christian rule historically, there are deep historic anti-Semitic roots in the Arab world. --Jethro B01:10, 21 October 2012 (UTC)[reply]
19th century Britons mostly regarded Africans as an inferior people whose interests can be neglected, much like how supposedly enlightened people regard animals today. They didn't hate the Africans in the same sense that Hitler hated the Jews. Nobody accused the Africans of engaging in a worldwide conspiracy, or killing their God, or taking over their economy.
With regard to Arabs, I should clarify. Obviously Arabic societies never treated Jews with anything that could be called "equality" today. However, I stand by my point that the history of Jews in Arabic societies prior to the 20th century, unlike the history of Jews in Europe, was not defined by hatred and persecution. Your Saudi example is unfair because Wahhabism is essentially a nutjob movement that was considered the fringe of the fringe, both today and in 1937. The Banu Qurayza massacre was the result of a military conflict. Saying that Muhammad killed the men and enslaved the women because of antisemitism is like saying that Agamemnon killed the Trojan men and enslaved the Trojan women because he didn't like Anatolian gods. --140.180.242.9 (talk) 05:32, 21 October 2012 (UTC)[reply]

This has been discussed in books on the subject. Historic evidence is that antisemitism flowers without this type of logic. You'll find it endemic in societies where Jewish are a sizable minority and where Jews are a tiny minority or barely existent. You'll find it where Jews are comparatively rich and poor and neither in particular, where Jews are influential or not influential, visible or invisible, where there are political factors and where there are none, where there are religious factors and where there are none.

A particularly good analysis of this can be found in Why the Jews?, for one example. TheAmazon page for the book includes an excerpt from Chapter one, from which here's a brief quote:

How are the universality, depth, and permanence of antisemitism to be explained? Why such hatred and fear of a people who never constituted more than a small minority among those who most hated and feared them? Why, nearly always and nearly everywhere, the Jews? Many answers have been offered by scholars. These include, most commonly, economic factors, the need for scapegoats, ethnic hatred, xenophobia, resentment of Jewish affluence and professional success, and religious bigotry. But ultimately these answers do not explain antisemitism; they only explain what factors have exacerbated it and caused it to erupt in a given circumstance.

Hope that helps, --Dweller (talk) 04:31, 21 October 2012 (UTC)[reply]

Wow, great quote! --Jethro B 05:28, 21 October 2012 (UTC)[reply]
An IP editor above said "It's hard to hate "the people o'er yonder" with a passion if those people are 5000 km away and have little impact on your life." Well, back in the 1960 Australians (and, I suspect, Americans) became convinced that those gooks, slopes and chinks were worth making our enemies in Vietnam. Hardly anybody here had even met a Vietnamese person, but for several years government had support in sending our troops to kill Vietnamese people. Yes, I know it was complicated, but proximity certainly isn't required to establish "hate". HiLo48 (talk) 05:22, 21 October 2012 (UTC)[reply]
The key phrase is "have little impact on your life". The Americans didn't believe that communism had little impact on their lives; in fact, they believed it was the greatest and most imminent existential threat they've ever faced. The 5000 km thing doesn't apply when your opponent can launch weapons into Earth orbit and have them reenter at any point on the globe. --140.180.242.9 (talk) 06:07, 21 October 2012 (UTC)[reply]
Needless to say, the average Jew living in Germany in the 1930's was utterly incapable of launching weapons into the orbit and attacking Germany. That, however, didn't prevent Hitler from his famous "stab in the back" conspiracy theory. --Jethro B06:20, 21 October 2012 (UTC)[reply]
The Vietnamese couldn't "launch weapons into Earth orbit and have them reenter at any point on the globe", but they were the ones who were vilified. HiLo48 (talk) 07:01, 21 October 2012 (UTC)[reply]
"I'm sure we all agree that we ought to love one another, and I know there are people in the world that do not love their fellow human beings... and I hate people like that." - Tom Lehrer Blueboar (talk) 19:20, 21 October 2012 (UTC)[reply]
And in the same song, as I recall: "Oh, the Catholics hate the Protestants / And the Protestants hate the Catholics / And the Hindus hate the Moslems / And everybody hates the Jews. ←Baseball Bugs What's up, Doc?carrots02:24, 22 October 2012 (UTC)[reply]

Pistols in USA Movies

Why are these often held horizontally? Kittybrewster 22:30, 20 October 2012 (UTC)[reply]

Because some adolescent gangsta thought it looked cool, and/or because it's less likely to obscure an actor's face. Certainly not to improve shooting accuracy. —Tamfang (talk) 22:46, 20 October 2012 (UTC)[reply]
We have an article about this: side grip. Looie496 (talk) 23:46, 20 October 2012 (UTC)[reply]
There was a joke about it in a Simpsons episode... AnonMoos (talk) 06:09, 21 October 2012 (UTC)[reply]
There was also a skit about it (SNL ?), where somebody had to show them how to fire a gun properly, so they could start hitting each other instead of random bystanders. Between not aiming their guns properly and baggy pants that make running away difficult, they seem to have a death wish. StuRat (talk) 17:45, 21 October 2012 (UTC)[reply]
Mythbusters did an episode about various pistol grips shown in movies... and came to the conclusion that the "gangsta" sideways grip was veryinaccurate (although more accurate than the "shoot from the waist" style favored by movie "gangsters" of an earlier era). Blueboar(talk) 19:02, 21 October 2012 (UTC)[reply]
Wild speculation: in a drive by shooting, if you only want to lower your window a little bit (because it's tinted or bulletproof, for example) then you might want to turn a gun that way to get it out the window, especially if it has a banana clip or something. Wnt (talk) 19:08, 21 October 2012 (UTC)[reply]

October 21

religious parties with spiritual leader

Shas party of Israel has a Sephardi Rabbi Ovadia Yosef as their spiritual leader. Hezbollah had a spiritual leader, late ayatollah Mohammad Fadhallah. What other religious parties have religious scholars as their spiritual leader? — Preceding unsigned comment added by 70.29.34.72 (talk) 02:01, 21 October 2012 (UTC)[reply]

Iran has an ayatollah as their supreme leader, see Ayatollah Khamenei. --Jethro B 02:28, 21 October 2012 (UTC)[reply]
The mighty Christian Heritage Party of Canada has a Baptist deacon, Jim Hnatiuk, as its current leader. --NellieBly (talk) 23:27, 22 October 2012 (UTC)[reply]

Ian Paisley founded and led (politically, not religiously) the Democratic Unionist Party between approximately 1972-2009. He was an ordained minister in the Presbyterian Church; and minister and moderator of the Free Presbyterian Church of Ulster. 213.120.90.59 (talk) 12:15, 24 October 2012 (UTC)[reply]

Arlington National Cemetery

Hi, my grandfather (1845 – 1940) fought in the American Civil War and I would love to have him buried in the Arlington Cemetery. What are the procedures and is it possible? Thank you indeed. Iowafromiowa (talk) 13:16, 21 October 2012 (UTC)[reply]

See: Arlington Cemetery#Burial criteria. Following the various links to sources will give you further information. Blueboar (talk) 13:26, 21 October 2012 (UTC)[reply]

News coverage of Pussy Riot on American TV

OK, it's been over 15 years since I lived in the U.S., and maybe things have changed since then, but I can't imagine a network news anchor like Diane Sawyer using the word "pussy" on air. So how is Pussy Riot referred to on TV news in America? Or is the evening news there still as parochial as it was back in the '90s and nothing that happens outside the U.S. is mentioned at all? Angr (talk) 17:37, 21 October 2012 (UTC)[reply]

The fact that it also has a non-obscene meaning of "cat" makes it OK. If the band was "Cunt Riot", they probably wouldn't say it. StuRat (talk) 17:43, 21 October 2012 (UTC)[reply]
Freed Pussy Riot member: Putin influenced sentence. Bus stop (talk) 17:55, 21 October 2012 (UTC)[reply]
  • The James Bond movie Goldfinger had a character named Pussy Galore all the way back in the 60s -- if that goes, anything goes. Looie496 (talk) 18:34, 21 October 2012 (UTC)[reply]
    • Although that wasn't on American TV, at least initially. I wonder when was the first time it aired on American TV, and was anything done to distort the name Pussy? Also, I remember that James Bond himself pronounced it /'pusi/ instead of /ˈpʊsi/. I've always wondered whether that was just Sean Connery's accent or whether it was a way around the censors. Anyone know? Duoduoduo (talk) 20:53, 21 October 2012 (UTC)[reply]
While I don't think there would be a problem with an anchor saying the name of the band on air (if necessary, they would just "bleep" over the potentially offensive word)... I am not sure if the issue has come up. It is true that the Pussy Riot story has not had much coverage in the US (and perhaps none on the major network news). I don't think it is due to parochialism... other international news stories have been covered extensively. Blueboar (talk) 18:53, 21 October 2012 (UTC)[reply]
I think wikt:pussy already is a euphemism, sort of like wikt:boys (the latter term seems to me to have only recently become popular). Wnt (talk) 19:04, 21 October 2012 (UTC)[reply]
I have a dim memory that a blue movie was advertised in my local newspaper (Champaign, Illinois) as P.... Talk, in 197x. —Tamfang (talk) 22:23, 21 October 2012 (UTC)[reply]
If I remember correctly, when Goldfinger was first aired on US broadcast TV, the lady in question's name was overdubbed as "Miss Galore". 69.62.243.48 (talk) 22:26, 21 October 2012 (UTC)[reply]

Not all that long ago, I Love Little Pussy was considered an unexceptionable nursery rhyme... AnonMoos (talk) 13:05, 22 October 2012 (UTC)[reply]

  • I've heard plenty of discussion of Pussy Riot on NPR. They just say "Pussy Riot", with no bleeps or censoring. Some of this content may have been from PRI, but plenty originates from US agencies and reporters. Now, I suppose TV networks may have additional, self-imposed restrictions that the follow, but as far as I know, radio and TV broadcasts are bound by the same FCC laws, meaning that TV anchors could say it too. Lastly, it is humorous to hear the staid and solemn NPR anchors say "Pussy Riot", which I imagine is part of why the group picked the name :) SemanticMantis (talk) 15:25, 22 October 2012 (UTC)[reply]

How can it be that this thread has lasted so long without mention of Mrs Slocombe's pussy? --Dweller (talk) 09:14, 23 October 2012 (UTC)[reply]

Language question: Does the "pussy" pun work in Russian in the same way as in English? HiLo48 (talk) 09:18, 23 October 2012 (UTC)[reply]
Not that I'm aware of. But the lexicon of rude Russian words is vast, and I don't pretend to know anything of its intricacies. Nothing would surprise me. -- Jack of Oz [Talk] 10:52, 23 October 2012 (UTC)[reply]

Multilateral battles

Nearly all big battles have exactly two sides to them, even if multiple countries or groups are represented by each side.

Are there any examples of battles that took place with three or more mutually antagonistic opponents? So instead of side A vs. B, there was side A vs. B vs. C, all of which were equally interested in killing one another (A vs. B, A vs. C, B vs. C)?

I'm suspecting probably not, just because the logistics of such a thing would be difficult and generally would favor a third side waiting to let the first and second sides thin each other out, but the idea of it intrigued me so I thought I'd ask. --Mr.98 (talk) 18:41, 21 October 2012 (UTC)[reply]

I'd say communist chian under Mao, when the Communists fought Japan and the Nationalists, the Nationalists fought the Japanese and the Communists, and the Japanese foughtboth the Communists and Nationalists (since they're Chinese). Or perhaps the Boxer Rebellion to a certain extent, depending on how you view it. --Jethro B 19:08, 21 October 2012 (UTC)[reply]
The Continuation War is a very informative article to read ("There were even several cases of Jewish officers of Finland's army awarded the German Iron Cross, which they declined"), but I suppose it doesn't actually count. The Spanish Civil War, surely. Wnt (talk) 19:17, 21 October 2012 (UTC)[reply]
What were the three or more sides in the Spanish Civil War? I thought it was just two coalitions against each other. For example, our article in paragraph three refers to "both sides in the war". Duoduoduo (talk) 20:45, 21 October 2012 (UTC)[reply]
@ Duoduoduo : see Barcelona May Days . Later, during the Huesca battle, anarchists troops refused to attack, and the franquists kept Huesca...T.y. Arapaima (talk) 09:07, 22 October 2012 (UTC)[reply]
Stalin considered purging anarchists and Trotskyists more important than actually winning the war against Franco, and the NKVD was present in Republican Spain towards the end... AnonMoos (talk) 12:57, 22 October 2012 (UTC)[reply]
I don't know a real battle yet, but the final shootout in Extreme Prejudice is a movie example. --KnightMove (talk) 19:49, 21 October 2012 (UTC)[reply]
During the Crusader invasions of Egypt in the 1160s-70s, the Fatimids, Ayyubids and crusaders were sometimes fighting a three-way war. Adam Bishop (talk) 21:51, 21 October 2012 (UTC)[reply]
I think your instinct is right, that they normally form two sides first, until one is defeated, then the remaining sides may fight amongst themselves. For example, in Syria, the rebels are composed of both secular and Islamic fundamentalist forces. It's quite predictable that, after Assad goes, they will then fight each other for control (with perhaps the Kurds joining in, too). So, in a situation of anarchy, then you can get everyone fighting everyone else. The Lebanese Civil War is a previous example of this. StuRat (talk) 21:58, 21 October 2012 (UTC)[reply]
Do you mean "battle" as in a one-day actual physical battle, or are you using the word as a metaphor for "war"? Because the first seems difficult logistically. --NellieBly (talk) 23:34, 21 October 2012 (UTC)[reply]
I mean an actual battle, not just a war. There are plenty of instances of multi-party wars. --Mr.98 (talk) 23:56, 21 October 2012 (UTC)[reply]
To add some magic to this, {{Infobox military conflict}} has a "combatant3" parameter to indicate a third, distinct combatant in a battle. Searching through the first few hundred examples of infoboxes that have this parameter instantiated, I've only found one real contender so far. The others are either fictional, between species of wildlife, or involved factions that just didn't want their own people killed. Perhaps more exhaustive analysis of this search would yield more results. Orange Suede Sofa (talk) 00:27, 22 October 2012 (UTC)[reply]
All of those except Narnia are interesting links — thanks! --Mr.98 (talk) 17:17, 22 October 2012 (UTC)[reply]
I seem to remember, in WW1, possibly in Tanzania, the British were fighting a battle against the Germans, and then the locals joined in and fought both sides, during which the British and Germans had a temporary armistice to fight off the locals. After doing so, they resumed combat. I can't find any references for this, but I read it somewhere. KägeTorä - (影虎) (TALK) 07:04, 22 October 2012 (UTC)[reply]
You won't find the word "Tanzania" in any such reference. I'm sure you're aware of this, but just in case. -- Jack of Oz [Talk] 20:23, 22 October 2012 (UTC)[reply]
Yes, of course. It was German East Africa at the time. Sorry, about that. KägeTorä - (影虎) (TALK) 20:33, 22 October 2012 (UTC)[reply]
See the brilliant second sentence of our article on Yugoslav Wars:
he wars were complex: characterized by bitter ethnic conflicts among the peoples of the former Yugoslavia, mostly between Serbs (and to a lesser extent, Montenegrins) on the one side and Croats and Bosniaks (and to a lesser degree, Slovenes) on the other; but also between Bosniaks and Croats in Bosnia (in addition to a separate conflict fought between rival Bosniak factions in Bosnia).
Summary: what a sad mess. --Dweller (talk) 09:09, 23 October 2012 (UTC)[reply]
Yes, "he wars" always are. Fighting women, on the other hand, are fun to watch. :-) StuRat (talk) 14:02, 25 October 2012 (UTC) [reply]
... in oil ... —Tamfang (talk) 20:08, 25 October 2012 (UTC) [reply]

Problems concerning scientific prodigies

Sometimes you read media reports about ingenious wonder children who move to major cities and start a university career in math or physics at the age of 10 or so. However, I assume that such kids and juveniles often face a bunch of social and legal problems about which I'd like to know more - are there reports about such problems? --KnightMove (talk) 19:29, 21 October 2012 (UTC)[reply]

The potential problems are not limited to "scientific" prodigies... the same issue can affect other prodigies (child actors, children with musical talent, etc.) Prodigies who are pushed into adult situations too soon often have sociological and emotional difficulties. Being gifted can be lonely... a prodigy may be intellectually ready for great things but may not yet be emotionally ready for them. Blueboar (talk) 19:48, 21 October 2012 (UTC)[reply]
Ok... what about legal problems? A child can't rent an apartment and live on his own. Or, at what age is this possible in the USA? --KnightMove (talk) 20:03, 21 October 2012 (UTC)[reply]
In the case of a child who attends university, the university will act In loco parentis... and assume the legal responsibility for the child. This is what happens when a child attends a boarding school. Blueboar (talk) 21:00, 21 October 2012 (UTC)[reply]
You may find this article an interesting read about the experience of Ruth Lawrence and the effect on her and her family then and now. Her father effectively dedicated himself to accompanying her, which solved the practical problems but not the others. - Karenjc 20:11, 21 October 2012 (UTC)[reply]
Child protection policies make it difficult now for universities to take on the in loco parentis role. A boarding school ought to know what it is doing, and should have a full range of support structures in place. A university is geared to work with young adults (and older adults), not children. Itsmejudith (talk) 21:33, 21 October 2012 (UTC)[reply]
See minor emancipation, KnightMove. μηδείς (talk) 02:04, 22 October 2012 (UTC)[reply]
Yes, but I'm not really sure how that's related. Judith's point is valid: Universities are designed to handle the social development of the average 18-22 year old, not the social development of, say, the average 12 year old. Doogie Howsers may be intellectually ready for advanced physics, but they may not be ready to be socially integrated into, or be at a stage of life similar too, their classmates at a typical University. --Jayron32 05:20, 22 October 2012 (UTC)[reply]
Instances of Dougies I've heard about, the kid has either attended a local university, or he lives with a parent or someone else looking after him. So not in a dorm.--Wehwalt (talk) 01:01, 26 October 2012 (UTC)[reply]

October 22

women priests

In Judaism, Reform Jews and Conservative Jews and Reconstructionist Jews have rabbis who are women. Is Judaism the only religion in the world that have female priests in specific sects? No debate, just simple answer like no or yes. Thanks.--70.31.22.91 (talk) 01:16, 22 October 2012 (UTC)Don Mustafa[reply]

No. See Ordination of women. AndyTheGrump (talk) 01:18, 22 October 2012 (UTC)[reply]
Rabbis are not priests and certainly not priestesses. See http://www.jewfaq.org/rabbi.htm μηδείς (talk) 02:02, 22 October 2012 (UTC)[reply]
To summarise the Christian situation briefly - there's a complete spectrum. The Anglican communion is divided on the subject. The Presiding Bishop of the Anglican Church in the USA is a woman (and also a priest, necessarily). The Church of England is just getting set to create its first women bishops, but has had women priests for some time. The Lutheran Church of Sweden has women bishops too - the Bishop of Stockholm, for example.
The Roman Catholic church and the various Orthodox churches do not have women priests.
Not all Christian churches have priests at all. AlexTiefling (talk) 09:06, 22 October 2012 (UTC)[reply]
And, of course, women played and play important rules in all kinds on non-Christian religions. See e.g. Vestal Virgin and Pythia for classical examples. --Stephan Schulz (talk) 10:30, 22 October 2012 (UTC)[reply]
I found Religious sexism: when faith groups started (and two stopped) ordaining women. Alansplodge (talk) 10:34, 22 October 2012 (UTC)[reply]
Some religions, unlike the Abrahamic religions, explicitly promote the elimination of prejudice. See Baha'i Faith and the unity of humanity. Baha'i has no priests (and considering that rabbis aren't priests, the OP's requirements are probably not strict), but its elected councils are open to both men and women. --140.180.242.9 (talk) 18:34, 22 October 2012 (UTC)[reply]
Not true, 140. The highest elected council, the Universal House of Justice, is definitely not open to women of any age, experience or wisdom. A 21-year-old male is, however, eligible. Bielle (talk) 18:46, 22 October 2012 (UTC)[reply]
In any case - in what sense is Baha'i not 'Abrahamic'? AlexTiefling (talk) 19:28, 22 October 2012 (UTC)[reply]
@Bielle: You're most definitely right. I was somehow under the impression that it wasn't an elected council.
@Alex: It's not Abrahamic in the sense that it doesn't claim to have originated from Abraham and considers numerous religions to be "true", many of which have no relation to Abraham and explicitly claim another founder. In that sense, they're similar to delusional liberals who believe all religions to be true while ignoring the fact that all of them are ludicrous, self-contradictory, and mutually contradictory. --140.180.242.9 (talk) 04:45, 23 October 2012 (UTC)[reply]
Nevertheless, it developed from a form of Islam (Shaykhism/Babism), which is usually enough to count as "Abrahamic", unless there has been radical discarding of historical influences... You can look at The Abolition of Man by C.S. Lewis (a determined Christian who definitely did not think that all religions are "true") for a systematic defense of the idea that almost all religions contain valid moral foundations... AnonMoos (talk) 07:46, 23 October 2012 (UTC)[reply]

@all. While it is true that the House of Justice is limited in it's membership it is insufficient to summarize the Baha'i principles on the equality of the sexes with just that fact. Individually members of the House are not seen as the highest individual office - that would be Continental Counsellors and yes there have been women. Before that institution there were Hands of the Cause and again there were women - indeed the most prominent in the 20th century were women (see Martha Root, and Rúhíyyih Khanum .) Additionally the principles-in-action represented by the clear prominence of women at the national level outstrips all other international organizations and almost every (I don't know of one, but who knows) national organization in freely voting for a higher percentage of women to office holders. As for priests - yes there are none and that goes with being intermediaries in religious rites to be "right" with God - as there are none of the former there are none of the latter. And in particular to being Abrahamic - it is true the religion acknowledges non-Abrahamic religions this does not invalidate it as Abrahamic. Obviously it acknowledges Abraham and claims to have descend ended from two of his wives, and there are reviews that make many parallels in teachings between the religions as is detailed in the Abrahamic religions article. As for not taking seriously that there are problems of interpretation - well there are two parts to that -taking it seriously, and wrestling with those differences. On the first part let me offer, in another connection, that my father in law was a conscioustous objector to WWII and that during his time in the forest of Oregon in a work camp he became a Baha'i and then had to go through boot camp and served as a non-combatent in war as a medic. I can't say all Baha'is take all things seriously, but clear some do and it matters. I've not seen a systematic study of how Baha'is wrestle with the contradictory approaches taken within and among all the religions but a common thought and one you can find in a religion-by-religion analysis is that if you look at the original scriptures you can find bridges of understanding that resolve differences others later made into particular understandings and lost the ability to think again what they meant. This is why - in the Abrahamic lineage - you have religion after religion, rather than a succession of one religion (similarly in the system of eastern faiths with Hinduism to Buddhism which could have been a succession of one religion.) But that would not be possible today with the administrations of each religion. Smkolins (talk) 12:03, 26 October 2012 (UTC)[reply]

Who is the person Obama is bowing to?

Who's Obama bowing to in this picture? Thanks. μηδείς (talk) 01:59, 22 October 2012 (UTC)[reply]

Viktor Yushchenko ---Sluzzelin talk 02:07, 22 October 2012 (UTC)[reply]
Wow, he looks great since the poisoning. μηδείς (talk) 03:05, 22 October 2012 (UTC)[reply]
Yuschenko's disfigurement was primarily chloracne and bloating, both of which fade over time. You can see the fading over the years — 2006 (bloating and chloracne) 2007 (just chloracne) 2009/2009 (much faded chloracne). (Compare before/after in 2004). I suspect some of his scarring may be permanent but it's gotten a lot better. --Mr.98 (talk) 16:54, 22 October 2012 (UTC)[reply]
It's not unambigiously clear that Obama is in fact bowing. It's quite possible that he is meley looking at and commenting on Yushchenko's shoes. Roger (talk) 15:32, 22 October 2012 (UTC)[reply]
Maybe he's both bowing and shaking hands at the same time, as he did much to the amusement of the incumbent Mikado?--Wehwalt (talk) 15:41, 22 October 2012 (UTC)[reply]
Yes... bowing and shaking hands at the same time. Obama is known for doing this when greeting other heads of state (especially when he is the visitor to their country). Some criticize him for this practice (seeing the bow as a sign of submission, which should not be done between equals). Others don't have a problem with it (seeing the bow as a sign of respect, perfectly acceptable between equals). Blueboar (talk) 15:45, 22 October 2012 (UTC)[reply]
Obviously he's about to pull the old "your shoelace is untied" gag. Clarityfiend (talk) 21:07, 22 October 2012 (UTC)[reply]

Amazons

According to our article on the Amazons:

"According to ancient sources, (Plutarch Theseus,[74] Pausanias), Amazon tombs could be found frequently throughout what was once known as the ancient Greek world. Some are found in Megara, Athens, Chaeronea, Chalcis, Thessaly at Skotousa, in Cynoscephalae and statues of Amazons are all over Greece. At both Chalcis and Athens Plutarch tells us that there was an Amazoneum or shrine of Amazons that implied the presence of both tombs and cult. On the day before the Thesea at Athens there were annual sacrifices to the Amazons."

As far as I know, the Classical Greeks thought the Amazons were humans, not goddesses. Not only that, they were enemies of Greece and never had a single friendly encounter. So why were the Amazons worshipped? --140.180.242.9 (talk) 06:58, 22 October 2012 (UTC)[reply]

Remember that to the ancient Greeks, the Gods were just normal humans who happened to be immortal. And there were those in-between, like Hercules. So, they could think of the Amazons, at various times, as legendary humans or as goddesses. StuRat (talk) 07:08, 22 October 2012 (UTC)[reply]
140.180.242.9 -- In ancient Greek culture, local ancestors and culture heroes / "demigods" were honored, and sometimes sacrificed to, without being confused with gods as such... AnonMoos (talk) 12:51, 22 October 2012 (UTC)[reply]
But the amazons were neither the ancestors of the Greeks nor demigods. The only full mortals I know of who were sacrificed to were legendary founders (and Greeks) like Perseus. --140.180.242.9 (talk) 18:23, 22 October 2012 (UTC)[reply]
They weren't particularly divinities, but they were semi-mythical figures with striking personal characteristics, and if some of them developed local associations, they might be considered "heroes" in a sense... AnonMoos (talk) 00:50, 23 October 2012 (UTC)[reply]

Whitecollar : 2 cultural questions please

Hello L.O. (Learned Ones) ! The TV serie Whitecollar is gaining audience in France, thanks to its aesthetic qualities and winding intrigues, & the IIIrd season, 14° episod (Pulling Strings) has been welcomed : bound to be, it features Matt. Bomer as a beloved character of ours : Arsène Lupin (or at least as his great-grand-son) , and a good value for us here, a Stradivarius violin. Could you give me some cultural references tips about :

1/ the doll retrieved after 30 years : great round sullen eyes, concave profile with sunken little nose, stiff bleached hair, limp body, makes you think of an abused off-spring from alcoolic parents... I don’t remember having seen such a freak in little girls’s hands during the ’80. Was this doll à la mode in the US then , did it have a specific name ?

2/ I didn’t see clearly what New York concert hall Neal and Sara were attending : a wide amphitheatrum with big low armchairs, with mustard-colored (I think) walls and furniture. Can you tell me what theater it was ? Thank you beforehand for your answers, T.y Arapaima (talk) 08:50, 22 October 2012 (UTC)[reply]

1) Cabbage Patch Kids: [20] ? They were quite the fad in the 1980s. StuRat (talk) 09:11, 22 October 2012 (UTC)[reply]
Looking at screenshots (ETA: here) it doesn't seem to be a cabbage patch kid. --NellieBlyMobile (talk) 16:02, 22 October 2012 (UTC)[reply]
Blimey! It bears a slight resemblance to a Troll doll, but I can only find pictures of smiling ones.[21] Alansplodge (talk) 16:21, 22 October 2012 (UTC)[reply]
It rings no bells with me. This wouldn't be the first fictional fad. —Tamfang (talk) 18:30, 22 October 2012 (UTC)[reply]
According to [22] it's a Little Miss No Name, a Hasbro line from 1965. (What, do we not have an article?) Marnanel (talk) 20:36, 22 October 2012 (UTC)[reply]
Could this be the concert hall? That's the Metropolitan Opera House at the Lincoln Center for the Performing Arts. (ETA Or the Avery Fisher Hall?) --NellieBlyMobile (talk) 16:02, 22 October 2012 (UTC)[reply]

Automatic Pistols

All the (semi)automatic pistols I have seen require manual loading of first cartridge, i.e. after inserting a loaded magazine you pull and release the bolt slide so that first cartridge gets into breech, then you fire and need not pull bolt next time, hence "automatic". My question is that are there any models that do not require the loading of first bullet ? If so then how does it manage to get first cartridge from magazine into breech ? 124.253.95.164 (talk) 09:56, 22 October 2012 (UTC)[reply]

Answer to first question - No. Answer to second question - Moot (see answer to first question). All automatic weapons invented so far require a deliberate action by the operator to move the first round from the feed mechanism into the chamber. This applies equally to hanguns, rifles, machineguns all the way up to large automatic cannon. Roger (talk) 13:15, 22 October 2012 (UTC)[reply]
The question reminds me of a funny blunder in The Killing (Kubrick's heist movie, 1956): Elisha Cook picks up an empty pistol, works the slide and then inserts a magazine, leaving the chamber empty.
A quibble worth noting imho: With most pistols, when you fire the last round of a magazine, the slide stays open. You then replace the magazine and release a catch, and the slide closes, loading the first round with less work on your part. —Tamfang (talk) 18:28, 22 October 2012 (UTC)[reply]
I know that, of course. I know my bit about guns, especially autos, to know that " All automatic weapons invented so far require a deliberate action by the operator to move the first round from the feed mechanism into the chamber. This applies equally to hanguns, rifles, machineguns all the way up to large automatic cannon." , as told above. That is the reason I am asking this question ! A vendor told me about a .25 that works (he claims) thus ! and we are not talking about the bolt held back by previously exhausted magazine as Tamfang has told... — Preceding unsigned comment added by 124.253.60.148 (talk) 05:51, 24 October 2012 (UTC)[reply]
Not always a good thing, because it tells everyone that you've fired your last shot. Alansplodge (talk) 12:03, 23 October 2012 (UTC)[reply]

Last living veteran of the Second Anglo Afghan War?

Last living veteran of the Second Anglo-Afghan War? Thank you. Iowafromiowa (talk) 10:19, 22 October 2012 (UTC)[reply]

The best I can do is; The Second Anglo-Afghan War 1878-1880 which links to Yahoo Groups: The Second Anglo-Afghan War and says that "Some topics already mentioned include... longest living Afghan war veterans." You have to join the group to read the messages. Alansplodge (talk) 13:13, 22 October 2012 (UTC)[reply]
I should have known; WHAAOE. See List_of_last_survivors_of_historical_events#1850–1900; "Hugh Theodore Pinhey - February 6, 1953 (aged 96) - Last veteran of the Second Anglo-Afghan War." Alansplodge (talk) 13:17, 22 October 2012 (UTC)[reply]
That's an amazing list. It even names the "last individual who claimed to have spoken to Virgin Mary." No more claimants then?--Shantavira|feed me 14:57, 22 October 2012 (UTC)[reply]
That's surprising to me. Do Catholics no longer ask her to do something or other now and at the hour of their deaths amen?--Wehwalt (talk) 15:43, 22 October 2012 (UTC)[reply]
Not just Catholics... although, apparently she does not reply as often as she used to do. Perhaps the entry should be changed to "last individual to claim that the Virgin Mary spoke to him/her" Blueboar (talk) 15:54, 22 October 2012 (UTC)[reply]
That's not right either, since I'm sure there are many mentally ill people who have imagined that she's spoken to them. Perhaps "last individual believed by the Roman Catholic Church to have spoken with the Virgin Mary"? This also avoids the implication that Wikipedia is judging the merits of any claims. --NellieBlyMobile (talk) 17:13, 22 October 2012 (UTC)[reply]
Even more amazing is that whenever Joe and Rose Kennedy had a child, it's regarded as a historical event. -- Jack of Oz [Talk] 18:51, 22 October 2012 (UTC)[reply]
That 1917 Fatima claim is wrong, anyway. According to Marian apparition, the Catholic Church has accepted the claims of later apparitions and conversations with the Virgin Mary: in Beauraing 1932/33, Banneux 1933, and Akita 1973. -- Jack of Oz [Talk] 18:58, 22 October 2012 (UTC)[reply]
Oh, I see what's going on. It's saying that Lúcia Santos was the last survivor of a conversation with Mary (1917), because she lived till as late as 2005. I'm still not sure that's a correct claim. Sister Agnes Sasagawa, who conversed with Mary at Akita in 1973, has not to my knowledge died, so she has outlived Lúcia Santos. -- Jack of Oz [Talk] 20:17, 22 October 2012 (UTC)[reply]
I guess they must meant that she was the last of the three children to see her at (edit) Fatima. I've edited the list to be clearer. --NellieBly (talk) 23:14, 22 October 2012 (UTC)[reply]
Thanks, Nellie. -- Jack of Oz [Talk] 23:33, 22 October 2012 (UTC)[reply]

More about our man Hugh; a long letter that he wrote to his mother describing the Siege of Kandahar can be seen here. "Hugh Pinhey was an Assistant Superintendent in the telegraphy department, but for the duration of the siege was given the rank of lieutenant and assigned to the 4th Bombay Rifle Corps". Alansplodge (talk) 16:45, 22 October 2012 (UTC)[reply]

A serious flaw in any "Last living veteran of the <"civilized country"> versus <"savage tribe"> wars" list, is that reliable information exists for the veterans of only one side. In this particular case - who was the last Afghan veteran? Roger (talk) 14:23, 24 October 2012 (UTC)[reply]
Agreed, perhaps "last British veteran" would be more accurate. Alansplodge (talk) 22:02, 24 October 2012 (UTC)[reply]

Oath of Office of the President of the United States

Since this is the Oath of Office of the President of the United States [the district], an employee of the congress under the constitution of the US of 1789 and the chief administrator for the united States of America. What information is available regarding the "Oath of office for the President of the United States of America" under the Articles of Confederation 1777(?) as found in the present/current United States Code????

Thank you, robert: carr — Preceding unsigned comment added by 208.58.114.72 (talk) 17:21, 22 October 2012 (UTC)[reply]

Under the Articles of Confederation there was no such office as President of the United States. The nearest thing was the President of the Continental Congress, but that was a far less powerful office, having a one year term and very limited authority. The Articles did not specify an oath for that office. Looie496 (talk) 18:28, 22 October 2012 (UTC)[reply]
Indeed. The President of the Congress was basically the Speaker; he was in charge of maintaining order at the meetings of Congress and almost nothing else. There was no single Chief Executive of the U.S. under the Articles of Confederation (also no unified currency, barely any military, no federal judiciary, etc. AFAIK, all they did was send ambassadors abroad to secure loans and trade agreements and not much else). --Jayron32 22:41, 22 October 2012 (UTC)[reply]
Being a member of the Continental Congress was a thankless and largely powerless office, and being the President of the Continental Congress also had its drawbacks. It is very easy to envision someone being elected, but not wanting to take on the office (Poor health, etc). Thus there was a need for some action to be taken at some point in time whereby the electee actually accepted and assumed the office. Journals of the Continental Congress were preserved. So what words were uttered, what actions were taken, and how was the action logged in the journal of the Continental Congress by the Secretary thereof? Edison (talk) 02:53, 23 October 2012 (UTC)[reply]
A volume of the Journal says that the secretary and clerks had to take "an oath of fidelity to the United States and an oath of fidelity for the execution of their respective trusts." It is likely that the president had to take similar oaths. Edison (talk) 03:00, 23 October 2012 (UTC)[reply]
Possibly not. As a delegate from his state, the president was already an elected official and a member of Congress, so a new oath may have been superfluous. The secretaries and clerks were new hires, however. —Kevin Myers 00:46, 24 October 2012 (UTC)[reply]

Kippah type fabric Reconstructionist

In the Kippah article, you mentioned the type of kippahs for different sects of jewish and the fabric. What about Reconstructionists? Which fabric do they use for kippahs? — Preceding unsigned comment added by 70.29.34.207 (talk) 17:29, 22 October 2012 (UTC)[reply]

You'll get a better chance of a good answer by posting this question at Talk:Kippah. You might want to crosspost a link to that question at Talk:Reconstructionist Judaism, but please don't cross post by putting up the question twice. --Dweller (talk) 09:04, 23 October 2012 (UTC)[reply]

sects protestantism

What are the main sects of Protestantism practiced in the world today in USA, Sweden, Norway, Iceland, Germany, Netherlands, Denmark, South Africa, Anglophone Caribbean, UK, Australia and New Zealand? — Preceding unsigned comment added by 70.29.34.207 (talk) 18:03, 22 October 2012 (UTC)[reply]

By sect, do you mean branch/movement or are you referring to denominations? Norway, Sweden, Finland, Denmark and Iceland each have a very large state/national church, which is by far the largest denomination in their resprective countries. All of these are Lutheran, but few of the members actually attend church services. Germany likewise has mainly Lutherans, although these are divided into many individual (though often cooperating) churches. The Protestant Church in the Netherlands is the largest protestant denomination in the Netherlands. This church and most other Dutch protestant churches originated from the Dutch Reformed Church, and are mostly rooted in reformed (calvinist) theology. I'll let others address the other countries. - Lindert (talk) 18:17, 22 October 2012 (UTC)[reply]
Your desired data for the US are available at Christianity in the United States, with a comprehensive list of like articles at Christianity by country. — Lomn 18:49, 22 October 2012 (UTC)[reply]

Maximilian Kolbe

Hello,

I am curious about the article Maximilian Kolbe. What happened that it was viewed 56000 times in one day and was vandalized almost consequently? Just curious. Regards.--Tomcat (7) 18:21, 22 October 2012 (UTC)[reply]

That was October 10. The article appeared on the main page in the "On this day" area, marking the 30th anniversary of his elevation to sainthood. Looie496 (talk) 18:33, 22 October 2012 (UTC)[reply]

how are saints made?

Yesterday on the radio I heard several times that someone was "made" a saint. Am I right in thinking that canonization (at least in Rome) doesn't make a saint but rather recognizes that someone is a saint? —Tamfang (talk) 18:36, 22 October 2012 (UTC)[reply]

You are correct, but the point is a technical one, and it's almost certain the reporter got it wrong. AlexTiefling (talk) 18:39, 22 October 2012 (UTC)[reply]
Saints are not made, they are born. Now seriously, just check canonization for more details, you just get recognized to be a saint. 80.39.52.222 (talk) 20:58, 22 October 2012 (UTC)[reply]
If you can "install" a pope without the aid of a rivet gun, it only stands to reason that you can "make" a saint without a factory. :-) StuRat (talk) 22:37, 22 October 2012 (UTC) [reply]
I just figured the radio reporter had confused saints with the British system of titles of nobility, or possibly the mafia. ←Baseball Bugs What's up, Doc? carrots22:41, 22 October 2012 (UTC)[reply]
Speak softly, Bugs. They won't want to hear that. --NellieBly (talk) 23:02, 22 October 2012 (UTC)[reply]
"They" being Saints, Peers of the Realm or mafiosi? Surely a conjunction rarely found in one sentence. Alansplodge (talk) 00:09, 23 October 2012 (UTC)[reply]
Maybe "created" is the more commonly used term in titles of nobility. "He was created the Earl of Oil", or whatever. In the underworld a "made guy" has committed his first murder for the family business. It's curious to think of a saint as being a "made guy". ←Baseball Bugs What's up, Doc? carrots05:12, 23 October 2012 (UTC)[reply]
I'm suddenly reminded of The Bishop. Dismas|(talk) 04:57, 23 October 2012 (UTC)[reply]

Christian traditions...

Where and how can I find information about specific methods or beliefs of different Christian intepretative traditions? Often, people use the general term "in some traditions" without nailing on which tradition they may be referring to, thereby drawing a blank. Even the Wikipedia article Sacred Tradition talks about how this tradition is passed down orally, in the lives of Christians, thereby giving the impression that the only way to understand Christianity fully is to meet a Christian in person and assume that Christian knows/understand scripture as well as tradition passed down by his/her ancestors or learned from the church. Furthermore, the Christian would presumably use the conversation as an opportunity to proselytize the non-Christian, making the Christian life and faith attractive in every way to the point of bringing in more converts. How do Christians know what traditions to reject and what traditions to accept and what traditions are ought to be modified, or do they just accept whatever is part of their church without question? This may hint at the pejorative connotation of the word indoctrination. 140.254.227.51 (talk) 18:58, 22 October 2012 (UTC)[reply]

Well, would you take it from me that I'm a practising member of the Church of England, that I have no interest in converting you, and that I know a reasonable amount about my own tradition and others?
It's important to understand the distinction between a tradition and a denomination. For example, the high church tradition is found in Anglicanism and in Lutheranism. Those denominations are separate (although some of their constituent churches are linked by the Porvoo Agreement), and each contains many congregations (a sizeable majority) who do not represent that tradition - but also many who do.
And it's not just about scripture and the interpretation of scripture. Indeed, when the Roman Catholic church refers to its own three teaching traditions, only one of those is scripture. The others are reason and tradition itself. The idea that Christian life and worship is all about the Bible is itself the product of one particular tradition - which is often called fundamentalist, but might more properly be called evangelical.
So if you have specific questions, please ask. AlexTiefling (talk) 19:18, 22 October 2012 (UTC)[reply]
The fundamentalist tradition would fit under the evangelical tradition, which would be a subset of Protestantism, as many evangelicals use the Protestant Bible (66 books, no Apocrypha). With so many Christian traditions around, it is easy for one particular denomination to think that that denomination is the one true church or one true faith and thus is superior than the other churches or denominations. Some Christians treat Christianity as a big-marketing strategy to help emotionally/socially isolated people or social outcasts or impoverished people in order to convert these people to Christianity. Also, given that some churches explicitly describe on their church websites that "salvation is free", they are hinting that anybody can become a Christian and receive God's "free gift", even though they may be forgetting that they are expecting that the Christians ought to behave what the church wants them to behave. 140.254.227.51 (talk) 20:00, 22 October 2012 (UTC)[reply]
(EC) :"Tradition" is a word that is often used nowadays to mean Christian denomination (our article uses the word twice in the first paragraph). It's a move away from "this is our faith and other Christians are wrong" towards "this is our faith, others have different traditions". So I suspect "in some traditions" means "in some denominations". Use of the word "church" in this meaning can be confusing, as it can refer to a) all Christians, regarded as "Christ's body on earth" Saint Paul's First Letter to the Corinthians, Chapter 12: verses 12-27 b) a particular denomination c) a subset within that denomination as the Church of England and the Church in Wales within Anglicanism d) a particular local congregation or e) an actual building. "How do Christians know what traditions to reject and what traditions to accept"? The most obvious answer is that they usually go to church on Sunday (see meaning "e") where those traditions are enacted, and explained during the sermon. Alansplodge (talk) 19:35, 22 October 2012 (UTC)[reply]
So, the only way to see the traditions enacted or come to life is to actually enter a church, or if you are a non-Christian, pretend to be one and enter a church, so that you can witness the sermon, the style, the manner, et cetera, and compare that experience to all Christian churches you may have visited in your local area - which, again, can be quite subjective - in order to track down what a particular church believes in. I suppose one needs to keep one's eye open when walking inside a church to see what others are doing and that person does the same. In Western churches, everyone does the same thing at the same time, which to Eastern churches would seem very regimented. In Eastern churches, everyone does different things at the same time and stand up to worship, which to Western churches would seem very disorganized. The non-Christian visitor would somehow have to find a way to keep that in mind and make an attempt to blend in with the crowd. Sort of like Waldo. 140.254.227.51 (talk) 19:50, 22 October 2012 (UTC)[reply]
You will not have to pretend to be a Christian at all. Usually there are no restrictions for attending a service/sermon. If anyone asks, just tell them you are interested to learn about their faith. You could also ask if you can speak to the pastor or to an elder sometime after the service to learn more, or you can find the church's website, which will usually contain a statement of faith and more. - Lindert (talk) 20:13, 22 October 2012 (UTC)[reply]
(after ec) Given that you asked this question, you've also done an awful lot to answer it too - mostly by delivering a sermon of your own. The short answer to your original question 'where and how can I find information' is (obviously) right here, because Wikipedia is an encyclopedia. But I made a genuine offer to answer specific questions about my own tradition and those in which I've studied. If you've got questions, ask away. I'll try to back my answers up with links and so on where possible. AlexTiefling (talk) 20:14, 22 October 2012 (UTC)[reply]
OK. There was this organization on campus that offered Bible study sessions every Wednesday. So, I attended a couple of times, and during my time there, it seemed that the group derived its interpretations on annotations made by "Living Stream Ministry". I looked up the ministry on Wikipedia, and noticed that it was an evangelical Christian ministry. Nevertheless, the group displayed on its website that it was "interdenominational", accommodating all denominations, not just "evangelical". One time, the group had these tiny pieces of paper that advertised "The Recovery Version" of the Bible, specifically the New Testament. How trustworthy or representative of Christianity are evangelicals? Do they just represent a non-denominational Protestant tradition? 140.254.227.51 (talk) 20:48, 22 October 2012 (UTC)[reply]
Yeah, that would be a non-denominational evangelical Protestant group. For the maximum amount of amusement, you should tell them you're Roman Catholic. Adam Bishop (talk) 21:01, 22 October 2012 (UTC)[reply]
Just to clarify, most evangelicals are not non-denominational. The answer on whether they represent Christianity well will obviously differ on whom you ask. - Lindert (talk) 21:34, 22 October 2012 (UTC)[reply]
(e/c) Going to an organization that offers Bible study lessons might not really be the best idea. There's a good chance you'll be surrounding yourself with zealots. You'll immediately stand out as a new face/outsider, and might perhaps be interrogated or be seen as a potential convert if you profess disbelief. .
As an agnostic atheist raised in a strongly Catholic society (the Philippines), I've had more than enough experience in being a non-Christian visitor in Christian mass. If you really want to see the different Christian traditions, try going instead to larger congregations, pick a mass held in a cathedral if you can. That way you won't have to defend yourself from overzealous conversion. In such environments, you won't have to pretend to be Christian. You won't even be asked. People don't really take much notice of others in large churches, except during the part where they say "peace be with you" to the people around them. The same is true with the older and larger denominations like the Anglican Church or the Orthodox Church. Just try to follow what everybody else is doing (e.g. stand when they stand). However, it's perfectly alright if you don't sing, chant, eat the bread (in fact, you shouldn't), or make the sign of the cross, you can even elect to stay seated during the time when the participants kneel. That's usually what non-Catholic people here do when taking part in a Catholic mass (usually in cases of mixed-religion families). Nobody will glare at you, they only do that when you fall asleep, lol. It's far less uncomfortable than going to evangelical churches or one of those extremely small protestant churches. Any stranger can enter one without having to be confronted with his beliefs, whereas in evangelical churches, you will likely immediately be noticed.
Admittedly, despite being an atheist, I'm more sympathetic to Catholics, as I was raised one, and most of my friends and relatives are Catholics. I know Catholics have a rather unfair image in American culture of being ultratraditionalist and pious due to their adherence to rituals and the "no sex before marriage"/"no divorce"/"no prophylactics" thing, but it's rather the opposite really. They're far less aggressive in converting people (with 1 billion members, they're pretty secure), are more tolerant of other faiths (at least at present times), largely self-sufficient (i.e. they don't trick your grandma into giving them her life savings), their clergy don't have personal net worths in the billions like Pat Robertson, and they're actually less fundamentalist than the Protestant churches that dot rural America. -- OBSIDIANSOUL 22:25, 22 October 2012 (UTC)[reply]
(outdent) OK - this I can answer.
1) The idea of non-denominational ministries is very much associated with Protestantism, especially in America. The reason is simple - one can't be a non-denominational Catholic, Anglican or Orthodox, because membership of those traditions more or less requires adherence to a corresponding denomination. In Europe, there are major churches representing Protestant traditions such as Lutheranism and Presbyterianism - and in some places, those are the state-backed established churches. It's in America that free-floating Protestantism thrives.
2) Evangelicalism is definitely a movement or tradition,and not a denomination. So it's entirely possible for something to be interdenominational and yet thoroughly evangelical. Of course, they might not appear so completely interdenominational if you mentioned you were Roman Catholic, or Orthodox. The idea of a genuinely interdenominational ministry that insists not only on a particular canon of scripture, but also on a specific translation (obscure, and both commissioned and sold by the ministry itself) seems odd to me. I can generally expect to go into a church of my own denomination or any of the other major ones round here, and find several different translations on hand. I might even hear as many as three translations in the course of a single service. Most of those are just that - translations. No interpretation is provided, except perhaps for some tellingly biased chapter headings and a few notes from the translators.
3) Evangelicalism is huge. It's probably the largest single cross-denominational movement in Christianity. So my observations about Living Stream are in no way indicative of all Evangelicals! Indeed, Evangelicals have contributed substantially to the Bible translations used in my own and other denominations, and to their worship styles. I don't think it's possible to make useful generalisations about the trustworthiness of evangelicals.
4) The claim to be cross-denominational is considerably undermined by the fact that there is a close connection between the Local Churches and Living Stream.
5) So I would suggest that Living Stream are not really cross-denominational, or representative of evangelicals generally. I have no way to judge their trustworthiness, except to say that if you put "Recovery Version" into Google, one of its leading auto-complete suggestions is "Bible cult". Our article about the Local Churches movement has some poorly-referenced claims about the specifically Chinese origins of this movement.
6) Please feel free to ask further. I've got an associate's degree in theology (from an institution which does not grade according to doctrinal soundness), so some technical and historical queries may be easier to answer than others. AlexTiefling (talk) 21:44, 22 October 2012 (UTC)[reply]
I'm thinking this may differ from place to place. My experience with the phrase "non-denominational" is that it's often used to deceive potential congregants into seeing a church as more open and more mainstream than it actually is. And they always are a member of a denomination - they just don't want to admit it right off. --NellieBly (talk) 23:00, 22 October 2012 (UTC)[reply]
I think I'd view a claim of being 'non-denominational' as valid if (a) it applied to a Free church which genuinely was not part of any larger structure, or (b) it applied to a publisher, service provider or other organisation which produced material which was applicable to the practice and worship of several denominations. But I broadly agree with your criticism of the way the term is used. I'd also suggest that it's used to appear less mainstream. I often see the term 'mainline protestant' used in the US in a vaguely negative sense to refer to (eg) Lutherans - the implication being that Protestant groups with more divergent eccelsiology (such as Baptists) or fewer ties to 'corrupt' mainstream society. Some of these organisations are themselves both modern and regressive all at once; the Southern Baptists are the only large denomination to have abandoned the ordination of women to the ministry. Amazingly, not all of their existing women ministers walked out. (See thread further up about this topic.) AlexTiefling (talk) 23:09, 22 October 2012 (UTC)[reply]
The internet resource I refer to is Beliefnet, which will give you the answers for questions about Christianity and other religions too. --TammyMoet (talk) 10:15, 23 October 2012 (UTC)[reply]
Well... I would not say beliefnet.com (or any other website) can give the answers to religious questions, but it certainly gives an answer to such questions. Blueboar (talk) 14:51, 23 October 2012 (UTC)[reply]
Indeed. How much bloodshed could have been avoided if certain groups had not insisted they were the one sole true religion. There are many one sole true religions. -- Jack of Oz [Talk] 20:37, 23 October 2012 (UTC)[reply]

Start all over. Frank Mead's Handbook of Denominations is a great introduction to religion in the USA; despite its name, it doesn't restrict itself to Christians, so you won't see groups being left out simply because the author thought that they weren't truly Christians. Granted, it's just in the USA, but so many denominations in other countries are present in the USA that this will be a good introduction to Christianity in other countries. You'll need to buy it or borrow it from a library, as it's new enough that it's not online, but this work is perhaps the best thing that you can get in any way, shape, or form. Nyttend (talk) 02:27, 24 October 2012 (UTC)[reply]

The OP should also understand, if it's not already clear, that "tradition" in theological terms means something a bit different from the everyday, informal use of the word. Nor is there a single, universally agreed definition among various churches. If you re-read the Sacred Tradition article as well as the related Sola scriptura article, you will see that in the theological sense, "tradition" to some people means "a collection of practices and doctrines which accrue, gradually becoming something more developed" - or in other cases, "patristic, conciliar, and liturgical texts" and commentaries on them - or the "historical continuity of interpretation [of the Bible] and teaching" of a certain church. But generally speaking, all that is meant by "sacred tradition" of one church or another is in written form somewhere - basically IMO it boils down to "all the teachings of a church beyond the words of the Bible" - but it's not a big secret that only people who are already members of that church know - it can be looked up and researched for a given church or denomination. Whether it's worth your while to do so is another question, of course. Textorus (talk) 02:25, 25 October 2012 (UTC)[reply]

You can always read Wikipedia. Though, it will probably take some time trying to remember all sorts of religion-related trivia. Certainly, looking up and researching your desired information for a given church or denomination would be useful. However, one may have to keep in mind that the words are descriptory and may not capture the emotions. Perhaps, experience and emotions at a real church service can add to the knowledge about the church. 140.254.227.51 (talk) 18:33, 25 October 2012 (UTC)[reply]

Cycling and drug cheats

Why has this sport been dominated by drug cheats? Ankh.Morpork 21:03, 22 October 2012 (UTC)[reply]

Simply because it is hardly possible to compete without it, and apparently drug tests were inadequate for detecting the drugs used in many cases. Maybe a better question would be why other sports haven't. Or have they? - Lindert (talk) 21:48, 22 October 2012 (UTC)[reply]
That's circular cycular reasoning, Lindert, It's become "hardly possible to compete without drugs", precisely because the playing field has been distorted so badly by people taking drugs in the first place in order to give themselves an unfair advantage over their opponents. It's almost an "if you can't beat 'em, join 'em" situation. -- Jack of Oz [Talk] 22:21, 22 October 2012 (UTC)[reply]
I understand Lindert's argument. One person in sport X uses performance-enhancing drugs, gets away with it, and wins. The losers then realize they must take performance-enhancing drugs if they are to win (even if they don't know that the winner was using them). So, "one bad (undiscovered) apple spoils the bunch". StuRat (talk) 22:35, 22 October 2012 (UTC)[reply]
Yes, it's survival of the fittest. The fittest are the ones that use performance-enhancing drugs. It's only natural that fair players 'die out'. - Lindert (talk) 22:38, 22 October 2012 (UTC)[reply]
Although let's be careful about suggesting inferring that all successful cyclists are abusing drugs. Alansplodge (talk) 01:38, 23 October 2012 (UTC)[reply]
Yes, don't do that, or you will be infer it. StuRat (talk) 05:16, 23 October 2012 (UTC) [reply]
(edit conflict) The answer is, of course "Because they can". For decades, it was ridiculously hard to get caught, you had to be a total idiot in order to get caught. If using PEDs meant you got to make shitloads more money than everyone else that wasn't, AND there was next to no chance of getting caught if you were careful, then it seems easy to understand why. --Jayron32 22:36, 22 October 2012 (UTC)[reply]
(ec)I don't think cycling is the only professional sport dominated by doping, doping occurs in practically all professional sports. It just seems to be the one sport where there has recently been an overall campaign against it in the media as well as in some sports organisations. --Saddhiyama (talk) 22:39, 22 October 2012 (UTC)[reply]
No other sport has experienced the same level of systemic abuse. Since 1996, it has only happened once that all top three cyclists in the Tour de France have tested clean (2012) and it has occurred 13 times that all three, or two of the top three, were found to have used performance enhancing drugs. Lance Armstrong, Floyd Landis, Alberto Contador, Bernhard Kohl, Frank Scleck are examples of recent high-profile cases and it is no exaggeration to state that they represent the norm, not a minority. Ankh.Morpork 08:05, 23 October 2012 (UTC)[reply]
I agree that doping is a problem in many professional sports. I think, at least until recently, endurance athletes have had an easier time getting away with it. The drug of choice in say, sprinting or baseball is an anabolic steroid, whose metabolites are detectable in the urine a lot longer than EPO is, which a cyclist uses. It makes it easier for a cyclist to dodge drug tests: if he can get even short notice of an upcoming test, he can get clean pretty easily. Inject some saline right before blood is drawn for testing. Etc. EPO is also produced naturally by the body, making it harder to tell whether a cyclist has taken the stuff, or is just producing it at higher levels than average. The typical way to tell the difference is different patterns of glycosylation, but it's pretty subtle. Buddy431 (talk) 00:38, 23 October 2012 (UTC)[reply]
For more info how Armstrong was beating testing, as detailed by the recently released report from the USADA, you can look at [23]. There were several factors: off-season drug tests could be pretty easily avoided, the tests weren't that sensitive (or even didn't exist for some of the stuff they were taking), the team learned about "unannounced" drug tests ahead of time, the team was able to mask the signal of drugs with different techniques. That's not to say that similar techniques don't occur in other sports. The steroid THG was nicknamed The clear because (for a while) there was no test for it. This was what was used in the US baseball doping scandal a few years ago, as well as other athletes in other sports. Buddy431 (talk) 01:05, 23 October 2012 (UTC)[reply]
A lot of good points have been made above, but there's also a simple factor. To the best of my knowledge, there is no drug that improves coordination skills in the long term, so it is unlikely that any drug will have much of an effect on golfing ability, for example. In sports like baseball, where coordination and strength play a role, the effect of building muscle will be muted if someone has poor coordination. Hence, in endurance sports, there is just more to gain. IBE (talk) 02:33, 23 October 2012 (UTC)[reply]
Well because initially they (drugs, blood doping, etc) were legal - see blood doping: "Blood doping probably started in the 1970s but was not outlawed until 1986. While it was still legal, it was commonly used by middle and long-distance runners.". After they were outlaw, concerns arise regarding the reliability and competence of some laboratories doing the testing. The World Anti-Doping Agency has also been criticized for not having an effective criteria for testing for EPO. The cycling's governing body might not be trusted to do its own anti-doping, and efforts to develop a coherent anti-doping strategy have been undermined by political in-fighting. See (or listen) to the two part BBC podcast: Secrets in the blood part 1part 2 (First broadcast on 28 Jul 2008, so even back then it's a widely know secret and nothing new.) Royor (talk) 04:19, 23 October 2012 (UTC)[reply]
I've been close to a few sports at top level (no, not as a competitor), and I would suggest that cycling wasn't really trying as hard as some others to find the cheats. Timing of tests was well known, and too far from competition time. And Armstrong did bring in the crowds and money. One sport I'd like to hear more about, but which we don't hear much of in the English speaking world, is cross country ski racing. A fiend gave up 15 years ago because he was too principled to do the drugs all the leaders were doing. As for golf and similar sports, the likely area to find drugs is their use in recovery from injury. Steroids can help a lot there. HiLo48 (talk) 07:28, 23 October 2012 (UTC)[reply]
A quick Google gave lots of entries for Nordic skiing and drugs - for example, the entire Finnish ski team was banned because of drugs, and lots of Russians. It seems that there were lots of bans in the early 00's, but that the number of bans have dropped off since - with the exception of Russians. It seems still to be widespread there. With ski jumping, maybe appetite suppressants would be the drug of choice: Sven Hannawald retired because of the ravages of anorexia, and I'm sure others suffer too. Personally I'd love to know what Ole Einar Bjorndalen was on, I'd love to believe it was fresh air and an addiction to exercise. Let's see what the coming winter brings to this fantastic sport. --TammyMoet (talk) 10:10, 23 October 2012 (UTC)[reply]
Not all sports are equally dependent from endurance alone. Soccer needs some talent, basketball needs some talent, baseball needs some talent. You cannot get those things from a drugs. Comploose (talk) 09:24, 23 October 2012 (UTC)[reply]
True, but I can't see the relevance. HiLo48 (talk) 10:20, 23 October 2012 (UTC)[reply]
Answering the question: the field is dominated by drug cheats because it's possible to dominate the field cheating with drugs. If wannabe soccer/basketball/baseball could cheat through drugs, their whole respective fields would also be dominated by drug cheats, but clean players have good chances to dominate through talent. It's just a summary of things said above. Comploose (talk) 11:18, 23 October 2012 (UTC)[reply]
The fact that the international organization is going to leave all those years' titles vacant does not speak well of the stewardship of the sport by the organization. ←Baseball Bugs What's up, Doc? carrots02:51, 24 October 2012 (UTC)[reply]

Abbreviations: "Ant. ad. Magn."

What do the above abbreviations stand for? Also, it was near an abbreviation "Ern." The "Ern." abbreviation is not a name, but a title. Lastly, there was "Ern. Ant." before a name. It might be French or Latin.

Thanks for your time. Ribbiters (talk) 23:42, 22 October 2012 (UTC)[reply]

It seems it's Latin and stands for "Antiphon ad Magnificat", apparently some kind of hymn. "Ern. Ant." is used as an abbreviation of a name here e.g.: Ern. Ant. Nicolai = Ernst Anton Nicolai. - Lindert (talk) 00:17, 23 October 2012 (UTC)[reply]
The antiphon for the Magnificat is a short prayer or anthem used before the Magnificat itself at Evening Prayer. The most famous examples are the Great Advent Antiphons, used in the week before Christmas Eve - but there are plenty of others. AlexTiefling (talk) 12:00, 24 October 2012 (UTC)[reply]

October 23

Family tree

I apologize if my question is in the wrong Reference Desk subheader, but (as an example) if my mother divorces my father and marries another man, what is my family relationship to that man? Thank you in advance. 71.146.0.234 (talk) 04:58, 23 October 2012 (UTC)[reply]

He would be your stepfather and you would be his stepson. ←Baseball Bugs What's up, Doc? carrots05:06, 23 October 2012 (UTC)[reply]
Thanks. As you can see, I am the equivalent of a tomato when in it comes to family relations. 71.146.0.234 (talk) 05:18, 23 October 2012 (UTC)[reply]
A cross-pollinated tomato or a grafted tomato? —Tamfang (talk) 17:02, 23 October 2012 (UTC)[reply]
And a tomahto or a tomaydo? -- Jack of Oz [Talk] 23:55, 23 October 2012 (UTC) [reply]
Definitely a tomaydo. 71.146.0.234 (talk) 04:49, 24 October 2012 (UTC)[reply]
Probably a cross-pollinated one, but it acts as a grafted one on Sundays. 71.146.0.234 (talk) 04:49, 24 October 2012 (UTC)[reply]
As for whether he would appear on your family tree, that all depends on what you want out of it. If you only list blood relations, then he doesn't belong. However, if he has children with your mother, those half-siblings of yours are blood relations, so you might want to add him back in, to explain where they came from. :-) StuRat (talk) 05:15, 23 October 2012 (UTC)[reply]
Thanks. 71.146.0.234 (talk) 05:18, 23 October 2012 (UTC)[reply]

Stock prices

If people are trading thousands of shares from stock X at price P and then someone pays P + 1 for one single share, will the stock price be now this P + 1? Comploose (talk) 09:22, 23 October 2012 (UTC)[reply]

It depends which type of "stock price" you mean. The last traded price will now be P + 1, so in that sense the answer is yes. On the other hand, if you are tracking an average price for the day (or for a longer period) then a single trade will not change that by much. Gandalf61 (talk) 10:18, 23 October 2012 (UTC)[reply]
What is the reported price in news tickets and such? Comploose (talk) 11:08, 23 October 2012 (UTC)[reply]
Newspapers usually quote the previous day's closing price. The exact definition of "closing price" can be a little complicated. You can see the London Stock Exchange glossary here, but I didn't particularly understand the definition! --Tango (talk) 11:37, 23 October 2012 (UTC)[reply]
Once again, it depends. The Google ticker shown here is displaying the last traded price. However, in a quote driven market you might see the mid price which is the average of the lowest offer and highest bid prices currently quoted in the market - there might not be any actual trades executed at this price. Gandalf61 (talk) 11:54, 23 October 2012 (UTC)[reply]

Why didn't Israel respond to Iraqi attacks during the War???

Thank you. Iowafromiowa (talk) 11:05, 23 October 2012 (UTC)[reply]

The US asked Israel for friendly restrain, for not setting the whole area in fire. Since Saddam Hussein's strategy was to drag Israel into the conflict, it was perfectly reasonable to do the opposite. Comploose (talk) 11:13, 23 October 2012 (UTC)[reply]
Yeah, this is pretty much the main reason. Futurist110 (talk) 22:10, 23 October 2012 (UTC)[reply]

See Gulf_War#Iraq_launches_missile_strikes. The US was nominally part of a Coalition of forces, which included many Arab states. It was felt that Israeli retaliation would jeopardise the Coalition. --Dweller (talk) 12:08, 23 October 2012 (UTC)[reply]

A good deal of Coalition effort was expended in trying to prevent Iraqi Scud missiles being fired at Israel; thereby hoping to prevent domestic political pressure from forcing Israeli retaliation. Special forces surveillance combined with air strikes in the launch areas were effective, but not entirely so.[24] The success of the Patriot missiles that the US sent to defend Israel is still being debated.[25] Alansplodge (talk) 12:13, 23 October 2012 (UTC)[reply]
Actually, the missiles were a great success. Admittedly, it was a political success. The operational success is indeed very much up to debate... --Stephan Schulz (talk) 14:28, 23 October 2012 (UTC)[reply]

How much does interest cost the Greek Government?

Greece has tons of national debt at the moment, how much (annually, ideally) is it costing to pay the interest on? What % is that of GDP? And finally, I've seen on a couple of sites that 'experts' say that once you are paying 12% of GDP in interest that is a historical tipping point for default. Are there any good references for that? Many thanks, 46.30.55.66 (talk) 14:18, 23 October 2012 (UTC)[reply]

In 2011, interest payments by the government (only) were €15 billion, or 7.1% of GDP. Source: [[26]]. Bear in mind that this does not include principle, short-term lending or debt service by the private sector. DOR (HK) (talk) 06:01, 24 October 2012 (UTC)[reply]
Edit: should be "short term debt repayment, including both principle and interest," not "short term lending.DOR (HK) (talk) 06:03, 24 October 2012 (UTC)[reply]
Thanks a lot, that's great. (OP on a different IP) 86.166.191.232 (talk) 08:13, 24 October 2012 (UTC)[reply]

What games do Boy Scouts of America play that Scouts in England do not?

What games do Boy Scouts of America play that Scouts in England do not? A Scout in my troop here in England needs to teach Cub Scouts an American Scouts game that is not played in England, in order to earn his Global Challenge badge. 82.31.133.165 (talk) 14:56, 23 October 2012 (UTC)[reply]

I do not know myself (I was a scout for about a year in my early teens and rose to the high rank of Tenderfoot) but surely there are materials online that would provide better proof than a claim by some random on a Wikipedia forum?--Wehwalt (talk) 15:00, 23 October 2012 (UTC)[reply]

Online I can find some games from England and perhaps some games from America, but from these websites I am uncertain if any of the games are common only in America and not England. 82.31.133.165 (talk) 15:23, 23 October 2012 (UTC)[reply]

I realise I'm about to completely ignore your question, but please bear with me. A better way to go about this might be to support the Cubs in finding out for themselves about games in US scouting. Learning by doing, and all that. I would suggest speaking to your District or County International Adviser for support in making contact with a group in the US, and then asking them about their weekly programme. If you can't get hold of your International Asviser, try ringing Gilwell's Information Centre on 0845 300 1818 or 0208 433 7100 - they are incredibly helpful and knowledgable. Alternatively, if none of these things are possible, try getting the Cubs to do some research themselves - http://www.scouting.org would be a good place to start. - Cucumber Mike (talk) 15:31, 23 October 2012 (UTC)[reply]
(ec)The best practice on the Ref Desk has always been to link to reliable sources in answering a question. In the USA, there is Boys' Life magazine, which might lists games that Scouts play. Many issues over a period of many years are available for reading online via Google Books, such as this one from 2009. Sadly, the "games" section on page 50 only talks about videogames, not the sort where you run around playing hide and seek or whatever. The handbooks for the Boy Scout and Cub Scout organizations include games, but I no longer have any of those handy for reference in my home. An online source for games played by scouts in the US is "Boy Scout Trail". Edison (talk) 15:35, 23 October 2012 (UTC)[reply]
Well... I know that American Scouts play pick-up games of Baseball from time to time... unlikely in the UK. Blueboar (talk) 15:44, 23 October 2012 (UTC)[reply]
Kickball is even easier to put together than baseball: same basic rules, less equipment necessary. --Jayron32 16:41, 23 October 2012 (UTC)[reply]
One game they've at least played in the recent past has to do with a literal application of the old joke about starting a fire "by rubbing two boy scouts together." ←Baseball Bugs What's up, Doc? carrots02:55, 24 October 2012 (UTC)[reply]
Hilarious. You should take your act on tour. AlexTiefling (talk) 09:39, 24 October 2012 (UTC)[reply]
Well, there might be child-molesting going on the UK scouts too, but the US situation is what got publicity recently. ←Baseball Bugs What's up, Doc? carrots12:43, 24 October 2012 (UTC)[reply]
The Scout Association of the United Kingdom has been at the forefront of the development of robust child protection procedures, but no system is entirely infallible. My opinion is that jokes aren't an appropriate response to this issue, moreover, I fail to see how it is connected with the original question. Alansplodge (talk) 19:03, 24 October 2012 (UTC)[reply]
Dark humor. And the joke about rubbing two scouts together was probably innocent, as I'm fairly certain it was on TV at a time when vulgar stuff wasn't allowed on American TV. — Preceding unsigned comment added by Baseball Bugs (talkcontribs) 12:11, 25 October 2012 (UTC)[reply]
Well maybe. But when our voluntary youth worker posted his question above in good faith, he was probably hoping to avoid a load of innuendo about child abuse. That is all I have to say. Alansplodge (talk) 12:23, 25 October 2012 (UTC)[reply]
I admit to being highly suspicious of both the Catholic church and the scouts as institutions. As to the OP's question, I would think baseball and/or softball would be obvious answers, as those games are not likely to be commonly played in the UK. Pitching horseshoes also comes to mind. ←Baseball Bugs What's up, Doc? carrots12:28, 25 October 2012 (UTC)[reply]

Back to the point; try MacScouter's Great Games Resource which lists 120 "Games for Younger Scouts" (it has a picture of Cub Scouts on the title page). Good hunting. Alansplodge (talk) 19:15, 24 October 2012 (UTC)[reply]

New Mexico vs. Arizona

New Mexico is currently projected to go solid Democrat, while Arizona is solid Republican. What accounts for the difference? Arizona is several times larger in population than New Mexico and its economy is also several times larger; New Mexico seems more dependent on government funding (lots of military bases, two national labs) than Arizona. Arizona gets in the news for its harsh immigration policies; New Mexico, one would suspect, would have similar issues. New Mexico seems unique among states along the southern border of the US in its political affiliations (California excluded but California is its own weird thing). Why is New Mexico a bastion of Democratic support? --Mr.98 (talk) 15:17, 23 October 2012 (UTC)[reply]

I think you hit the nail on the head with your observation that in New Mexico, the military is a major employer. The military tends to disproportionately employ the young and minorities, both of which tend to vote more Democratic. Also, minor in comparison, the city of Taos, New Mexico is a bit of an artist colony (see Taos,_New_Mexico#Taos_art_colony), and you know how those crazy beatniks vote ! StuRat (talk) 15:29, 23 October 2012 (UTC)[reply]
Yeah, I think that the larger percentages of Latinos/Hispanics and Native Americans in New Mexico are the main things that make it more Democratic than Arizona. Futurist110 (talk) 22:08, 23 October 2012 (UTC)[reply]
Fivethirtyeight.com did a pair of articles on the two a while ago: Arizona; New Mexico. NW (Talk) 19:28, 23 October 2012 (UTC)[reply]
It was always so, at least post-1848. Arizona has always had more of an Anglo population than NM. That is, basically why they did not enter as one state, as Teddy Roosevelt wanted.--Wehwalt (talk) 19:38, 23 October 2012 (UTC)[reply]
Also, New Mexico is over 9% American Indian, while Arizona is about half that. Angr (talk) 20:59, 23 October 2012 (UTC)[reply]

Mr.98 -- In northern New Mexico there's a large grouping of Hispanics who are all U.S. citizens, and who are very conscious that they are not recent immigrants, but in fact were there before the "Anglos" were... AnonMoos (talk) 23:46, 23 October 2012 (UTC)[reply]

Winner-Take-All

Considering all states (except the two weird ones :-) ), is there a reason why states adopted a winner-take-all system when casting its electoral votes for a presidential candidate? I can understand the disadvantages of the system, the wiki article on that is quite good. But nowhere (including wikipedia and google) can I find the reason almost all of the states would adopt such a system? At least what would be the (political science) theory behind adopting such a system? Does it give one party an advantage over another? Also I can't find its history anywhere either. Has this been in place since the constitution or was it adopted slowly over the last two centuries? Did states gradually implement it or did they all do it at once? States are given almost complete power over their local election process so we have this huge variety of different rules. Winner-take-all is the only rule which seems almost universal but it isn't federally mandated like voting age being 18. I can't imagine all of the states unanimously agreeing on something out of their own free will. Just curious what led to it. It also seems to be the reason why we have had two party system for more than a century no to mention gives the media too much power and influence.70.58.0.141 (talk) 22:22, 23 October 2012 (UTC)[reply]

We did this a few weeks ago. It's to the advantage of a state, in a certain sense, because it makes it more likely that the person who is elected will be the plurality choice of the voters in that state. This is especially true for large states — in an election that's close in all states, an individual in a large state actually has more chance of flipping the election as a whole than a voter in a small state (whereas without winner-take-all it would be the reverse).
The downside is that if the election isn't close, the state gets ignored. Wait, did I say downside? --Trovatore (talk) 22:29, 23 October 2012 (UTC)[reply]
States that lack the winner-take-all rule are virtually ignored by the candidates, since there are very few electoral votes in play. If the candidate worked like hell there, maybe he could flip the vote from 45% in his favor to 55%, but that would only amount to a 10% difference in the number of state electors he got (which might not be any actual electors, if the state has less than 10). Compare this to a winner-take-all state where a much smaller effort, say one that flips his support from 49% to 51%, would gain him 100% of that state's electors (and deny them to his opponent). StuRat (talk) 03:27, 24 October 2012 (UTC)[reply]
Yes, winner-take-all makes candidates pay much more attention to large swing states. However, large states that are fairly safe for one party, like California or Texas, would probably get more attention if they dropped winner-take-all. Right now everyone knows Obama will get all of California's votes. If California went proportional, Obama would still get the majority of them, but the candidates would have to address California-specific interests in an attempt to increase or reduce Obama's margin.
However, the party in power in such states has to balance that against the fact that dropping winner-take-all would improve the other party's chance of winning the presidency. --Trovatore (talk) 04:07, 24 October 2012 (UTC)[reply]
I believe Republican strength still lies in Orange County, various areas in the California Central Valley, and in some regions in the eastern part of the state. So, yes, if California ever went proportional, it would be more than likely than some electoral votes would instead go to the Republican candidate. Zzyzx11 (talk) 04:58, 24 October 2012 (UTC)[reply]
The red states already have a disproportionate advantage in the electoral college, and turning a state like California into "proportional" electoral voting would ensure that a Republican gets elected President every time. I'm sure Republicans would like that. But you can't do that. The people don't elect the President, the states do. If they want a true proportional vote, then the electoral college needs to be abolished in favor of a direct popular-vote election. Don't hold your breath waiting for that to happen. ←Baseball Bugs What's up, Doc? carrots05:10, 24 October 2012 (UTC)[reply]
Lord, please let this not become a political debate. At least, not here.
Bugs, this isn't the place to be advocating any kind of political changes, and you know that. -- Jack of Oz [Talk] 05:26, 24 October 2012 (UTC)[reply]
I'm not advocating any change. I'm explaining the consequences of such a change. ←Baseball Bugs What's up, Doc? carrots05:55, 24 October 2012 (UTC)[reply]
Sure you are. I believe you. Just, remind me: What was that you were saying below about facts vs. spin? -- Jack of Oz [Talk] 06:46, 24 October 2012 (UTC)[reply]
Looking at the math a little closer, I see that my blanket statement was incorrect, but much depends on how the electoral votes are allocated. I took just the 2008 results and figured out the electoral vote proportioned in two different ways. The original electoral vote was 365 to 173. If you split each state's electoral votes to two decimal places, it comes out 283.14 to 246.45, with 8.41 for the miscellaneous candidates. That matches the popular vote proportion exactly. So Obama still wins, and it appears to nullify the red state vs. blue state effect. If you round within each state, you get Obama with 280, McCain with 243, and 16 missing due to the rounding. And that raises the question of what's going on with Nebraska, the one state that split its electoral vote in 2008. 5 electoral votes, 4 given to McCain and 1 to Obama. But based on popular vote proportions in the state, it should have been 3 and 2. ←Baseball Bugs What's up, Doc? carrots07:29, 24 October 2012 (UTC)[reply]
It turns out that they create "districts" within the states, and it's clear that a state could gerrymander around the theoretical need to apportion its popular vote. ←Baseball Bugs What's up, Doc? carrots07:33, 24 October 2012 (UTC)[reply]
States that want to go proportional can do it any way they want to. They have almost plenary power to appoint their own electors. --Trovatore (talk) 07:55, 24 October 2012 (UTC)[reply]
Yes they do, currently. If there is eventually a clamor for a direct election, obviously that would require a constitutional amendment and would nullify the red-states' advantage - which is why such an amendment will never pass. ←Baseball Bugs What's up, Doc? carrots12:30, 24 October 2012 (UTC)[reply]
This supposed "red-state advantage" is actually not clear at all. In 2000, a change of a small number of votes in Florida and Ohio, combined with a slightly better Bush performance in a few blue states, could have easily resulted in Bush winning the popular vote but Gore winning the election. In recent weeks CNN has been showing the popular vote between Obama and Romney as all tied up within the margin of error, but still has been giving a clear electoral advantage to the president, so an entirely plausible outcome of this election is that Romney will win the popular vote but Obama will be re-elected. --Trovatore (talk) 22:37, 24 October 2012 (UTC)[reply]
There's a move going on, slowly, that may eventually render the electoral college obsolete. There is a growing number of states who are making agreements with other states to commit their electors to whoever wins the national popular vote. This agreement won't take effect until at least 270 electoral votes worth of states enter the agreement, and they are only about halfway there yet. Unless a constitutional challenge is mounted succesfully, this approach might not only render the electoral college moot, it could revolutionize the approach to creating amendments. You could call it a pseudo-amendment. ←Baseball Bugs What's up, Doc? carrots01:27, 25 October 2012 (UTC)[reply]
I don't see how it could "revolutionize the approach to creating amendments" in general. The scheme relies on the idea that states can appoint their electors any way they want, and can therefore decide to give them to the winner of the national popular vote, if they want. That is probably true, but it seems very specific to this one issue. Most things that you'd want to accomplish by an amendment can't be implemented by just a certain number of states deciding to do it. --Trovatore (talk) 02:04, 25 October 2012 (UTC)[reply]
Well, one thing I could see would be in the area of abortion rights. Since the GOP will never get the 2/3 majority needed in the US House and Senate, and might or might not be able to get a right-wing activist court to overturn Roe v. Wade, they might take their strategy to the states and create a pseudo-amendment. ←Baseball Bugs What's up, Doc? carrots12:09, 25 October 2012 (UTC)[reply]
I don't see how they could do that. The electoral vote scheme doesn't override anything in the constitution -- it's just a way of collaborating on how to vote. Specifically what could a group of anti-abortion states agree to do that would have the effect of allowing them to ban abortion? Duoduoduo (talk) 16:14, 25 October 2012 (UTC)[reply]
It's the National Popular Vote Interstate Compact... AnonMoos (talk) 12:30, 25 October 2012 (UTC)[reply]
  • Part of the benefit of the electoral college system is that it requires candidates to focus on winning a broader area instead of just a higher vote. The nine most populous states List of U.S. states and territories by population outweigh the population of the other 41. But those 41 have an advantage of 64 electoral votes (82-18=64) over the top nine in the electoral college system. Hamilton was an elitist, and his argument that electors were more qualified than the general populce no doubt appealed to him. But I doubt his rationale was the real reason why the small states were happy with the compromise. μηδείς (talk) 17:12, 25 October 2012 (UTC)[reply]

Early Voting

A different question, why do states implement early voting? And it also looks to me like as if various states can be quite aggressive in promoting it too as if they prefer/want me to vote early. I want to wait until I make my decision so why would I vote early? Is it just so that the lines are shorter on the general election day? It doesn't sound like a good enough reason when you see how much emphasis is put on voting early. Do all states wait until the general election day to count the early votes or do some states start releasing info before November? Because if counting is done and announced before November 6 then it has the potential to drastically change the other voters' minds and swing the election.70.58.0.141 (talk) 22:22, 23 October 2012 (UTC)[reply]

Have you read Early voting, Absentee ballot, and Postal voting? As far as I can tell, such votes are never counted until polls are closed on voting day, precisely to avoid influencing the election. I've heard rumors that absentee ballots are often not counted until days or weeks later, which means (in the case of a U.S. presidential election) they don't get counted until the state has already been called for one or the other candidate. Presumably these rumors only apply to states without no-excuse early voting, as it wouldn't make sense to allow and even encourage your voters to vote early if you're not going to count the votes until it's too late for them to make a difference anyway. Angr (talk) 22:38, 23 October 2012 (UTC)[reply]
There's no "too late for them to make a difference". If the election has "already been called" in that state, it means that it wasn't close enough for the absentee ballots to change the result, and therefore they would never have made a difference, no matter when counted. If it's close enough for them to matter, you're usually in recount territory, and then they will definitely go into the mix. --Trovatore (talk) 22:41, 23 October 2012 (UTC)[reply]
I expressed myself badly. I knew what I meant. Angr (talk) 22:59, 23 October 2012 (UTC)[reply]
It might be many things but the first thing that comes to my mind is the number of volunteers that the town/district expects to be able to recruit for election day. If there are enough voters who come in and vote early, that means that the town potentially has to find fewer volunteers. In a small town like mine, it's nothing for a few people to come in every day during election day and cast their vote. It likely doesn't add that much work to the town hall staff's day. And then there's the added benefit of the small parking lot at my polling place (our volunteer fire department) not filling up with cars and lines going out the door.
Also, I can take my ballot home, look at the names, research each one of the less important races, and make a more informed decision. Dismas|(talk) 00:04, 24 October 2012 (UTC)[reply]


I have a very strong suspicion as to the reason California, specifically, promotes it so hard. Not in every case, but as a statistical average, Republicans are more likely to vote than Democrats. However, Democrats control the legislature, and while Republicans don't do very well here electorally, the Democrats would like them to do even worse. If they can increase turnout, they will increase Democratic percentages, because the marginally-committed voters they pick up are disproportionately Democratic-leaning. --Trovatore (talk) 00:12, 24 October 2012 (UTC)[reply]
Hence the various national efforts by Republicans to disenfranchise voters who might lean Democratic. ←Baseball Bugs What's up, Doc? carrots02:44, 24 October 2012 (UTC)[reply]
That's the other spin, yes. --Trovatore (talk) 02:49, 24 October 2012 (UTC)[reply]
It's not "spin". The facts speak for themselves. ←Baseball Bugs What's up, Doc? carrots02:53, 24 October 2012 (UTC)[reply]
It is spin. Everyone has a chance to vote; no one is excluded if he wants to vote badly enough. The question is, how hard do we want to try to get people to vote who don't want to that badly? --Trovatore (talk) 02:59, 24 October 2012 (UTC)[reply]
It sounds like BB is taking about things like voter ID cards etc. That being the case, whether you want to say 'how hard do we want to try to get people to vote who don't want to that badly' or 'how much do we want discourage people from voting who don't want to that badly' are both ultimately POV. Nil Einne (talk) 04:33, 24 October 2012 (UTC)[reply]
Voting is a right, not a privilege. The Republicans' efforts have been to make this right as tedious and difficult as possible for certain classes of people, and many of those attempts are being tossed by the courts who see them for what they are: attempts to discourage Democrats from voting. That's not spin, it's fact. ←Baseball Bugs What's up, Doc? carrots05:05, 24 October 2012 (UTC)[reply]
Can you show us a link? -- Jack of Oz [Talk] 05:23, 24 October 2012 (UTC)[reply]
After Trovatore does. :) ←Baseball Bugs What's up, Doc? carrots05:54, 24 October 2012 (UTC)[reply]
How 'bout this one? --Trovatore (talk) 07:58, 24 October 2012 (UTC)[reply]
Very good. Meanwhile, you may find Voter ID Laws in the United States somewhat enlightening. ←Baseball Bugs What's up, Doc? carrots12:36, 24 October 2012 (UTC)[reply]

Promenades en Océanie: les Tubuaï et l'archipel de Cook

Does anybody know when the events written down by M. Aylic Marin in Promenades en Océanie: les Tubuaï et l'archipel de Cook occurred? It was published in 1885, but that would be impossible since the figures mentioned in it died in the 1870s. I need the year he went to those islands and even exact dates if possible. I translated some of it here. Also is M an abbreviation of his first name or Monsieur? And who was he?--KAVEBEAR (talk) 22:51, 23 October 2012 (UTC)[reply]

I'm surprised you didn't try googling for "Aylic Marin". The first link it takes you to is http://fr.wikipedia.org/wiki/%C3%89douard_Petit_%28%C3%A9crivain%29, which answers most of your questions. Looie496 (talk) 23:39, 23 October 2012 (UTC)[reply]
So that narrow the visit between 1880 and 1885. Can somebody find a more specific year?--KAVEBEAR (talk) 23:48, 23 October 2012 (UTC)[reply]

Rurutu and Rimatara

Was the people of Rurutu and Rimatara converted into the Calvinism denomination or the Wesleyan denomination? --KAVEBEAR (talk) 23:01, 23 October 2012 (UTC)[reply]

Well, Rimatara was converted by two native missionaries dropped off by Samuel Pinder Henry, who was the son of William Henry, who was an Anglican missionary of the low church sort. So most likely Anglican, but possibly slanting in the Calvinist direction. Looie496 (talk) 23:32, 23 October 2012 (UTC)[reply]

October 24

Priest vs. Clergy

Are there more priest in prison for child offenses than other types of clergy, say Baptist or Lutherans? I am not singeling out anyone, I really am interested in the percentages. Many thanks, JeffJDLane13 (talk) 01:20, 24 October 2012 (UTC)[reply]

When you say "priest", are you referring to Catholic clergy? -- Jack of Oz [Talk] 01:21, 24 October 2012 (UTC)[reply]

Yes I am and thanksJDLane13 (talk) 01:45, 24 October 2012 (UTC)[reply]

Define "more". Total per denomination? Or percentage of the given denomination's clergy? ←Baseball Bugs What's up, Doc? carrots02:41, 24 October 2012 (UTC)[reply]
Also: in one particular country, or worldwide? Marnanel (talk) 04:58, 24 October 2012 (UTC)[reply]
And bear in mind that there are 1.2 billion Roman Catholics, and only 75 million Lutherans.--Shantavira|feed me 07:27, 24 October 2012 (UTC)[reply]
The OP did say he's interested in the percentages, which I suppose means looking at the relative proportion of offenders within each denomination. The Catholics do seem to get most of the bad press in this area. That's partly because there are a lot more Catholic priests and brothers than religious of other denominations, and it may be that some much smaller group has a higher proportion of offenders within their ranks than the Catholics do. But I hope that's never going to be used to downplay or point-score, along the lines of "We Catholics only have 15% of our people up on child molestation charges worldwide. You Discalced Restricted Calathumpians of the Blessed Foreskin of Zebedee have 25%, and you're the folks who obviously need to clean up your act before pointing the finger at us". -- Jack of Oz [Talk] 11:19, 24 October 2012 (UTC)[reply]
Overall, I expect you would find the percentage of convicted felons in any denomination is an extremely low percentage. You occasionally hear about child-molesting charges in Protestant denominations. It seems to be same-sex among the Catholic and opposite-sex among the Protestant, which is not necessarily an endorsement or a condemnation of either denomination. ←Baseball Bugs What's up, Doc? carrots12:41, 24 October 2012 (UTC)[reply]
Not necessarily an endorsement? You mean it could be a plus, either way? And not necessarily a bad thing, either way? Regardless of who commits them, are you actually defending these atrocities?
Here's what the Deputy Police Commissioner of Victoria said last week to an official enquiry being held by the Victorian Parliament. -- Jack of Oz [Talk] 21:06, 24 October 2012 (UTC)[reply]
I think Bugs merely meant that there was no moral lesson to be drawn from the apparent gender biases of child abusers from different denominations, which is certainly true. However, I'm not convinced that such biases are statistically significant, and anecdotal evidence has suggested that women and girls abused by celibate clergy have a harder time being believed, because of the alleged same-sex preference factor. Moreover, Bugs seemed to find the topic amusing enough when he joked about it in the Boy Scout thread further back - in response to a question apparently asked by a Scout, no less. AlexTiefling (talk) 07:44, 25 October 2012 (UTC)[reply]
I call it "dark humor". The common thread among these institutions is their public condemnation of "non-conforming" adult sexual interactions, while also tolerating and hiding predatory sexual interactions. As for the scouts specifically, the joke about starting a fire by rubbing two scouts together I first heard several decades ago, probably on TV, and I don't think it had any nefarious implications, although I was really too young to know about such things. ←Baseball Bugs What's up, Doc? carrots11:55, 25 October 2012 (UTC)[reply]
The title of this section keeps bugging me. Priests are clergy; there are clergy who are not priests; there are priests who are not Roman Catholic. AlexTiefling (talk) 11:49, 24 October 2012 (UTC)[reply]
Yes. It's a bit like saying "Pastors vs. Ministers". ←Baseball Bugs What's up, Doc? carrots12:37, 24 October 2012 (UTC)[reply]

The question is comparing apples and oranges. Baptist and Lutheran ministers may marry. You'd have to compare among all unmarried or all married ministers. μηδείς (talk) 16:04, 24 October 2012 (UTC)[reply]

I don't get you Medeis. Married men are just as likely to be child-abusers as are single men, possibly more so as they have more opportunity. DuncanHill (talk) 19:25, 24 October 2012 (UTC)[reply]
You don't get me? More like you are not familiar with how to set up an objective experiment. One compares like to like and controls for independent variables so far as possible. μηδείς (talk) 05:00, 25 October 2012 (UTC)[reply]
wouldn't the later be part of the point though? Nil Einne (talk) 04:31, 25 October 2012 (UTC)[reply]
You could do both. Set it up as an observational study and use multivariate analysis to see if unmarked Catholic priests were more likely to abuse than other clergy in general and if they were more likely than other unmarked clergy You could even see what effect adjusting for the relative density of clergy has. Itsmejudith (talk) 07:11, 25 October 2012 (UTC)[reply]
That's what you might do in order to discover whether or not there's any correlation between marital status and propensity to sexually offend. But Medeis seems to be taking this as a given. Until otherwise demonstrated, the marital status of sex offenders is about as relevant as the colour of their hair or their astrological sign. Unless it's being suggesting that men abuse children or rape people only because they're sexually frustrated as a result of not being married. If so, where's the proof? And what about people who are not married in the eyes of any church or the law, but have ready access to one or more adult sexual partners? -- Jack of Oz [Talk] 08:06, 25 October 2012 (UTC)[reply]
I googled [list of clergy convicted of sexual abuse] just to see what might turn up, and there is no shortage of such lists. This one may be of some interest. The OP could add up the counts given for first letter of last name and divide by the total number of priests to get some very rough idea of a percentage. I googled [number of catholic priests in usa] and one of the items that turned up was Catholic Church in the United States, which suggests America has 40,000+ priests. One caution would be that 40,000 is only the active number of priests, so the ratio of suspects to priests would be inflated, but it would give an upper bound. It's possible someone has done the math on both Catholics and other denominations, which would require further googling. ←Baseball Bugs What's up, Doc? carrots12:06, 25 October 2012 (UTC)[reply]

Tax cuts

If we are to offer people in the U.S. tax cuts with the premise of helping American achieve jobs, innovation and etc., can it be mandated that the money offered must remain in the U.S.? Why would give tax breaks for someone to take it out of the the offering country to another because the make more profit? Wouldn't this defeat the intended purpose? Thanks JDJDLane13 (talk) 01:28, 24 October 2012 (UTC)[reply]

The Reference desks are intended for questions that can be answered with facts or references. That doesn't hold for this question, so it really isn't appropriate here. Looie496 (talk) 01:52, 24 October 2012 (UTC)[reply]
The first question is appropriate for the ref desk. The second and third are the problem. Someguy1221 (talk) 02:01, 24 October 2012 (UTC)[reply]
How exactly would you enforce such a law ? It wouldn't be enforceable on an individual level like that. However, multinational corporations are required to report where they spend their money, so it might work on that level. StuRat (talk) 03:19, 24 October 2012 (UTC)[reply]
Money is fungible... the multinational would simply take some of the funds it was originally planning on spending in the US and spend it elsewhere instead. Blueboar (talk) 03:44, 24 October 2012 (UTC)[reply]
On a technical level, the refund could be in the form of a debit card that is only authorized for purchases from American corporations, although that doesn't guarantee the money will stay in the US in the end. Don't food stamps now come in the form of a debit card that only works at places that sell groceries? Someguy1221 (talk) 03:29, 24 October 2012 (UTC)[reply]
I believe food stamps were always restricted to certain foods. They now have the bridge card, with similar restrictions. Also, a tax cut isn't the same as a tax refund. If the government never has the money, it can't really control what people do with it. StuRat (talk) 03:35, 24 October 2012 (UTC)[reply]
The premise of the question is that keeping the money in the US is best, but that's not at all clear. Suppose that I got a tax cut of $20,000, and I used the money to buy $15,000 of Vietnamese blue jeans with fancy custom stitching on the pockets, then spent $2,000 to have them shipped here and to have, I don't know, $1,000 worth of rivets from Mexico riveted onto the legs in downtown Los Angeles, and then I spent $2,000 on web banner ads on a Canadian website, and I sold half of the jeans here in the US for $30,000, and half in Canada for $20,000. My profit is $30,000, all of which I then use to buy a Honda Accord that was assembled in Ohio. And I use the $20,000 left over to do this again with the blue jeans. Is this so bad? An American (me) now has a $30,000 car plus this interesting cash engine of a blue jeans business. Globalization is super-complicated and although there is an obvious appeal to spending money within the US, because it adds to the country's GDP, it might be better for numerous Americans in the chain if the money goes outside of the US in exchange for something else. Tarcil (talk) 03:56, 24 October 2012 (UTC)[reply]

People are talking about "money offered" and "refunds", which is not what tax cuts are about. The govt would simply be taking less off you than they would otherwise have taken. They're not giving you anything. You're still giving money to them, but less of it. -- Jack of Oz [Talk] 05:21, 24 October 2012 (UTC)[reply]

The idea of Tax expenditures is well-accepted in some contexts... AnonMoos (talk) 06:09, 25 October 2012 (UTC)[reply]
They might be giving you something. Tax credits fall into two classes: refundable and non-refundable. The difference is that only refundable tax credits can bring your tax burden below zero. (An example of a refundable tax credit is the EIC in the US.) If you have enough refundable tax credits on your income tax, you will receive money from the government at the end of the year. They are commonly used as a subtle method of wealth distribution to ensure the poor don't actually starve, while not riling the right wing about "handouts". Marnanel (talk) 06:42, 25 October 2012 (UTC)[reply]

Multiple manslaughter

Is "multiple manslaughter" really a crime in Italian law, or does the phrase simply mean "multiple counts of manslaughter"? Our article on the 2009 L'Aquila earthquake speaks of scientists being convicted of "multiple manslaughter", as does the BBC, so I'm rather confused. Nyttend (talk) 02:29, 24 October 2012 (UTC)[reply]

The news stories I've seen in the Italian press say omicidio colposo plurimo, "plural manslaughter". I don't know whether that's a legal phrase in Italian or not, but at least the Beeb came by it honestly. --Trovatore (talk) 03:02, 24 October 2012 (UTC)[reply]

Mayor of Rurutu

Does anybody know the names and terms of the Mayors of Rurutu? I need to know the name of the mayor in 1982.--KAVEBEAR (talk) 05:41, 24 October 2012 (UTC)[reply]

According to Teuruarii IV's article, his grandson, Toromona (Solomon) Teuruarii, was "mayor of Rurutu in the 1970s". The JOURNAL OFFICIEL DE LA POLYNESIE FRANGA1SE (sic) lists "Solomona Teuruarii" as le maire on 8 May 1980. Clarityfiend (talk) 07:10, 24 October 2012 (UTC)[reply]

Pierre De Chevigne

Was Pierre De Chevigne Secretary of Defence in the French Government in November 1952 because an account in the National Archives,ref. 751G.00/11-2452 of RG 59 of 24 November, has him visiting Indochina for three weeks using that title. You have withthat title only from May to June in 1958.124.176.54.133 (talk) 07:38, 24 October 2012 (UTC)[reply]

The French wikipedia has a detailed list of Chevigné's various mandates [27]. He was Secretary of State for War ("Secrétaire d'État à la Guerre") from August 1951 to June 1954, keeping the title under five successive Prime Ministers in those thays of short-lived governements. The title means he was a junior Minister of Defense responsible for the Army; in those days, France has a Minister of National Defense, an Associate Minister of National Defense (at times), and Secretaries of state for War, for the Navy and for the Air Force. He was briefly the actual Minister of Defense from May-June 1958, in the short-lived Pflimlim Government that was the last government of the IVth Republic. In November of 1952, René Pleven was the Minister of National Defense, and Chevigné was one of three Secretaries of State reporting to him. See here [28]. --Xuxl (talk) 08:34, 24 October 2012 (UTC)[reply]

Electoral College - Tie Scenarios

I am sure they exist somewhere, but I can't to seem to find them. Where can I find a list of possibilities for an Obama/Romney tie? or even colored maps showing such scenarios? Also, what is the most likely path towards a tie come election day? Hisham1987 (talk) 08:17, 24 October 2012 (UTC)[reply]

This has been discussed a bit on FiveThirtyEight.com. The entry on 1st October 2012 discusses it in detail. 86.166.191.232 (talk) 09:43, 24 October 2012 (UTC)[reply]
See Electoral College (United States)#Contingent presidential election by House and Twelfth Amendment to the United States Constitution. In any case where someone doesn't receive a clear majority (more than half) of the electoral votes, then the election of the President goes to the House of Representatives, and the election of the VP goes to the senate, with each state's delegation (not individual legislators, the delegation as a whole) getting one vote. A clear majority (not merely a plurality) is needed for a win: it can be a perfect tie, or it can be a case where there are three candidates receiving votes, and none gets more than 1/2. Per the text of the twelfth amendment "if no person have such majority, then from the persons having the highest numbers not exceeding three on the list of those voted for as President, the House of Representatives shall choose immediately, by ballot, the President. But in choosing the President, the votes shall be taken by states, the representation from each state having one vote" --Jayron32 12:03, 24 October 2012 (UTC)[reply]
Note that the Senate would choose the vice president from among the top two electoral vote recipients for vice president, a majority being necessary. This would be the newly-elected House and Senate, by the way, as they do not meet for the joint session at which the electoral vote is counted until after January 3. Not sure if the vice presidential tiebreaking vote applies, if it goes 50-50.--Wehwalt (talk) 12:12, 24 October 2012 (UTC)[reply]
I think Jayron's and Wehwalt's posts miss the point of the OP's question, which I believe is what plausible combinations of states going for Romney and states going for Obama would lead to a tie. Interesting question. I can't see how to get to the 1 October article on FiveThirtyEight.com that was referenced by 86.166.191.232. Duoduoduo (talk) 14:28, 24 October 2012 (UTC)[reply]
It's here [29]. It's easy to get to by scrolling to the bottom of the blog and following the link to older enteries and keep going back until you find the enteries for 1 October 2012 and then finding the relevent post. You have to go back a few times since the blog is updated regularly but it shouldn't take long, it was on page 5 when I looked for it just now (and you can always modify the page number in the URL to skip a few pages). Nil Einne (talk) 15:57, 24 October 2012 (UTC)[reply]
Thank you everyone - OP here. The fivethirtyeight link answers my questions perfectly. — Preceding unsigned comment added by Hisham1987 (talkcontribs) 16:13, 24 October 2012 (UTC)[reply]
(edit conflict)I know he said this was answered, but I just spent like 30 minutes working this out, so I'm posting it anyways, after the edit conflicts. AH! Sorry, in that case, assuming no state ends up going for a third party candidate, there are quite a number of scenarios leading to a 269-269 split. In general, most of the states aren't "in play", that is some states will vote Republican, and others Democrat regardless of who is running as a candidate (back in the days of the Solid South, this was referred to as Yellow Dog Democrats: people who would vote for a Yellow dog than vote for a Republican. This concept still applies to both parties in certain states, such that there is no realistic chance of having a close race in them). Looking at the article Red states and blue states and Electoral College (United States) we see the following breakdown, based on recent past performances of the parties:
  • Romney is probably a "lock" in the following states: Idaho 4, Montana 3, Utah 6, Wyoming 3, North Dakota 3, South Dakota 3, Nebraska 5, Kansas 6, Oklahoma 7, Texas 38, Alaska 3, Alabama 9, Mississppi 6, Georgia 16, South Carolina 9, for a total of 121 votes
  • Obama is probably going to win all of the following states: Washington 12, Oregon 7, California 55, Hawaii 4, Minnesota 10, Wisconsin 10, Illinois 20, Michigan 16, Maryland 10, Delaware 3, Pennsylvania 20, New Jersey 14, New York 29, Vermont 3, Massachusetts 11, Connecticut 7, Rhode Island 4, Maine 4, D.C. 3, for a total of 242
  • The following states are thus "in play", or "swing states" as they are popularly known: Nevada 6, Arizona 11, New Mexico 5, Colorado 9, Iowa 6, Missouri 10, Arkansas 6, Louisiana 8, Indiana 11, Ohio 18, Kentucky 8, Tennessee 11, West Virginia 5, Virginia 13, North Carolina 15, New Hampshire 4, Florida 29, for a total of 175
Several of the above "swing states" are probably more in one camp than the other, but this is probably the largest list of states where the election could go either way (realistically, I've seen several news reports that narrows it down to just 4-6 states: Ohio, Virginia, Florida, and North Carolina being the most important as they are closest to a 50-50 split, and have enough electoral college votes to make it worth the while. Also, there are a few "solid" states I wouldn't be surprised to swing this election. Georgia has a strong, affluent African-American population in and around Atlanta that could swing the state Democrat, likewise a state like Pennsylvania could also be in play for the Republicans, likewise Romney's home state of Michigan, or Massachusetts where he was Governor). However, just on the lists above, which gives the widest, most realistic chance of making the election, if every candidate took exactly the states they are supposed to, you could have the following swing states break the following ways:
  • Obama gets: Virginia (13), New Hampshire (4) Missouri (10) = 27 more than his "locks" = 269 total
  • Romney gets all the rest for 269 total.
There's your tie. There's probably many other ways it could break down for a tie, (especially if either Nebraska or Maine split their vote, as they do from time to time) but there's one realistic scenario. --Jayron32 16:24, 24 October 2012 (UTC)[reply]

Finding ties in the electoral college is an interesting example of a subset sum problem. Duoduoduo (talk) 16:25, 25 October 2012 (UTC)[reply]

For any given value of 'interesting'. --Jayron32 18:58, 25 October 2012 (UTC)[reply]

Commanders who never lost a battle

From the introduction of the Jan Žižka article: "Žižka is ... one of six commanders in history who never lost a battle (alongside Alexander the Great, Scipio Africanus, Genghis Khan, Alexander Suvorov, and Khalid ibn al-Walid)." The introduction of Alexander Suvorov lists Lucius Cornelius Sulla instead of Scipio Africanus. But other articles attribute the same virtue also to other military leaders, in a trice namely Paul von Lettow-Vorbeck and possibly Uesugi Kenshin. I assume the lists above to be original research and POV; where do they come from? Are there lists that might possibly claim justifiably to be complete? But then, there might be several military leaders who just won a single battle in their life and then died of a disease... any help to clear this is welcome. --KnightMove (talk) 08:37, 24 October 2012 (UTC)[reply]

This seems a rather vague thing to try to define. Would a commander who never fought a battle count ? How about a commander who only fought one ? Or a commander who avoided all battles except the easy victories ? None of these sound particularly heroic, do they ? StuRat (talk) 08:57, 24 October 2012 (UTC)[reply]
Yes, it's a pointless categorisation. Someone who only became involved in a war in its dying stages could easily score that label. HiLo48 (talk) 09:37, 24 October 2012 (UTC)[reply]
To say nothing of the fact that a number of the commanders above probably did loss a battle. Alexander the Great for example lost a number of engagements, it has just become common not to count any of them as a full battle, but in that case what are we really recording? 86.166.191.232 (talk) 09:41, 24 October 2012 (UTC)[reply]
I agree it is a pointless characterisation, and furthermore, unless it is directly stated in a reliable source, it is synthesis and should be removed from Wikipedia on that ground. I suggest you remove it; or discuss it on the article's talk page if you are not confident in doing that. --ColinFine (talk) 10:11, 24 October 2012 (UTC)[reply]
What about American commanders who led only in the Spanish-American War, say, and retired before WWI?--Wehwalt (talk) 12:14, 24 October 2012 (UTC)[reply]
It can't possibly be supported by a viable source in any event. Definitely a contentious claim and almost assuredly someone's original research; definetly needs to be removed. Snow (talk) 05:17, 25 October 2012 (UTC)[reply]
The list is blatantly incomplete. Marlborough is famous for never having fought a battle he didn't win, and never having besieged a city he didn't take. Looie496 (talk) 14:47, 24 October 2012 (UTC)[reply]
I've just removed this claim from the article per the above discussion. Nick-D (talk) 22:45, 25 October 2012 (UTC)[reply]

Why hasn't the electoral college system used in the United States caught on in the rest of the world?

With the US presidential election happening in a few weeks, this question came into my mind. The United States is extremely unique among presidential republics in that people don't actually directly vote for the president, but rather the people vote for a group of people who will make the decision for them. As I pointed out in another related question here, this is, to my knowledge, the only system of kind in the world (the electoral colleges of India and Pakistan being composed of lawmakers rather than a group of people elected specifically to elect the President). The closest equivalents would probably be the Election Committee of Hong Kong which elects the Chief Executive (although I'm not sure if the Committee's members themselves are elected or are chosen by interest groups) or the Assembly of Experts which elects Iran's Supreme Leader (although he isn't a president). Interestingly, the American sytle of government has been copied around the world, especially in South America, except for the electoral college. The question is, why? Why has the concept of an Electoral College not caught on in the rest of the world? I know that the College was a product of the Great Compromise, but if other countries can copy other elements of the US government (like having an executive president with veto powers), then why didn't they copy the Electoral College? Narutolovehinata5 tccsdnew 12:45, 24 October 2012 (UTC)[reply]

Why would they? What would be the advantages of the elctoral college system? The US is one of the oldest electoral democracies around, and as a result it has a highly anachronistic electoral system. --Soman (talk) 12:49, 24 October 2012 (UTC)[reply]
Honestly, I'm not a big fan of the electoral college system (I prefer direct elections), but I can see why they used it. it would benefit states with small populations. This would mean that even the electoral votes of, say Wyoming or Alaska, would have the same weight as the votes of Texas. Not really, but it would mean a more proportionate voting. That is why they had the Great Compromise in the first place. Narutolovehinata5 tccsdnew 12:56, 24 October 2012 (UTC)[reply]
(ec) It's not at all clear on the surface that the Great Compromise -- the disproportionate power granted to small states in the Senate -- is tied to the Electoral College. Without knowing the Federalist Papers offhand, I rather suspect that the reasons for the College shape closely to Article 1 Section 3, that is, the indirect election of Senators (which has no possible ties to state size). Note further that there is no constitutional requirement that Electors vote in accordance with, well, anything. Strictly speaking, there's not even a requirement that a popular election be involved in Elector selection (and this is a key component of the National Popular Vote Interstate Compact, which is attempting to tie state electoral votes to the winner of the national popular vote, regardless of state-level results). So no, it's not a Great Compromise thing, but rather a means of insulating the more powerful offices (i.e. anything but Representatives) from the common voter. I'm sure the Federalist Papers go into detail somewhere about why that gap was written in, both for the President and for Senators. — Lomn 13:48, 24 October 2012 (UTC)[reply]
Relevent Federalist Papers to the topic worth reading (a great insight to understanding the mindset and rationale of the Constitution writers themselves) are Federalist No. 49 and Federalist No. 51 (on the need for checks and balances generally), Federalist No. 10 (on the problem with factions, and on the need to institute measures in the government to combat factions, even factions of the majority, aka Tyranny of the majority), Federalist No. 39 is relevent, as a sizable portion of it deals with the inherent nature of whether the national government was intended to represent the Governments of the States, or the People directly: measures were built in to address both, but directly relevent is that the Presidency and the Senate were supposed to be representatives of the states, while the House of Representatives were representatives of the people. However, if we really want to get down to brass tacks, the relevent paper is Federalist No. 68, titled by Hamilton "The Mode of Electing the President", the text is availible at Wikisource if you want to read it, but especially germane here is the passage:

It was desirable, that the sense of the People should operate in the choice of the person to whom so important a trust was to be confided. This end will be answered by committing the right of making it, not to any preëstablished body, but to men chosen by the People for the special purpose, and at the particular conjuncture. It was equally desirable, that the immediate election should be made by men most capable of analyzing the qualities adapted to the station, and acting under circumstances favorable to deliberation, and to a judicious combination of all the reasons and inducements which were proper to govern their choice. A small number of persons, selected by their fellow-citizens from the general mass, will be most likely to possess the information and discernment requisite to such complicated investigations. (bold mine)

The founding fathers believed in representative democracy over direct democracy because they believed that the great masses of people were easily swayed by small, inconsequential things, and that it would be trivial to establish a majority which would decide issues that, in the end, would not be in the best interest of the country. The idea would be that wise, well-trained, and well informed individuals would themselves be selected by the people, and THOSE individuals would make all of the important decisions. The "electoral college" was intended to be as independent and vital as the Senate and House of Representatives; it was basically intended to be a third, special-purpose legislature which would meet, debate and deliberate, and then elect a president from their deliberations. That was the clear intent of the body from Federalist No. 68. Now, that it very quickly devolved into a mere formality is a historical fact, but was not something the Constitution writers envisioned or hoped for: the original intent was that the Electoral College would be chosen, not to represent specific candidates (as is done today), but to represent the people generally, and would through their collective wisdom and deliberation, elect a President themselves, but that the members of the College would be unencumbered by direct association with any party or faction. It is somewhat ironic that the actual operation of the College works exactly opposite of that: the members of the Electoral College are tied directly to the party they represent, and are in many cases actually forbidden by state law from voting independently. --Jayron32 14:30, 24 October 2012 (UTC)[reply]
Judging from what I read at the time of the Bush-Gore debacle, and from the article on the topic, the whole point was a compromise between secondary election (by individual states) and direct election (just one person - one vote). The downside is that it currently works much like a direct election, with this intermediate business of electoral colleges. I see no reason to imitate. IBE (talk) 13:40, 24 October 2012 (UTC)[reply]
(ec) And that (the fact that it was designed to suit the peculiarities of US politics back then) is in a nutshell why it has not caught on elsewhere.
The electoral college is mechanically similar to any other method of choosing a leader via electorates - for example the way the UK prime minister is chosen (voters voting in electorates for MPs, the candidate commanding the support of the largest number of MPs gets to be prime minister). However, in reality the electoral college is just a slightly less democratic version of a directly elected system. So in countries where there isn't the same precise pressues that led to the electoral college model being adopted (and, countries being as different as they are, these would be few and far between), it would be more expedient to simply choose a directly elected model, or an appointed model, or a parliamentary model, or a different kind of compromise to suit their own situation.
So, as the OP mentioned, the "electoral college" in Hong Kong is partly directly elected, partly elected by interest groups, and partly appointed. It is a different kind of compromise, this one between the pressues of democracy, interest groups and the Chinese government's wish for control over the process. --PalaceGuard008 (Talk) 13:43, 24 October 2012 (UTC)[reply]
I would also be cautious about simply regarding a country's system as "copied" from another. Most mature political systems are compromises of some sort, adopting elements of earlier systems and applying certain innovations or reforms. There are quirks to each country's system, so it would be rare to find one that's identical to another in any respect, not just the electoral college. --PalaceGuard008 (Talk) 13:47, 24 October 2012 (UTC)[reply]
Narutolovehinata5 -- the electoral college was well-adapted to the U.S. situation in the late 18th century and early 19th century: i.e. a semi-loose federation spread over a wide geographic area with relatively poor communications, where there was a legitimate concern that voters in one state might not know enough about politicians in other states to have an informed opinion on them, and there was little desire to have centralized government control to the degree that voting qualifications and election administration could be standardized between the individual states in a manner which would allow for throwing all the votes in the states together into one common national pool (a necessity for a national popular vote system). So as late as 1824 there weren't really national presidential candidates, but only a series of regional candidates; as late as the Dorr Rebellion of the early 1840s there was resistance in some areas even to Jacksonite "universal white manhood suffrage"; and as late as 1860, South Carolina didn't allow individual voters to vote for president at all.
Of course, there's no reason why what was found to be necessary for the U.S. in the late 18th century and early 19th century should be considered ideal for any other country in 2012... AnonMoos (talk) 14:04, 24 October 2012 (UTC)[reply]
That's fascinatng, AnonMoos -- as late as 1860, South Carolina didn't allow individual voters to vote for president at all. Do you mean just in 1860, or in all years up to and including 1860? Can you provide a reference about this? Thanks. Duoduoduo (talk) 14:38, 24 October 2012 (UTC)[reply]
It's also true: From 1788-1860 inclusive, South Carolina's state legislature selected it's Presidential Electors, without the use of a popular election. Start at United States presidential election, 1860 and work backwards and you can confirm that there was no popular vote in South Carolina. As you go backwards, you can also confirm that there was no popular vote in progressively more and more states. South Carolina was a pretty long hold-out in not using a popular election, but United States presidential election, 1828 shows that Delaware didn't hold a popular vote either, and United States presidential election, 1824 shows 6 states which appointed the electors in the Legislature, and United States presidential election, 1816 has a near even split: of the 19 states at the time, 10 used a form of popular vote, with the other 9 having electors appointed by the legislature. Prior to that, about half (more or less, depending on the number of states at the time) didn't have a popular vote. --Jayron32 15:24, 24 October 2012 (UTC)[reply]
After the 1824 election (whose outcome was considered deeply undemocratic by many, see Corrupt_Bargain#Election_of_1824), there was the rise of "Jacksonian democracy", which in presidential elections meant a close approximation to universal white manhood suffrage, and direct popular election of presidential electors -- Rhode Island and South Carolina being the main holdouts... AnonMoos (talk) 16:58, 24 October 2012 (UTC)[reply]
Please say "male sufferage". When you say "manhood sufferage" it makes it seem like only a specific part of the body is voting. Of course, that lends an interesting twist on the phrase "pulling the lever" (from the old "lever style" ballot-marking machines) to refer to voting. --Jayron32 17:37, 24 October 2012 (UTC)[reply]
And actually the word is "suffrage". I don't even want to think about "manhood sufferage"...--TammyMoet (talk) 17:57, 24 October 2012 (UTC)[reply]

I'd vote against that... --Jayron32 18:01, 24 October 2012 (UTC)[reply]
I didn't come up with the phrase -- see article Universal manhood sufferage... AnonMoos (talk) 19:28, 24 October 2012 (UTC)[reply]
Which redirects to the correctly spelt Universal manhood suffrage. Which in turn ought to be moved to Universal male suffrage because the "manhood" version of the terminology is outdated, but that's already a redirect to Universal suffrage. -- Jack of Oz [Talk] 20:53, 24 October 2012 (UTC)[reply]
"Universal manhood suffrage" was the historically-used term, and doesn't create any potential confusion about whether boys are allowed to vote... (By the way, I was spelling it correctly before I picked up on Jayron32's misspelling.) AnonMoos (talk) 00:23, 25 October 2012 (UTC)[reply]
In 2005 there was an unsuccessful proposal on Talk:Universal manhood suffrage to change manhood to male. Duoduoduo (talk) 16:47, 25 October 2012 (UTC)[reply]
I'm going to change the redirect for Universal male suffrage away from Universal suffrage to Universal manhood suffrage. Duoduoduo (talk) 16:47, 25 October 2012 (UTC)[reply]

Why? Because there doesn't exist a single country structured like America. America is unique in this respect, and is foremost a nation of states. During the Civil War, a lot of people chose sides based on what state they lived in, even if they disagreed with ideaology over slavery. Befor the American Revolution, people identified themselves not as Americans, but as a Virginian or a New Yorker... In orderto properly represent the views of all these states, rather than represent only the views of about 5 large states with large populations that can elect a president, the electoral college is a unique system that works. --Jethro B 00:02, 25 October 2012 (UTC)[reply]

Not exactly so. Switzerland was assembled much like America: originally as an association of essentially independent states that have gradually, over time, grown into a more federal (than confederal) relationship. See Old Swiss Confederacy for some perspective. South Africa also has its history as a voluntary federation of independent states, but it is today a unitary state. America is certainly an unusual type of nation, given that more states are "unitary states" without any subnational unit having true sovereignty, and most of the remaining federations have a "Strong federal" model whereby the National law supercedes State/Provincial law in more cases than not: In countries like Canada, unless something is specifically assigned to the Provinces as their responsibility, it is reserved to the Federal government. The U.S. has a "weak federal" model whereby the National government has only a series of "enumerated powers" and other than those specific powers and roles (such as military, currency, interstate commerce, etc.) all other sovereign powers are specifically reserved to the states. Unusual, but not sui generis. Other states with similar arrangements include the aforementioned Switzerland, the United Arab Emirates and Malaysia. --Jayron32 03:08, 25 October 2012 (UTC)[reply]
Every country is different. Australia is also an assemblage of formerly separate colonies, and federal powers are also enumerated in the constitution, either as exclusive to the federal government or as shared by the federal and state governments (but with federal laws prevailing to the extent of any inconsistentcy). All other powers not so enumerated is reserved to the states.
However, Australia has never had a need for an electoral college system because the Australian head of government is selected by the majority parliamentary party from among its ranks, and the Australian head of state in practice (the Governor-General) is mostly ceremonial and nowadays basically appointed by the government.
Every country is different, and no doubt reasons also exist why the other countries you listed would not have adopted an electoral college system. --PalaceGuard008 (Talk) 09:35, 25 October 2012 (UTC)[reply]

female muezzin and female imams reading the prayer out loud during Maghrib, isha and fajr prayers.

What does islam say about women being muezzin for women-only congreagtion only and also, if a woman leads the maghrib, isha and fajr prayers for the women-only congregation, does she read the prayers out loud or not? — Preceding unsigned comment added by Donmust90 (talkcontribs) 16:20, 24 October 2012 (UTC)[reply]

I expect that someone who knows more about this than I do will be along shortly. But just as with the earlier question about female clergy in Christianity, there's no single unified position about this within Islam. I remember reading recently about a large programme in Turkey to train women for ministry as imams. This was met with a wide range of reactions, and this reflects the complexity and diversity of opinion - popular and scholarly - even in such a clearly Muslim-majority country as Turkey. AlexTiefling (talk) 16:32, 24 October 2012 (UTC)[reply]
Wikipedia articles Islamic_feminism#Equality_in_leading_prayer and Women as imams... -- AnonMoos (talk) 17:05, 24 October 2012 (UTC)[reply]

October 25

the price of David Copperfield's caul

What does "winner to spend" mean? That the winner receives 5s spending-money? Or that the winner, to claim the grand prize, must put up another 5s (twice the price of the raffle ticket)? —Tamfang (talk) 00:36, 25 October 2012 (UTC)[reply]

Sounds like he won an option. μηδείς (talk) 02:55, 25 October 2012 (UTC)[reply]
Or the bidders are submitting binding offers, with the winning bid selected by lottery. It is not clear just from the passage quoted whether the winner, once selected, has the option of backing out of the purchase or not (if they do, it would be an option, if not, it wouldn't be.) --PalaceGuard008 (Talk) 09:28, 25 October 2012 (UTC)[reply]
Speaking purely as a reader (I think I may have formally studied the novel at school, but that was a looong time ago), I understood it to mean your second alternative, that the winner would have to pay an additional 5/- over and above his 2/6 raffle ticket in order to claim the prize: thus the organisers would (before expenses) gain 50 x 2/6 + 5/- = £6 10/-. {The poster formerly known as 87.85.230.195} 84.21.143.150 (talk) 13:27, 25 October 2012 (UTC)[reply]
For any reader wondering what on earth this is all about, our Caul articles says that it is "...is a piece of membrane that can cover a newborn's head and face immediately after birth." It continues; "A legend developed suggesting that possession of a baby's caul would give its bearer good luck and protect that person from death by drowning." The winner of David Copperfield's caul was an old lady who "... was never drowned, but died triumphantly in bed, at ninety-two." Alansplodge (talk) 17:46, 25 October 2012 (UTC)[reply]
Thanks, I was wondering about that, and why anyone would want a magician's caul. :-) StuRat (talk) 18:28, 25 October 2012 (UTC)[reply]

Roman Catholic incorporation?

In The Castilian Fathers at the Council of Basel, it states "Cardinal Cervantes incorporated on November 29, 1432 along with three members of his household." Obviously, he didn't go into business. What does this mean? Clarityfiend (talk) 05:30, 25 October 2012 (UTC)[reply]

I think what we're talking about here is College (canon law). Marnanel (talk) 05:43, 25 October 2012 (UTC)[reply]
Makes sense. Thanks. Clarityfiend (talk) 06:52, 25 October 2012 (UTC)[reply]
Resolved

The mechanics of the Imperial Diet of the HRE

So, I was just reading through some articles about the Holy Roman Empire, and while perusing through List of Reichstag participants (1792), I had some questions about how the Reichstag worked during this time period. Reading Imperial Diet (Holy Roman Empire) doesn't give a lot of insight as to the mechanics of how various Diets operated, excepting to note that "The precise role and function of the Imperial Diet changed over the centuries" without noting the operation of the Body. In List of Reichstag participants (1792) there are several things I have questions about regarding how that Diet would have operated, based on the voting membership:

  • Several "seats" in the Diet are held by the same person. For example, Archduke Maximilian Francis of Austria has several seats on the body: One as Elector of Cologne, a separate one as Bishop of Munster, and another as Grand Master of the Teutonic Knights. When Maximilian Francis officially recorded a vote, did it represent the vote of one man (as his vote) or the did it count as 3 votes (one for each office he held)?
  • Several members of the Diet are foreign princes who held important offices elsewhere. For example, many of the seats are held by George III of the United Kingdom through his various German titles. However, George never set foot in Hanover, and I don't believe he attended the Diet in person, which brings up the question of his votes: Could he "vote by mail", sending envoy of his official vote on various issues, or could he "vote by proxy" by sending a representative in his stead to represent his voice, and vote for him? And if so, did he send one envoy (to represent him) or several (to represent each state he had title to)? Or, if he wasn't personally present, did he forfeit his right to vote?
  • Some seats are held simultaneously by more than one person, sometimes many people combined for one vote. For example, the vote held by The Duke of Saxe-Coburg was officially split between two men: did they have to achieve consensus to register their single vote, or did each man represent 1/2 vote in the tally? The vote of The Prince of Anhalt is similarly divided among 4 people, and others similarly.
  • Regarding the "Colleges" Were they handled differently for the split votes of some Duchies noted above, which collectively represented a single vote? I suspect, from the wording of our articles, that the single vote of the College of The Counts of Swabia would be arrived at by the College voting amongst itself first, and then recording that as their one Vote in the diet? Or was there a representative elected from the college that served as the person who made the votes in the diet? Or did one of the members of the College have hereditary right to represent it in the diet?

Just some questions that come to mind. Any ideas or thoughts? --Jayron32 18:46, 25 October 2012 (UTC)[reply]

Erzherzog Maximilian Franz von Österreich didn't inherit any votes, he was a self-made Diet seat-holder. All his votes weren't hereditary. I'm guessing the votes were cast in order of precedence, the name of the seat was called out and the holder stood and cast his vote for all to hear. I'm guessing if the King of the United Kingdom of Great Britain and Ireland didn't send a deputy then he would lose his multple votes. I'm guessing the split votes would require agreement before the vote or the holders would alternate each vote. For the circles that held one vote, I'm guessing the majority of an internal vote was used or maybe rounds like the election of the Pope.
Sleigh (talk) 21:21, 25 October 2012 (UTC)[reply]
There is a category de:Kategorie:Funktionsweise Reichstag (HRR) with a couple of small articles or stubs on the functioning of the diet, the split votes were called Kuriatstimmen and the personal votes Virilstimmen. --Pp.paul.4 (talk) 22:01, 25 October 2012 (UTC)[reply]
Can you translate, perchance, the relevent bits? I don't have much German skills. Danke schoen. --Jayron32 22:09, 25 October 2012 (UTC)[reply]
My best guess is: (Question 1) Counted as 3 votes. (Question 2) I do not think that personal presence ever played a role. Everybody was represented. (Question 3) By consensus. (Question 4) Internal consensus reached first. Please correct. --Pp.paul.4 (talk) 22:55, 25 October 2012 (UTC)[reply]
What am I supposed to correct? Do you have any documents or sources which inform your guesses? --Jayron32 01:54, 26 October 2012 (UTC)[reply]

Old people over here

I just want to know whether there are old people on here. I'm 77 and I remember it clearly to hear on the radio about the death of last surviving veterans, such as Albert Woolson of the American Civil War, or Hugh Theodore Pinhey, of the Second Anglo-Afghan War, or other people's death such as that of Samuel J. Seymour, who was the last witness to the assassination of President Lincoln. I remember it all. Is there anyone on here with the same memories? Thank you. Iowafromiowa (talk) 19:12, 25 October 2012 (UTC)[reply]

I'm sure there are other people your age on here. For my part, I'm acutely aware that (if I'm spared) I will one day be in your position. As it is, I'm old enough to be slightly awed by the people here who don't remember the Cold War, Thatcher and Reagan, and so on. AlexTiefling (talk) 20:44, 25 October 2012 (UTC)[reply]
Then you'll be really awed by people like me, who don't remember 9/11. --140.180.252.244 (talk) 10:30, 26 October 2012 (UTC)[reply]
It's chilling to realise the Beatles formed half a century ago, and last performed 43 years ago. And Lennon's been dead for almost 32 years.
I remember going to Anzac Day marches where veterans from the Boer Wars marched, but I don't remember hearing when the last Boer veteran died. They didn't seem to make as big a thing of such "last events" back then as they do now.
I was born in the dying months of 1950. The first historical event I remember was the arrival of television in Australia in September 1956. We lived in the country and didn't have TV till about 1963, but all our Sydney relatives got them early, and it was always a big talking point when the next lot of rellos got a TV. Ironically, I have no memory of the 1956 Melbourne Olympics, which was why TV was necessary, but it would have been radio news in our case, rather than TV broadcasts, so .... I imagine the first time I actually saw television was at Christmas 1956 when we visited Sydney, but the Olympics were over by then. I clearly remember hearing about the launch of Sputnik 1 in October 1957. I remember often hearing on the radio about some dude named what sounded to my tender ears as "President Ivanhoe". I have a vague memory of hearing when Oliver Hardy died in 1957, and when Mike Todd was killed in March 1958 (but mainly because he was Elizabeth Taylor's husband - yes, she was very well known even to 7 year old boys). But the first clear memory I have of a notable person dying was that of Pope Pius XII in October 1958. Church bells rang unexpectedly, and we didn't have to come to school the next day. Yippee. -- Jack of Oz [Talk] 20:53, 25 October 2012 (UTC)[reply]
I was born in 1963. The first historical event I remember knowing about was Expo '70 in Japan, I wanted my parents to take me there from New Jersey ... when I visited Osaka in 2002, I was careful to visit the site. The first death of a notable person I remember was J. Edgar Hoover, I was home from school (1972) and it was on TV. I remember watching a moon landing on TV, not certain which one, but my late mother told me that I fell asleep for Armstrong's small step. I suppose I became aware more of the world in 1972, as I remember watching Avery Brundage's famous Munich speech on September 6, 1972 (perhaps it was on tape). Also, my father had a reel-to-reel audio and 8-track tapes, neither of which he used often.--Wehwalt (talk) 21:08, 25 October 2012 (UTC)[reply]
Though I prefer not to give my precise age, like JackofOz I'm a sexagenarian and pretty active here (though not nearly so much as Jack, sheesh). There's not anywhere nearly so many of us as there are the young whippersnappers, but we're represented (and a lot wiser, heh, heh, heh). I don't have the memories you do, you've got a few years on me, but I can remember some things just short of those. (43 years since the last Beatles performance? I hadn't realized that until now. Now I'm depressed...) Best regards, TransporterMan (TALK) 21:17, 25 October 2012 (UTC)[reply]
I listened live to the radio broadcasts of Marilyn Bell's swim across Lake Ontario. Bielle (talk) 21:22, 25 October 2012 (UTC)[reply]
Seriously, you guys make me depressed to think that one day, I'll be as old as you. I'm not old enough to remember any major historical events except the 2008 financial crisis; my earliest memory was right after 9/11, which I wouldn't even call a major event. The Cold War might as well be the Peloponnesian War from 2400 years ago, because that's how much I can relate to either (and actually, if you read Thucydides, it's depressing how similar the Athenian/Spartan justifications for war mirror those offered for the "hot" proxy wars by the US/USSR). Considering the murderous passions that historical conflicts seem to incite in people generations after they take place, I'm very glad that I don't remember any. --140.180.252.244 (talk) 21:29, 25 October 2012 (UTC)[reply]
Even more depressing is to think that in 20 years or so, your "nostalgia" radio channel will be playing such eternal classics as "Call Me Maybe" and "Gangnam Style". ←Baseball Bugs What's up, Doc? carrots02:37, 26 October 2012 (UTC)[reply]
Hey, those will be widely praised after their creators die, just like a lot of famous works. Also, radio channels? If every square centimeter of my beloved city doesn't have high-speed WiFi coverage by 2022, I'll make it happen. --140.180.252.244 (talk) 10:30, 26 October 2012 (UTC)[reply]
Funny, I just had a nightmare about a nuclear war two nights ago. That dates you. My earliest historical memories are of the last moon shot and Watergate. Elvis's death and Jonestown I remember clearly. John David Stutts shooting Reagan and Mehmet Ali Agca shooting the Pope happened a few months ago, and 9/11 last week. μηδείς (talk) 21:52, 25 October 2012 (UTC)[reply]
As you grow older, hopefully you'll come to realize it's even more depressing not to reach our Mathusalean age. Clarityfiend (talk) 22:22, 25 October 2012 (UTC)[reply]
After you have lived about maybe 80 years to 100 years then you will have a feeling of you have lived enough. Seriously, I wouldn't live pass 200 years, it is just too much. Death always has been the natural cycle of life, you can't escape it no matter what. Even if one day we, humans, somehow can make ourselves live forever then that would defy natural and to me it is not a good thing. And I'm very sure when you live to a certain point you WILL want to die. Even the most lively people will want to die "eventually". Maybe not after 1 thousand years or 1 million years or 1 billions or 1 trillions.... Eventually you will want to die, you got my point. Perhaps that was the reason Albert Einstein, a genius, who has well understood the meaning of life, refused to get a surgery to prolong his life (doesn't matter how long the surgery could have prolonged his live but the point here is he doesn't want to prolong his life even for 1 second). When it is time to go, just be happy to go, that's my way of taking it. Just live a meaningful life so when you died you will feel the fulfillment of life! That's all there is to the world! 184.97.240.247 (talk) 03:48, 26 October 2012 (UTC)[reply]
Malthusian? Methuselahian? -- Jack of Oz [Talk] 22:35, 25 October 2012 (UTC)[reply]
As in like Mathusal, Methuselah's older, illiterate brother. Clarityfiend (talk) 22:57, 25 October 2012 (UTC)[reply]
I remember watching The Day After as a college student (1983), everyone in the dorm got together and watched it in the lounge. Very somber, as I recall, lots of crying, even a few of the guys.--Wehwalt (talk) 00:48, 26 October 2012 (UTC)[reply]
I just watched that again over the summer, great film. And it did influence my nightmare. Probably the best work the KGB ever did. μηδείς (talk) 02:02, 26 October 2012 (UTC)[reply]
I think that would surprise the writer, who is (or was) a card-carrying Republican. Oh, I forgot: anything to the left of Genghis Khan is commie. --NellieBlyMobile (talk) 04:15, 26 October 2012 (UTC)[reply]
I am not a Republican, your comment on party affiliation is silly. It's documented that the KGB led a misinformation campaign aimed at getting the American left to support unilateral disarmament. Carl Sagan's politically premissed nuclear winter paper data was never published. According to Sergei Tretyakov,[30] the KGB held a party to celebrate the showing of The Day After, which was coordinated with the nuclear winter propaganda campaign and a special on ABC which ran the miniseries. As for political labels, I am not interested. If you think there's some sort of essential distinction between the murderous dictator Genghis Khan and the murderous dictator Joseph Stalin your thought is too subtle for me. Nevertheless, The Day After was a great movie. μηδείς (talk) 05:52, 26 October 2012 (UTC)[reply]
When I was young the world population was just over a third of what it is now. I lived in a jungle and spoke another language. Now the jungle is cut down and there is a town there. My father and mother went back there a few years ago and found a couple of pictures of themselves hung up in the town hall, the people were astonished some of the 'ancestors' were still alive! Dmcq (talk) 10:19, 26 October 2012 (UTC)[reply]

Why does Obama vote in Chicago?

Why not Washington, where he lives, or Hawaii, where he was born? OsmanRF34 (talk) 21:27, 25 October 2012 (UTC)[reply]

If he does, that is because Chicago is where he is registered. Bielle (talk) 21:29, 25 October 2012 (UTC)[reply]
The basic answer is that when a person has a temporary residence, like Obama does in DC or while you're in college or on an extended but temporary job assignment, the law in the US affords quite a bit of choice to you to decide what your permanent residence (i.e. your domicile) is going to be. That determines where you vote. Ordinarily, and most easily, you'll either choose between where you are now, with the idea that though it started out as being temporary you don't intend to leave when the temporary situation ends) or the last place you did live as a permanent residence. That, in Obama's case, was Chicago. Regards, TransporterMan (TALK) 21:38, 25 October 2012 (UTC)[reply]
"Choice" — in practice, maybe. In theory, no. States have definite rules about whether you are a resident or not, and choice does not enter into them. "Intent", on the other hand, does enter into them, but your intent is a question of fact, and in theory they could attempt to prove that you are a resident even though you say you're not (or vice versa). --Trovatore (talk) 21:55, 25 October 2012 (UTC)[reply]
College students frequently vote in the district of their permanent residence and not where they spend the night when on campus. Living in a dormitory on a college campus doesn't make a student automatically and unambiguously a resident of the district where the dorm is located, and not a resident of, say, their parent's home where they used to sleep most nights. Presidents traditionally always vote in their "official" home district, and not Washington DC. A quick perusal will easily show George W. Bush voting in Texas, Bill Clinton voting in Arkansas, etc, etc, back through time. Obama isn't doing anything that every president prior to him didn't do.--Jayron32 22:07, 25 October 2012 (UTC)[reply]
On the other hand, the courts have pretty regularly upheld that students are allowed to vote in their college precincts, even if they live in, for example, a dormitory [31]. It really is, in most cases up to the student what they consider to be their residence. Buddy431 (talk) 23:16, 25 October 2012 (UTC)[reply]
Your link doesn't indicate, at least literally, that a student can choose where he/she votes. It says it depends on whether the student intends to return to the parental home on graduation. Of course, in many cases that's not particularly well-defined, but in theory a student who clearly intends to return home after graduation is supposed to vote there rather than where the school is. And conversely, a student who definitely does not intend to return home is probably not supposed to vote there. --Trovatore (talk) 00:37, 26 October 2012 (UTC)[reply]
True. My hometown being in Salina, Kansas that's where I cast my ballot (via absentee ballot) even though I have the choice to register as a voter in Norman, Oklahoma. I've even seen voter registration drives on campus the past couple years. I feel like I have more of a say casting my ballot back home than I do at university, so I stay registered at home. Ks0stm (TCGE) 00:28, 26 October 2012 (UTC)[reply]
There's not much about it in our Wikipedia article, but in Texas Waller County is semi-notorious for repeatedly down the years coming up with semi-ingenious schemes to inconvenience students at a historically-black college in the county from voting there (a right which they indisputably have under relevant state laws)... AnonMoos (talk) 22:56, 25 October 2012 (UTC)[reply]
No one cares where they vote for president. The issue is voting for local officials by out-of-towners living off and with their parents. That's a concern in every college town I have lived in. μηδείς (talk) 23:11, 25 October 2012 (UTC)[reply]
"Out of towners" who live there the majority of the year? The courts have consistently upheld the rights of college students to vote in the elections of their school's district if that's where they consider themselves to be residents. Buddy431 (talk) 23:19, 25 October 2012 (UTC)[reply]
I am not arguing for or against, just pointing out that the concern does exist, since yes, the interests of temporary student residents and families and property owners can certainly clash. μηδείς (talk) 01:48, 26 October 2012 (UTC)[reply]
It becomes an issue when the kids have their parents' state's drivers license, which is a privilege of state citizenship (or, in a few cases, employment). That's one of the issues over voter ID laws.--Wehwalt (talk) 23:25, 25 October 2012 (UTC)[reply]
A driver's license is not a prerequisite for voting. I'll be voting this year in the state that I go to school, while my driver's license is from the state where I used to live. An ID at the polls (what most current voter ID laws deal with) is to prove that you are who you say you are, not that you're a resident of the precinct (they check that when you register to vote). Buddy431 (talk) 23:31, 25 October 2012 (UTC)[reply]
You might want to look into what your new state has by way of requiring that you get a local license when you move into a new state ... I think there's something about it in the interstate compact on drivers, which is probably part of your state's law ...--Wehwalt (talk) 23:42, 25 October 2012 (UTC)[reply]
Sure, my state has rules about how quickly I have to get a new driver's license (90 days). But that has nothing to do with whether or not I am eligible to vote. Buddy431 (talk) 00:04, 26 October 2012 (UTC)[reply]
Both are generally privileges of state citizenship, though different time periods for a change on moving are involved, and it is possible to get a driver's license from a state sometimes though you are not a resident.--Wehwalt (talk) 00:52, 26 October 2012 (UTC)[reply]
It's probably got more attention this time because he voted early, which I think is a first for a president. When the spotlight shines, it illuminates everything. -- Jack of Oz [Talk] 22:33, 25 October 2012 (UTC)[reply]
So, did he vote Green, or Socialist? μηδείς (talk) 23:08, 25 October 2012 (UTC)[reply]
straight Official Monster Raving Loony Party ticket. Blueboar (talk) 00:17, 26 October 2012 (UTC)[reply]

(removed, just saw the immediate preceding post) The Masked Booby (talk) 00:33, 26 October 2012 (UTC)[reply]

I hear he wanted to write in for vice president and they wouldn't let him.--Wehwalt (talk) 00:54, 26 October 2012 (UTC)[reply]
I heard... well actually, I didn't hear, because he voted by Secret ballot. He can write in whoever he wants, and they'll only determine the validity of the ballot and the vote anonymously, when all votes are tallied. Buddy431 (talk) 01:24, 26 October 2012 (UTC)[reply]
You can't actually write in for vice president as you are voting for electors, so it was meant humorously.--Wehwalt (talk) 02:03, 26 October 2012 (UTC)[reply]
I can write in for vice president on my Kansas ballot...if you chose to write in there are blank lines for you to write in both the president and vice president you're voting for. Ks0stm (TCGE) 02:19, 26 October 2012 (UTC)[reply]
Interesting. Can you vote Obama for pres but someone else besides Biden for vp?--Wehwalt (talk) 02:27, 26 October 2012 (UTC)[reply]
In my state, there are several paired choices, and then a single line for a write-in. Obviously, write-ins have no chance of winning the popular vote. But if they did, the state would have to somehow come up with a slate of electors. ←Baseball Bugs What's up, Doc? carrots02:35, 26 October 2012 (UTC)[reply]
I saw an odd possibility in the paper. If there is an exact 50-50 split in the electoral college, the election is decided by the two houses of Congress, which will presumably vote on party lines. The House of Representives elects the president (1 vote for each state, presumably decided by the party that controls that state's delegation) and the Senate elects the president. Anyway the numbers work out that the House is Republican controlled and would pick Romney, while the Senate is Democrat controlled and would pick Biden. So we'd have Romney as President and Biden as VP. What a weird thought. 67.119.3.105 (talk) 07:30, 26 October 2012 (UTC)[reply]

(Outdent.) Something has just occurred to me: suppose you lived (say) in both Philly and Chicago, because you were a college student. Would it be possible to have a vote in electing the mayors of both cities? If you would, is there any theoretical or practical reason you could not also have a vote in electing the presidential electors from both PA and IL? Marnanel (talk) 04:00, 26 October 2012 (UTC)[reply]

Is Chicago really known for its stringent election rules? --NellieBlyMobile (talk) 04:08, 26 October 2012 (UTC)[reply]
Well... --Jayron32 05:26, 26 October 2012 (UTC)[reply]
Off topic, but it might be interesting to people to know that in the UK, you can register to vote at more than one address if you spend "an equal amount of time at each". This is frequently done by students. In a general election, you can choose which of them to vote in (and you don't have to decide in advance, just turn up to one or other polling station on the day). In local elections, you can legally vote in both locations on the same day, if you so choose. --OpenToppedBus - Talk to the driver 10:20, 26 October 2012 (UTC)[reply]
Marnanel -- at one point I was registered in two different counties at the same time (though I didn't want to be). If I had voted in both counties for the same election, then I would have committed a crime... AnonMoos (talk) 10:36, 26 October 2012 (UTC)[reply]

October 26

I know this company was broken into 33 different companies. I wonder how the process happened. Like who are the 33 different people that each own a company? Did John D. Rockefeller choose those people? Can those 33 people are all Rockefeller's descendants? And if his company was broken into 33 different companies so is that mean he is going to lose 32 out of 33 amount of his fortune? 184.97.240.247 (talk) 02:08, 26 October 2012 (UTC)[reply]

Standard Oil wasn't owned by one person. It was a Joint-stock company which means it was owned by its shareholders. Anyone that owns stock in a company is an "owner". Rockefeller didn't own Standard Oil, as one might "own" a car. He owned a lot of shares of Standard Oil, was a founder of it, and served as its President and CEO, but founding a company is not the same thing as owning a Sole proprietorship. The specific ownership structure of Standard Oil in the early days is described at Standard Oil. Being the founder of a company is more like being the founder of a club: there are many stakeholders and many owners (those people who owned shares). Rockefeller owned enough shares to have a Controlling interest in the company. In his position as President and CEO, he directed the company through its major business decisions, but again it wasn't "his company" in the sense that he owned it as one owns an object. Standard Oil explains his role pretty well in the opening. It is also important to note that Standard Oil was basically a Holding company: it's main operation was the aquiring and integration of other companies into itself, or the founding of specific companies with specific purposes. So, Standard either bought an existing company, or would found a new company (wholly owned by Standard Oil) to cover some operation, either to market specific oil products in specific geographic locations, or to handle some aspect of the exploration, transportation, or processing of crude oil. Thus, the "break up" of Standard Oil wasn't arbitrary; the new companies already operated as semi-autonomous units within the Standard Oil Trust. What happened was that these companies were legally seperated into truly autonomous units, which were supposed to run seperately so they had to compete with each other and with other, non-Standard oil companies. The way this worked was that individual "Baby Standards" were granted regional monopolies over the use of the "Standard" name in a certain geographic area, but that's pretty much how Standard worked pre-break up. The only thing that really changed via the breakup was the corporate governance and ownership structure: The companies sold their own shares and had their own independent governance, and there was no industry-wide national coordination between them, though many of the companies did coordinate bilaterally (for example only a few of the Baby Standards were actual suppliers as well as marketers: Kyso stations sold Exxon and Mobil products which were often rebranded as "Kyso", but they were manufactured by Exxon or Mobil) this page isn't strictly a "reliable source" by Wikipedia standards, but it does have some good information on the history of the growth, domination, and later dissolution of the Standard Oil trust, as well as the modern fates of all of those companies. --Jayron32 02:54, 26 October 2012 (UTC)[reply]
Alright to my understanding of your explanation, let me sum it up. So before the breakup there were many small companies that were part of the Standard Oil and after the breakup they became independent companies. So do those small companies within the Standard Oil has to pay the Standard Oil to breakup? Did the Standard Oil gain any money from the breakup? Or did it actually lose trusts in the companies without being paid?184.97.240.247 (talk) 03:29, 26 October 2012 (UTC)[reply]
That's a good question. Since anti-trust court proceedings are civil and not criminal, it would seem that prima facie even without a direct link to answer your question, that the shareholders of Standard Oil could not have been deprived of their property without receiving equal value. So Rockefeller et al would have lost control of the Baby Standards from a corporate governance point of view, but they could not be deprived of the value of their shares when the breakup occured. As to the specific means of their compensation, I don't have any direct evidence yet (i'll look), but I suspect that there are a number of ways that could have happened. First, they could have been given fair market value of their shares; i.e. forceably "bought out" of their company by being provided cash at "fair market value" based on reasonable estimates of the value of the company: I.E. if Rockefeller had owned 30% of the company, he would have been given cash equal to %30 of the value of the company. The mechanics of how that cash would be acquired, and what would have happened to his shares (i.e. who would have bought them) sounds like a nightmare. The other possibility I can think of (one that seems more likely) is that he was given equivalent non-voting shares equivalent to his former position in the companies: He still owns the same value of shares (the same part of the companies) but as his shares are no longer voting shares, he has no say in corporate governance. I'll do some searching to see if I can find any harder sources on what actually happened. --Jayron32 04:55, 26 October 2012 (UTC)[reply]
This source from The Economist doesn't directly answer the question, but has an interesting quote:

On May 15th 1911, Rockefeller, again out on the golf course, was told that the Supreme Court had found the firm guilty of antitrust violations, and ordered it to be broken up. “Buy Standard Oil,” he advised his playing partner. A good tip: its pieces proved to be worth far more apart than together.

In other words, if my second scenario holds true, financially the windfall would have benefited Rockefeller greatly, as he stood to make a LOT of money on his shares in the new, separate companies, even if they were non-voting shares. Still looking for more. --Jayron32 04:59, 26 October 2012 (UTC)[reply]
This source of somewhat uncertain reliability, does state "The Court ordered the Standard Oil Trust to dismantle 33 of its most important affiliates and to distribute the stock to its own shareholders and not to a new trust." That is, the stock of the new companies was distributed to the existing shareholders of the old Trust. More evidence for my second explanation, though it doesn't indicate the nature of those shares (voting, nonvoting, common, preferred, etc.) --Jayron32 05:03, 26 October 2012 (UTC)[reply]
More: "Although Standard was physically broken up, Rockefeller maintained his one-quarter ownership in the business, only this time it was one-quarter of thirty-some new companies. J.P. Morgan was said to have remarked, "How the hell is any court going to compel a man to compete with himself?" The new set of companies continued to dominate the markets well into the 1930s." So more evidence of how the dissolution was handled. Stock exchange from the old Trust to the new individual companies seems like how it happened. --Jayron32 05:10, 26 October 2012 (UTC)[reply]
Read this book starting on page 106-107 largely confirms the last two sources. --Jayron32 05:18, 26 October 2012 (UTC)[reply]

Geographical information on ancient cities

I am looking for geographical information on ancient cities. Primarily urban layout maps of cities built around large stone temples or acropoli. I am wondering about any large stone structure that is centrally located in an urban area, such as greek cities. For example, I am to understand that scholars believe the Egyptian pyramids were built far away from cities rather than in them. Mayan pyramids, however, were built in the center of large cities settled on flat open ground. A large stone in the sunlight would get warmer than the textured urban area adjecent, and would therefore create a convection current which caused air to flow across the city even during times of zero or very low winds. I am researching the possibility that early humans may have been aware of the phenomenon and even planned it into their cities in order to improve sanitation. The greek acropolis might have had this effect, as would a pyramid or maybe even a castle. I am also looking for any information that might suggest this is a possibility, or not a possibility. Such an engineering design might become obsolete with the advent of flowing water, which would allow for a level of sanitation not possible before then, and therefore post roman era (in Europe) would cease to be important to residents of such an area. — Preceding unsigned comment added by 66.188.214.218 (talk) 04:04, 26 October 2012 (UTC)[reply]

I'm looking for information that would relate to the possibility that early human engineers were using large stone structures to create convection currents in the middle of urban areas on days with little to no wind. So, for example, a bunch of buildings have shade, indoors, fabric canopies, plants, etc. that would create a certain air temperature in sunlight, and the centrally located acropolis, pyramid, or other large temple would be exposed stone and therefore attain a higher temperature. This would cause the air in the urban area to flow, even at night as the heat stored in the stone was released, improving sanitation before flowing water became accessable via such means as the aqueduct. — Preceding unsigned comment added by 66.188.214.218 (talk) 04:14, 26 October 2012 (UTC)[reply]

That theory doesn't sound feasible to me. For one thing, you'd want the central rock/stone to be black, but it often seems to be white. Also, there are many other reasons for building up in the center, from defense in depth to impressing all the citizens living below. Also, air pollution wasn't much of a problem prior to industrialization and cities with huge, dense populations. StuRat (talk) 04:16, 26 October 2012 (UTC)[reply]

Ok, so the a white pyramid would work backwards, as well as provide access to upper air currents. And as to the smell of someone not taking a bath or shower, disposing of their waste in a chamber pot, owning several animals, and rarely doing laundry, I would say it might have caused health problems if the air wasn't being ventilated. So it could be possible that the smell was bad enough without the modern density. — Preceding unsigned comment added by 66.188.214.218 (talk) 04:39, 26 October 2012 (UTC)[reply]

(edit conflict) The theory fails Occam's razor in the sense that it proposes a complex and far-fetched explanation for events that have more commonplace and easy-to-understand explanations (sometimes this is known as the "Zebra solution": If you hear hooves, you shouldn't think "Zebra" instantly, you should think "Horse": look for the easier explanation, because it is usually right). In the case of cities built around hills, like Rome or the various Greek cities like Athens built around the Acropolis, the answer doesn't have anything to do with knowledge of convection currents. It has to do with knowledge of pointy sticks: Defending a static position is easier from the top of a hill than anywhere else. Large structures are often located within cities because large structures need lots of people to build them, and cities have lots of people in a central location: you don't have to move your labor force far away to build your temple or pyramid or whatever. Even the Giza Necropolis, though not at the center of a city, is not far from one. It was within easy distance of Memphis, the capital of the Old Kingdom and thus the administrative and commercial center of the Kingdom. Using a latitude/longitude calculator available online and the coordinates from our articles finds that Giza was 18 kilometers from Memphis, i.e. not downtown, but certainly within a reasonable distance to administer the work being done there. --Jayron32 04:42, 26 October 2012 (UTC)[reply]

One further question I consider in response. Suppose that there indeed was such a convection current (it would occur regardless of engineering, the heating patterns of a surface determine the effects of convection) and suppose it did indeed create a healthier more pleasant atmosphere for those in the central location, it might have caused the people who lived there to be more generally successful, as well as drive immigration by making an area more popular, making that city able to grow more than a competing city. — Preceding unsigned comment added by 66.188.214.218 (talk) 04:49, 26 October 2012 (UTC)[reply]

Well, again per the Zebra principle: Why search for an obscure, unlikely explanation for why people formed cities when there are a plethora of better reasons already availible. If you are generally interested in what actual historians have to actually say on the actual reasons why cities likely formed this page here [32] has a fairly easy to follow outline of the major factors in urban history and the development of cities, along with lots of good other sources to read. Wikipedia also has some good information at City#Origins. Does this help direct your thinking here? --Jayron32 05:47, 26 October 2012 (UTC)[reply]
There are plenty of ways to make your city more windy. Just putting it on a hill, like the Greek acropolis, would have made it much windier because no trees can block the wind. Building near a body of water helps, for the same reason the OP described, except that the heat sink is the entire ocean instead of a tiny piece of stone. See sea breeze and the corresponding land breeze. Narrow corridors like streets with buildings on both sides tend to channel the wind, which increases its speed. All of these effects dwarf any gain that could be had by placing a stone downtown. Needless to say, there are much more compelling reasons to build streets or locate your city on a hill and near an ocean. I'd be surprised if anybody who founded a city thought "oh, I'll build it on a hill to make it windier", instead of "I'll build it on a hill so the other dude doesn't kill me with his army".
Semi-random aside, because I've lately been obsessed with the Trojan War: the city of Troy was built on a hill, in between 2 rivers near their confluence, and beside a bay that extended from the Dardanelles (the bay has since dried up). The gods must have been with them, because that's just an amazing strategic location. Homer frequently calls the city "windy Troy". Gee, I wonder why... --140.180.252.244 (talk) 08:18, 26 October 2012 (UTC)[reply]
We have an article on acropoleis in general, which is not very informative, and some small articles on the Acropolis of Rhodes and the Acrocorinth. The Acropolis of Athens article has much more information. Cities were built on an acropolis mostly for practical reasons of defense. It's more difficult to attack a city on a hill (although not impossible, the Persians sacked the Athenian acropolis for example). Rome and Byzantium were also founded on a single hill, originally. Adam Bishop (talk) 10:32, 26 October 2012 (UTC)[reply]

Child slavery in nineteenth-century America.

My son wants to know how children his own age (4 years) laboured as American slaves. Did their work differ significantly from that of grown slaves, both in the field and in the house? If so, how?

(I note our article's reference to Gwyn Campbell, "Children and slavery in the new world: A review," Slavery & Abolition, Aug 2006, Vol. 27 Issue 2, pp 261–285, but I have not yet had opportunity to read this.)

Any advice of interest to a 4-year-old is especially welcome.82.31.133.165 (talk) 12:06, 26 October 2012 (UTC)[reply]